[obm-l] Re: [obm-l] Re: [obm-l] Comunicação

2023-08-27 Por tôpico Rogerio Ponce
Ola pessoal!

Nesta lista, da qual participamos, qualquer um (mesmo que não esteja
inscrito na lista) pode acessar os arquivos, fazer pesquisas e ler
todos os problemas e suas solucoes.

No whatsapp, isto seria impossivel, a menos que o individuo ja
estivesse participando desde o inicio.

[]'s
Rogerio Ponce

On Sat, Aug 26, 2023 at 1:36 PM Esdras Muniz  wrote:
>
> Seria muito legal se existisse.
>
> Em sex, 25 de ago de 2023 18:24, Priscila Santana  
> escreveu:
>>
>>
>> Olá!
>>
>> Existe algum grupo de discussão de questões olímpicas no WhatsApp?
>>
>> Atte.
>>
>> Priscila S. da Paz
>>
>>
>>
>> --
>> Esta mensagem foi verificada pelo sistema de antivírus e
>> acredita-se estar livre de perigo.
>
>
> --
> Esta mensagem foi verificada pelo sistema de antivírus e
> acredita-se estar livre de perigo.

-- 
Esta mensagem foi verificada pelo sistema de antiv�rus e
 acredita-se estar livre de perigo.


=
Instru��es para entrar na lista, sair da lista e usar a lista em
http://www.mat.puc-rio.br/~obmlistas/obm-l.html
=


[obm-l] Re: [obm-l] cadeira de 3 pés

2023-01-23 Por tôpico Rogerio Ponce
Ola' Claudio!
Eu diria que as duas explicações estão erradas, pois não se depende de
ter apenas um plano definido pelas pontas dos pés, visto que uma
cadeira de 4 pés pode, perfeitamente, ter as pontas dos 4 pés em
apenas um plano, e, ainda assim, ela não é necessariamente estável.

Um explicação menos ruim é que, numa cadeira de 3 pés, sempre podemos
apoiar quaisquer 2 pés num piso (mesmo irregular), e, em torno do eixo
definido pelos 2 pés já apoiados, podemos girar a cadeira até que o
terceiro pé encontre o piso, de modo que a cadeira fique totalmente
apoiada.
Já numa cadeira de 4 pés, é comum que um dos pés fique sem contato com
o chão, permitindo que a cadeira oscile em torno do eixo definido
pelos 2 pés vizinhos ao pé sem contato.

[]'s
Rogerio Ponce

On Sun, Jan 22, 2023 at 11:23 PM Claudio Buffara
 wrote:
>
> Achei na internet duas explicações distintas para a estabilidade de uma 
> cadeira (ou mesa ou banco) de 3 pés.
> Aqui estão:
> https://www.somatematica.com.br/curiosidades/c98.php
> http://colegiofarroupilha.com.br/site/qual-cadeira-e-mais-firme-a-que-tem-tres-ou-quatro-pes/
>
> Qual das duas é a explicação correta?
> Ou nenhuma das duas? E, nesse caso, qual a explicação?
>
> []s,
> Claudio.
>
> --
> Esta mensagem foi verificada pelo sistema de antivírus e
> acredita-se estar livre de perigo.

-- 
Esta mensagem foi verificada pelo sistema de antiv�rus e
 acredita-se estar livre de perigo.


=
Instru��es para entrar na lista, sair da lista e usar a lista em
http://www.mat.puc-rio.br/~obmlistas/obm-l.html
=


[obm-l] Re: [obm-l] Questão de probabilidade

2022-06-29 Por tôpico Rogerio Ponce
-

Se houvesse apenas um cachorro preto - aquele que ja' seria mesmo
preto, por definicao - a probabilidade de escolhermos um cachorro
preto seria justamente 1/8. Entretanto, existe chance de que algum
outro cachorro tambem seja preto, de modo que a probabilidade de
escolhermos um animal ao acaso, e ele ser um cachorro preto, e' maior
que 1/8, e ja' sabemos que a afirmacao 08 esta' errada.


Mas, por curiosidade, vamos calcular a probabilidade de, escolhendo-se
um animal ao acaso, ele ser um cachorro preto.
Entao a probabilidade procurada e' igual 'a probabilidade de
escolhermos o cachorro que ja' seria preto somada 'a probabilidade de
escolhermos um dos outros animais, e ele ser preto, e ele ser um
cachorro.
Ou seja, a probabilidade pedida vale
[ 1/8 ] + [ 7/8 * 2/7 * 4/7 ] =  1/8 + 1/7 = 15/56

Uma outra forma de se pensar:

A probabilidade pedida e' a probabilidade de escolhermos um cachorro,
multiplicada pela probabilidade de ele ser preto.
Entre os 8 animais, a probabilidade de escolhermos um cachorro vale 5/8.

E a probabilidade de ele ser preto e' igual a soma da probabilidade de
ele ser o cachorro definido inicialmente como preto, somada 'a
probabilidade de ele ser um dos outros caes, e receber a pintura
preta.

Em sendo um cachorro, a probabilidade de ele ser o cao definido
inicialmente como preto vale 1/5.
E a probabilidade deste cachorro ser um dos outros caes vale 4/5, e,
neste caso, a probabilidade deste animal receber a pintura preta vale
2/7, pois temos 2 pinturas pretas entre um total de 7 animais.

Entao, a probabilidade pedida vale:
5/8 *  (1/5 + 4/5 * 2/7) = 5/8 * (1/5 + 8/35) = 1/8 + 1/7 = 15/56

De modo que a afirmacao 08 esta' errada.



Afirmacao 16) Se um animal for escolhido ao acaso, a probabilidade de
ele ser um gato malhado e' de 1/16.

-

Entre as 8 pinturas, a probabilidade de pegarmos o animal com a
pintura malhada e' 1/8. E a probabilidade deste animal ser um gato
(lembrando que um dos caes ja' e' preto) e' 3/7.

Assim, a probabilidade de escolhermos um gato malhado vale (1/8) * (3/7) = 3/56.

Portanto, a afirmacao 16 esta' errada.



Ao final de tudo, vemos que a unica afirmacao correta e' a 04.

[]'s
Rogerio Ponce


===





 54,1 Topo

On Wed, Mar 16, 2022 at 8:08 AM Professor Vanderlei Nemitz
 wrote:
>
> Bom dia!
> Na questão a seguir, do vestibular da UEM, penso que o espaço amostral tem 
> 105 elementos, pois um cachorro é preto (desconsideramos esse). Porém, com 
> esse pensamento, não consigo obter o gabarito, que diz que 02 e 16 são 
> corretas.
> Alguém poderia ajudar?
> Muito obrigado!
>
> Em um pet shop há 3 gatos e 5 cães. Sabemos que 3 desses animais são pretos, 
> 4 são brancos e 1 é malhado. Além disso, pelo menos 1 cachorro é preto. 
> Assinale o que for correto.
> 01) A probabilidade de haver exatamente 1 cachorro preto é de 1/6.
> 02) A probabilidade de haver pelo menos 1 gato branco e pelo menos 2 
> cachorros brancos é de 2/3.
> 04) A probabilidade de haver um cachorro malhado é maior do que a 
> probabilidade de haver um gato malhado.
> 08) Se um animal for escolhido ao acaso, a probabilidade de ele ser um 
> cachorro preto é de 1/8.
> 16) Se um animal for escolhido ao acaso, a probabilidade de ele ser um gato 
> malhado é de 1/16.
>
> --
> Esta mensagem foi verificada pelo sistema de antivírus e
> acredita-se estar livre de perigo.

-- 
Esta mensagem foi verificada pelo sistema de antiv�rus e
 acredita-se estar livre de perigo.


=
Instru��es para entrar na lista, sair da lista e usar a lista em
http://www.mat.puc-rio.br/~obmlistas/obm-l.html
=


[obm-l] Re: [obm-l] Re: [obm-l] Re: [obm-l] Re: [obm-l] Re: [obm-l] Questão de probabilidade

2022-06-24 Por tôpico Rogerio Ponce
Otima explicacao!
Obrigado, Ralph!

PS: e sim, a provocacao foi pra voce mesmo!
:)

[]'s
Rogerio Ponce


On Wed, Jun 22, 2022 at 1:00 PM Ralph Costa Teixeira  wrote:
>
> Ponce está provocando a gente... senti que esta flecha tinha um bocado a 
> minha direção...  :D :D :D
>
> Olha, tem duas "visões" sobre o que "probabilidade" significa.
>
> A primeira vai na linha de que só podemos falar de probabilidade sobre coisas 
> que ainda não aconteceram. Vai nessa linha: se os eventos estão no passado, 
> então já aconteceram, já estão definidos, e não faz sentido dizer que tinha x 
> de chance de ser assim ou y de ser assado. Se você já jogou a moeda justa, 
> não é mais 50/50 --  é 100% de ser cara, ou 100% de ser coroa, dependendo do 
> que ocorreu. Quem pensa assim vai dizer que dado um certo evento (sempre no 
> futuro), ele tem uma probabilidade dada; se duas pessoas diferentes derem 
> duas probabilidades diferentes para o mesmo evento, uma delas errou.
>
> Outra linha diz que podemos falar de probabilidade sempre que houver 
> incerteza; não interessa o que aconteceu ou o que vai acontecer, o que 
> interessa é o que você SABE sobre o acontecimento. Se você jogou a moeda 
> justa mas eu não sei nada mais sobre o lançamento, continua sendo 50/50 
> **PARA MIM**. Probabilidade passa a ser um conceito sobre INFORMAÇÃO, não 
> sobre os fatos em si (a probabilidade não está na moeda, está no que você 
> sabe sobre a moeda). Quem pensa assim vai dizer que a probabilidade do evento 
> depende não apenas do evento em si, mas da informação que se tem em mãos. 
> Quem pensa assim admite que duas pessoas diferentes podem dar probabilidades 
> diferentes ao mesmo evento SE SOUBEREM FATOS DIFERENTES a respeito do evento, 
> ou seja, probabilidade passa a ser bastante "subjetivo".
>
> Eu talvez tenha descrito mal a primeira interpretação, pois sou ferrenho 
> defensor da segunda. Ela simplesmente engloba a primeira, porque você pode 
> ter informação parcial sobre fatos que ocorrem no futuro. E falar de 
> probabilidade para descrever incerteza presente ou passada é MUITO útil! Eu 
> quero poder expressar incerteza sobre fatos passados com frases do tipo 
> "fulano tem x% de probabilidade de ter cometido tal crime", ou "tem y% de 
> probabilidade de ter petróleo nesse poço", ou "tem z% de chance de eu ter 
> COVID"... Se você tem uma reação negativa a essas frases, lembre o que elas 
> realmente significam (na segunda interpretação): claro que ou o cara cometeu 
> o crime ou não, não faz sentido dizer que ele cometeu o crime x% das vezes em 
> que fizermos um experimento de ele cometer o crime... mas o que aquilo 
> significa é "com a informação que eu tenho, numa escala de 0 a 1, eu tenho 
> x/100 de certeza que fulano cometeu o crime". E "certeza baseada em informaç�!
 �es" é sim quantificável -- e satisfaz exatamente as leis das probabilidades 
com as quais concordamos. "Subjetivo" não significa "posso falar qualquer 
coisa", significa apenas que a conta pode variar de pessoa para pessoa... mas, 
de novo, SE ESSAS PESSOAS TIVEREM INFORMAÇÕES DIFERENTES sobre o evento.
>
> Abraço, Ralph.
>
> On Wed, Jun 22, 2022 at 12:09 PM Rogerio Ponce  wrote:
>>
>> Olá Pedro e pessoal da lista!
>>
>> Segundo a opinião do Pedro, nao faz sentido perguntar qual a probabilidade 
>> de Jose ter conseguido um 6 ao jogar o dado ontem, pois isso ja' aconteceu, 
>> e, portanto, ja' esta' definido.
>>
>> Sera' que e' isso mesmo?
>>
>> []'s
>> Rogerio Ponce
>>
>>
>> On Mon, Jun 20, 2022 at 9:45 PM Pedro José  wrote:
>>>
>>> Eu na minha humilde opinião creio que a probabilidade exista quando pode 
>>> ser uma coisa ou outra. No caso já é definido o que os animais são. Então 
>>> já está tudo errado. A questão seria viável se dessem esses limitantes para 
>>> uma criança que pintaria os desenhos dos animais. Aí sim há probabilidade.
>>>
>>> Em sáb., 18 de jun. de 2022 03:33, Rogerio Ponce  
>>> escreveu:
>>>>
>>>> Ola' Vanderlei e pessoal da lista!
>>>>
>>>> Sem perda de generalidade, podemos imaginar que vamos fazer o seguinte:
>>>>
>>>> - uma pintura preta em um dos caes, escolhido aleatoriamente
>>>>
>>>> - uma pintura "malhada" em um dos animais, escolhido aleatoriamente entre 
>>>> os 7 animais nao pintados
>>>>
>>>> - duas pintura pretas, em dois animais, escolhidos aleatoriamente entre os 
>>>> 6 animais restantes,
>>>>
>>>> - quatro pintur

[obm-l] Re: [obm-l] Re: [obm-l] Re: [obm-l] Questão de probabilidade

2022-06-22 Por tôpico Rogerio Ponce
Olá Pedro e pessoal da lista!

Segundo a opinião do Pedro, nao faz sentido perguntar qual a probabilidade
de Jose ter conseguido um 6 ao jogar o dado ontem, pois isso ja' aconteceu,
e, portanto, ja' esta' definido.

Sera' que e' isso mesmo?

[]'s
Rogerio Ponce


On Mon, Jun 20, 2022 at 9:45 PM Pedro José  wrote:

> Eu na minha humilde opinião creio que a probabilidade exista quando pode
> ser uma coisa ou outra. No caso já é definido o que os animais são. Então
> já está tudo errado. A questão seria viável se dessem esses limitantes para
> uma criança que pintaria os desenhos dos animais. Aí sim há probabilidade.
>
> Em sáb., 18 de jun. de 2022 03:33, Rogerio Ponce da Silva <
> abrlw...@gmail.com> escreveu:
>
>> Ola' Vanderlei e pessoal da lista!
>>
>> Sem perda de generalidade, podemos imaginar que vamos fazer o seguinte:
>>
>> - uma pintura preta em um dos caes, escolhido aleatoriamente
>>
>> - uma pintura "malhada" em um dos animais, escolhido aleatoriamente entre
>> os 7 animais nao pintados
>>
>> - duas pintura pretas, em dois animais, escolhidos aleatoriamente entre
>> os 6 animais restantes,
>>
>> - quatro pinturas brancas nos 4 animais restantes
>>
>>
>> Analisando a afirmacao 04, por exemplo, verificamos que, no segundo passo
>> (pintura malhada) existem 4 opcoes de cachorro e 3 opcoes de gato.
>>
>> Assim, a probabilidade de haver um cachorro malhado (4/7) e' maior que a
>> probabilidade de haver um gato malhado (3/7).
>> Portanto, a afirmacao 04 esta' correta.
>> (e o gabarito esta' errado).
>>
>> []'s
>> Rogerio Ponce
>>
>>
>>
>> On Wed, Mar 16, 2022 at 8:08 AM Professor Vanderlei Nemitz <
>> vanderma...@gmail.com> wrote:
>>
>>> Bom dia!
>>> Na questão a seguir, do vestibular da UEM, penso que o espaço amostral
>>> tem 105 elementos, pois um cachorro é preto (desconsideramos esse). Porém,
>>> com esse pensamento, não consigo obter o gabarito, que diz que 02 e 16 são
>>> corretas.
>>> Alguém poderia ajudar?
>>> Muito obrigado!
>>>
>>> *Em um pet shop há 3 gatos e 5 cães. Sabemos que 3 desses animais são
>>> pretos, 4 são brancos e 1 é malhado. Além disso, pelo menos 1 cachorro é
>>> preto. Assinale o que for correto. *
>>> *01) A probabilidade de haver exatamente 1 cachorro preto é de 1/6. *
>>> *02) A probabilidade de haver pelo menos 1 gato branco e pelo menos 2
>>> cachorros brancos é de 2/3.*
>>> *04) A probabilidade de haver um cachorro malhado é maior do que a
>>> probabilidade de haver um gato malhado. *
>>> *08) Se um animal for escolhido ao acaso, a probabilidade de ele ser um
>>> cachorro preto é de 1/8. *
>>> *16) Se um animal for escolhido ao acaso, a probabilidade de ele ser um
>>> gato malhado é de 1/16.   *
>>>
>>> --
>>> Esta mensagem foi verificada pelo sistema de antivírus e
>>> acredita-se estar livre de perigo.
>>
>>
>> --
>> Esta mensagem foi verificada pelo sistema de antivírus e
>> acredita-se estar livre de perigo.
>
>
> --
> Esta mensagem foi verificada pelo sistema de antivírus e
> acredita-se estar livre de perigo.

-- 
Esta mensagem foi verificada pelo sistema de antiv�rus e
 acredita-se estar livre de perigo.



[obm-l] Re: [obm-l] Questão de probabilidade

2022-06-17 Por tôpico Rogerio Ponce da Silva
Ola' Vanderlei e pessoal da lista!

Sem perda de generalidade, podemos imaginar que vamos fazer o seguinte:

- uma pintura preta em um dos caes, escolhido aleatoriamente

- uma pintura "malhada" em um dos animais, escolhido aleatoriamente entre
os 7 animais nao pintados

- duas pintura pretas, em dois animais, escolhidos aleatoriamente entre os
6 animais restantes,

- quatro pinturas brancas nos 4 animais restantes


Analisando a afirmacao 04, por exemplo, verificamos que, no segundo passo
(pintura malhada) existem 4 opcoes de cachorro e 3 opcoes de gato.

Assim, a probabilidade de haver um cachorro malhado (4/7) e' maior que a
probabilidade de haver um gato malhado (3/7).
Portanto, a afirmacao 04 esta' correta.
(e o gabarito esta' errado).

[]'s
Rogerio Ponce



On Wed, Mar 16, 2022 at 8:08 AM Professor Vanderlei Nemitz <
vanderma...@gmail.com> wrote:

> Bom dia!
> Na questão a seguir, do vestibular da UEM, penso que o espaço amostral tem
> 105 elementos, pois um cachorro é preto (desconsideramos esse). Porém, com
> esse pensamento, não consigo obter o gabarito, que diz que 02 e 16 são
> corretas.
> Alguém poderia ajudar?
> Muito obrigado!
>
> *Em um pet shop há 3 gatos e 5 cães. Sabemos que 3 desses animais são
> pretos, 4 são brancos e 1 é malhado. Além disso, pelo menos 1 cachorro é
> preto. Assinale o que for correto. *
> *01) A probabilidade de haver exatamente 1 cachorro preto é de 1/6. *
> *02) A probabilidade de haver pelo menos 1 gato branco e pelo menos 2
> cachorros brancos é de 2/3.*
> *04) A probabilidade de haver um cachorro malhado é maior do que a
> probabilidade de haver um gato malhado. *
> *08) Se um animal for escolhido ao acaso, a probabilidade de ele ser um
> cachorro preto é de 1/8. *
> *16) Se um animal for escolhido ao acaso, a probabilidade de ele ser um
> gato malhado é de 1/16.   *
>
> --
> Esta mensagem foi verificada pelo sistema de antivírus e
> acredita-se estar livre de perigo.

-- 
Esta mensagem foi verificada pelo sistema de antiv�rus e
 acredita-se estar livre de perigo.



[obm-l] Combinatoria - quantas sequencias de comprimento "n" , com "p" elementos

2018-02-27 Por tôpico Rogerio Ponce da Silva
Ola' pessoal !
Existem quantas sequencias (diferentes entre si) de comprimento "n" ,
empregando-se somente "p" elementos, pelo menos uma vez cada um deles?

[]'s
Rogerio Ponce

-- 
Esta mensagem foi verificada pelo sistema de antiv�rus e
 acredita-se estar livre de perigo.

=
Instru��es para entrar na lista, sair da lista e usar a lista em
http://www.mat.puc-rio.br/~obmlistas/obm-l.html
=


[obm-l] Re: [obm-l] Duas questões de matemática.

2016-08-08 Por tôpico Rogerio Ponce
Ola' Douglas, a questao me parece perfeita.
Como as opcoes de resposta sao positivas, queremos a menor quantidade de
derrotas (ou seja, a maior quantidade de vitorias), que leve ao mesmo total
de premios.
Portanto, estamos falando das derrotas de maior valor (foram as 4 ultimas),
acompanhadas por uma com o valor necessario para completar a soma total.
Se considerassemos as 5 ultimas derrotas, o valor total seria ultrapassado.
So' pode ser a letra "E".
[]'s
Rogerio Ponce

2016-08-08 16:45 GMT-03:00 Douglas Oliveira de Lima <
profdouglaso.del...@gmail.com>:

> Olá amigos, gostaria de uma ajuda em uma filosofia e uma questão.
>
> 1)Na definição de ângulos suplementares, seria para dois ângulos ou pode
> ser para mais de dois?
>
> 2)(Essa questão gostaria de saber se está mal elaborada) Carlos e Ricardo
> disputaram 15 partidas de boliche e ao fim de cada partida o perdedor
> pagava um prêmio em dinheiro para o vencedor. O prêmio para a primeira
> partida foi R$ 15,00 e o prê- mio de cada partida seguinte foi R$ 5,00 a
> mais do que o valor da partida anterior. Ao final da disputa, ambos
> receberam o mesmo valor em dinheiro e nenhuma partida terminou empatada.
> Nes- sas condições, a maior diferença possível entre as vitórias e as
> derrotas de Ricardo é
> (A) 4. (B) 3. (C) 7. (D) 6. (E) 5.
>
> Att: Douglas Oliveira.
>
> --
> Esta mensagem foi verificada pelo sistema de antivírus e
> acredita-se estar livre de perigo.

-- 
Esta mensagem foi verificada pelo sistema de antiv�rus e
 acredita-se estar livre de perigo.



[obm-l] Re: [obm-l] Re: [obm-l] Indução dúvida

2016-01-19 Por tôpico Rogerio Ponce
Ola' pessoal,
me parece que a forma de pensar do Israel esta' perfeita.

A duvida dele se refere ao salto "se P(n) e' verdadeira" entao "P(n+1) e'
verdadeira".
Pois ele supos que se P(n) vale, entao, se P(n+1) fosse falsa, e ele
obtivesse a contradicao de que P(n+1) e' verdadeira, entao o salto estaria
provado.
E isto esta' correto.

[]'s
Rogerio Ponce


2016-01-18 23:30 GMT-02:00 Ralph Teixeira :

> Oi, Israel.
>
> Realmente muita gente faz essa confusao. Voce quer provar que
>
> "Para todo n natural, P(n) eh VERDADEIRA."
>
> O metodo de inducao, em sua versao mais simples, diz que basta mostrar
> duas coisas:
>
> i) P(1) eh VERDADEIRA
> ii) Para todo k natural,  (P(k)->P(k+1)).
>
> Note com cuidado onde estao os parenteses no item (ii): ele nao pede para
> provar que "[Para todo n natural, P(n) eh VERDADEIRA] -> [Para todo n
> natural, P(n+1) eh VERDADEIRA]", o que realmente seria obvio! Voce supoe
> que P(k) eh verdadeira para um k ESPECIFICO (mas arbitrario, deixe faca o
> raciocinio usando a variavel "k", nao troque por um numero) e quer mostrar
> que, se P(k) for verdadeira para ESTE k especifico, entao ela eh verdadeira
> para o proximo numero especifico, que seria k+1.
>
> Eh ateh por isto que eu prefiro escrever o (ii) com uma letra k ao inves
> de n, para nao dar confusao.
>
> Abraco, Ralph.
>
> P.S.: Se voce preferir, pode pensar assim: voce tem que provar que
> i) P(1) vale
> ii) P(1) -> P(2)
> iii) P(2) -> P(3)
> iv) P(3) -> P(4)
> e "assim por diante". Agora, gracas ao poder das variaveis, voce pode
> provar todas as linhas a partir de (ii) numa tacada soh, provando que
> ii,iii,iv,...) P(k) -> P(k+1)
> onde k eh um numero arbitrario (bom, do conjunto {1,2,3,4,...}).
>
> 2016-01-18 15:30 GMT-02:00 Israel Meireles Chrisostomo <
> israelmchrisost...@gmail.com>:
>
>> Em uma prova por indução, eu devo provar que P(n) implica P(n+1).Eu posso
>> fazer isso da seguinte forma: suponha que P(n) é verdadeira, e suponha que
>> P(n+1) é falsa, mas ao supor que P(n) é verdadeira e P(n+1) é falsa isto
>> implica que P(n+1) é verdadeira(contradição, pois supomos que P(n+1) é
>> falsa e no entanto é verdadeira, uma proposição não pode ser falsa e
>> verdadeira ao mesmo tempo)-tendo em vista que já provei o caso base, isto
>> pode ser considerado uma prova?Isto me pareceu correto, mas não sei se está
>> correto.Eu bem sei que posso provar a contra positiva, que é o caso
>> "inverso" ao que eu estou falando.Mas esse caso também é uma prova?
>>
>
>


[obm-l] Re: [obm-l] Re: [obm-l] Re: [obm-l] Re: [obm-l] Re: [obm-l] Determinante máximo

2015-08-25 Por tôpico Rogerio Ponce
Ihhh Bernardo, e' verdade !!!
Esqueci cofatores & cia. O que me veio 'a mente foi justamente a imagem do
processo para matrizes 3x3 que, bobamente, estendi para 4x4.
Abracos,
Rogerio Ponce

2015-08-25 23:02 GMT-03:00 Bernardo Freitas Paulo da Costa <
bernardo...@gmail.com>:

> 2015-08-25 22:48 GMT-03:00 Rogerio Ponce :
> > Ola' Bernardo,
>
> Oi Rogério.
>
> > usando a mesma pintura de um tabuleiro de xadrez, temos a diagonal
> principal
> > branca, e a diagonal secundaria preta.
> >
> > No caso dessa matriz 4x4, uma forma de se visualizar os termos que devem
> ser
> > multiplicados entre si (para obtermos cada uma das 8 parcelas do
> > determinante) e' a seguinte:
> >
> > a gente escreve 2 vezes a mesma matriz (uma ao lado da outra), e entao
> > tomamos os 4 termos da diagonal principal, e tambem os 4 termos de mais 3
> > linhas paralelas 'a diagonal principal (que, a exemplo do tabuleiro de
> > xadrez, tambem vou chamar de diagonais);
> >
> > e depois executamos um procedimento semelhante com a diagonal secundaria,
> > obtendo as 8 parcelas a serem somadas.
>
> Esse procedimento prático para calcular determinantes só vale em
> matrizes 3x3. Não vale em matrizes 2x2 (senão daria zero, como você
> pode verificar) e em matrizes 4x4 isso dá apenas 8 termos, quando na
> verdade há 4! = 24 termos no determinante.
>
> Não sei porque ainda se ensina isto, porque é um caso muito
> particular, e acaba gerando confusão quando se tenta ensinar o caso
> geral. Além de ser muito menos poderoso e versátil do que a definição
> por
> - Recorrência (Lagrange, expansão em linhas ou colunas, bom quando tem
> muitos zeros)
> - Eliminação de Gauss (que é cúbico, e é melhor quando a matriz é cheia)
>
> Abraços,
> --
> Bernardo Freitas Paulo da Costa
>
> --
> Esta mensagem foi verificada pelo sistema de antivírus e
>  acredita-se estar livre de perigo.
>
>
> =
> Instru�ões para entrar na lista, sair da lista e usar a lista em
> http://www.mat.puc-rio.br/~obmlistas/obm-l.html
> =
>

-- 
Esta mensagem foi verificada pelo sistema de antiv�rus e
 acredita-se estar livre de perigo.



[obm-l] Re: [obm-l] Re: [obm-l] Re: [obm-l] Determinante máximo

2015-08-25 Por tôpico Rogerio Ponce
Ola' Bernardo,
usando a mesma pintura de um tabuleiro de xadrez, temos a diagonal
principal branca, e a diagonal secundaria preta.

No caso dessa matriz 4x4, uma forma de se visualizar os termos que devem
ser multiplicados entre si (para obtermos cada uma das 8 parcelas do
determinante) e' a seguinte:

a gente escreve 2 vezes a mesma matriz (uma ao lado da outra), e entao
tomamos os 4 termos da diagonal principal, e tambem os 4 termos de mais 3
linhas paralelas 'a diagonal principal (que, a exemplo do tabuleiro de
xadrez, tambem vou chamar de diagonais);

e depois executamos um procedimento semelhante com a diagonal secundaria,
obtendo as 8 parcelas a serem somadas.

Pois bem, suponhamos que os termos da diagonal principal sejam diferentes
de zero.
Para conseguirmos o determinante maximo, devemos tentar construir o maior
numero de parcelas diferentes de zero, e isto implica em "reaproveitarmos"
(no calculo dessas outras parcelas) alguns dos termos que colocamos na
diagonal principal.
Como a diagonal principal e' branca, todos os 4 termos (diferentes de zero)
restantes tambem devem ser colocados em casas brancas (pois se uma diagonal
contem algum termo da diagonal principal, entao ela tambem e' branca).

Agora, sabemos*** (vide obs) que as casas pretas sao zero, e fica claro que
o teto para o valor maximo e' 4.
Entao, o problema passa a ser como distribuir "+1" e "-1" entre as casas
brancas, de modo que todas as parcelas contribuam positivamente.

A diagonal principal pode ser 1,1,1,1 ou -1,-1,-1,-1 ou 1,-1,-1,1 ou
1,-1,1,-1.
(as outras combinacoes sao equivalentes).
E, a partir deste ponto, eu fiz por inspecao.

OBS:
sabemos*** : de fato nao sabemos, estamos apenas chutando, e sendo
otimistas.
Poderia ser que esse procedimento nos levasse sempre a um determinante
igual a zero, por exemplo. O que realmente sabemos e' que o teto e' 4, e
caso seja possivel alcanca-lo, o caminho e' este.

[]'s
Rogerio Ponce


2015-08-19 6:54 GMT-03:00 Bernardo Freitas Paulo da Costa <
bernardo...@gmail.com>:

> 2015-08-18 23:56 GMT-03:00 Rogerio Ponce :
> > Ola' Eduardo Henrique,
> > imagine o quadrado 4x4 pintado como um tabuleiro de xadrez.
> > Para aproveitarmos ao maximo os valores diferentes de zero, eles precisam
> > estar todos nas 8 casas de mesma cor.
>
> Faz um certo sentido, mas eu não sei muito bem porquê. Ainda mais,
> como o determinante é invariante por permutação de linhas e colunas (a
> menos de sinal), se você trocar a segunda linha com a terceira, e a
> segunda coluna com a terceira, o determinante é igual mas o padrão
> muda.
>
> > Entao o problema se transforma em distribuir estes 8 valores de forma
> que as
> > 4 parcelas (diferentes de zero) sejam "favoraveis".
> > Assim, o maior valor do determinante seria 4, mas precisamos conseguir
> uma
> > arrumacao conveniente.
> > Esta daqui, por exemplo, e' suficiente:
> > 1   0   1   0
> > 0  -1   0  -1
> > 1   0  -1   0
> > 0  -1   0   1
> >
> > []'s
> > Rogerio Ponce
>
> Eu fiz as contas no computador, e realmente o máximo do determinante é
> 4. Eu estou pensando num argumento com "volumes". O determinante de
> uma matriz é sempre menor do que o produto das normas dos vetores
> linha (ou coluna) dela. Assim, como temos 8 números diferentes de
> zero, faz sentido botar dois em cada linha para que a norma de cada
> uma seja raiz(2), e daí o produto é 4. Se não fizermos isso, alguma
> linha fica com "mais" e outra com menos. Como toda linha tem que ter
> pelo menos um diferente de zero (senão o det=0), ficam as
> possibilidades (para os quadrados)
>
> 2,2,2,2 -> prod = 16, det = 4
> 3,2,2,1 -> prod = 12, det = 2 raiz(3)
> 3,3,1,1 -> prod = 9, det = 3
>
> Isso mostra que o det <= 4, qualquer que seja o caso, e daí "basta"
> achar um caso em que dá certo.
>
> Quanto mais "iguais", maior o produto (tem uma desigualdade das médias
> escondida?)
>
> Abraços,
> --
> Bernardo Freitas Paulo da Costa
>
> --
> Esta mensagem foi verificada pelo sistema de antivírus e
>  acredita-se estar livre de perigo.
>
>
> =
> Instru�ões para entrar na lista, sair da lista e usar a lista em
> http://www.mat.puc-rio.br/~obmlistas/obm-l.html
> =
>

-- 
Esta mensagem foi verificada pelo sistema de antiv�rus e
 acredita-se estar livre de perigo.



[obm-l] Re: [obm-l] Determinante máximo

2015-08-18 Por tôpico Rogerio Ponce
Ola' Eduardo Henrique,
imagine o quadrado 4x4 pintado como um tabuleiro de xadrez.
Para aproveitarmos ao maximo os valores diferentes de zero, eles precisam
estar todos nas 8 casas de mesma cor.
Entao o problema se transforma em distribuir estes 8 valores de forma que
as 4 parcelas (diferentes de zero) sejam "favoraveis".
Assim, o maior valor do determinante seria 4, mas precisamos conseguir uma
arrumacao conveniente.
Esta daqui, por exemplo, e' suficiente:
1   0   1   0
0  -1   0  -1
1   0  -1   0
0  -1   0   1

[]'s
Rogerio Ponce


2015-08-18 10:20 GMT-03:00 Eduardo Henrique :

> Amigos, alguma ideia de como resolver isso:
>
> Se tem uma matriz 4x4 com 8 0's, 4 1's e 4 -1's, qual o maior valor pro
> determinante dela?
>
> Att.
>
> Eduardo
>
> --
> Esta mensagem foi verificada pelo sistema de antivírus e
> acredita-se estar livre de perigo.
>

-- 
Esta mensagem foi verificada pelo sistema de antiv�rus e
 acredita-se estar livre de perigo.



Re: [obm-l] Problema

2015-07-07 Por tôpico Rogerio Ponce
Ola' Benedito,
Em modulo 5, existem cinco zeros, cinco grupos com "1,2,3,4" (onde 1 e'
complemento de 4, e 2 e' complemento de 3) , e o grupo "1,2".

O jogador "A" vence se chegar ao final com um par complementar (em modulo
5), e mais um numero qualquer, pois basta que ele entao apague este numero.

Assim, o jogador "A" comeca apagando o "1", por exemplo.
Imagine que este "1" pertencia ao grupo "1,2".
Ficou sobrando um "2", que pode ser associado a um dos zeros, formando um
par que vou chamar de "par estranho".

Agora, alem desse par estranho "0,2" , existem doze grupos de pares
complementares ( dos tipos "0,0" , "1,4" e "2,3" ).
A partir de entao, a cada jogada de "B", "A" apaga o complemento.

Observe que quando "B" apagar o primeiro dos cinco "0" existentes, "A"
considera que este zero pertence ao par estranho, e apaga o "2" associado.
Da mesma forma, se "B" apagar o primeiro dos seis "2" existentes, "A"
considera que este "2" pertence ao par estranho, e entao apaga o "0"
associado.

Ou seja, sempre que "B" apagar um numero, "A" apaga o complemento.
Ao final, sempre sobrara' um par complementar.

[]'s
Rogerio Ponce

2015-07-06 14:39 GMT-03:00 benedito freire :

> Qual é realmente a estratégia para vencer?
> --
> De: Mauricio de Araujo 
> Enviada em: ‎01/‎07/‎2015 14:24
> Para: obm-l@mat.puc-rio.br
> Assunto: Re: [obm-l] Problema
>
> ​ou melhor, A deve evitar enquanto puder apagar algum múltiplo de 5.​
>
> Em 1 de julho de 2015 14:21, Mauricio de Araujo <
> mauricio.de.ara...@gmail.com> escreveu:
>
>> A não deve apagar nenhum múltiplo de 5.
>>
>> Em 1 de julho de 2015 14:19, Mauricio de Araujo <
>> mauricio.de.ara...@gmail.com> escreveu:
>>
>>> ​Ao final do jogo, A terá apagado 13 números e B 12 números (para que
>>> sobre 2 números)... a estratégia vencedora de B seria apagar todos os
>>> números 3(mod5) e 4(mod5) além de 3 números 0(mod5) dos quatro existentes,
>>> ou seja, teria de executar 13 ações de apagar... como ele só joga 12 vezes
>>> A vence sempre (desde que jogue com cuidado)..​
>>>
>>> Em 1 de julho de 2015 13:30, Pedro José  escreveu:
>>>
>>>> Bom dia !
>>>> Está errado o jogador pode escolher a sobra de E ou F antes de cabarem
>>>> todos os números. Necessita de reanálise.
>>>> -- Mensagem encaminhada --
>>>> De: Pedro José 
>>>> Data: 1 de julho de 2015 10:54
>>>> Assunto: Re: [obm-l] Problema
>>>> Para: obm-l@mat.puc-rio.br
>>>>
>>>>
>>>>
>>>> Bom dia!
>>>>
>>>>
>>>> E={1,6,11,16,21,26} e F= {4,9,14,19,24} Para qualquer par (a,b) com a
>>>> Ɛ E e b Ɛ F ==> a + b ≡ 0 (mod5).
>>>> G= {2, 7, 12, 17, 22,27} e H = {3, 8, 13, 18, 23} Para qualquer  (a,b)
>>>> com a Ɛ G e b Ɛ H ==> a + b ≡ 0 (mod5).
>>>> J= {5, 15, 20, 25} Para qualquer par (a,b) com a,b Ɛ J==> a + b ≡ 0
>>>> (mod5).
>>>>
>>>> O jogador A só ganha se restarem dois números pertencentes a J, um a G
>>>> e outro a H, um a E e outro a F.
>>>> Portanto o jogador B vence fácil.
>>>>
>>>> Basta para cada escolha  a do jogador A que inicia, o jogador B deve
>>>> escolher -a | a + (-a) ≡ 0 (mod5).
>>>>
>>>> Se A escolhe em E, B escolhe em F e vice-versa.
>>>> Se A escolhe em G, B escolhe em H e vice-versa.
>>>> Se A escolhem J, B escolhe em J.
>>>>
>>>> Como a cardinalidade de E e G é maior que a cardinalidade de F e H e a
>>>> cardinalidade de J é par, ao final sobrarão um elemento s Ɛ E e t Ɛ  F
>>>> | s + t ≡ 3 (mod5)
>>>> Saudações,
>>>> PJMS
>>>>
>>>>
>>>> Em 1 de julho de 2015 06:46,  escreveu:
>>>>
>>>>> Problema
>>>>> Dois jogadores, A e B, disputam um jogo, em que jogam alternadamente.
>>>>> O jogador A começa. Uma jogada consiste em apagar um dos números inteiros
>>>>> do conjunto {1, 2, 3,..., 27} até que reste somente dois números. Se a 
>>>>> soma
>>>>> desses dois últimos números for divisível por 5, o jogador A vence, caso
>>>>> contrário, vence o jogador B.
>>>>> Se cada jogador faz suas melhores jogadas, quem vence: A ou B? Qual é
>>>>> a estratégia para vencer?
>>>>>
>>>>> --
>>>>> Esta mensagem foi verificada pelo sistema de antivírus e
>>>>> acredita-se estar livre de perigo.
>>>>>
>>>>
>>>>
>>>>
>>>> --
>>>> Esta mensagem foi verificada pelo sistema de antivírus e
>>>> acredita-se estar livre de perigo.
>>>>
>>>
>>>
>>>
>>> --
>>> Abraços
>>>
>>> oɾnɐɹɐ ǝp oıɔıɹnɐɯ
>>>
>>>
>>
>>
>> --
>> Abraços
>>
>> oɾnɐɹɐ ǝp oıɔıɹnɐɯ
>>
>>
>
>
> --
> Abraços
>
> oɾnɐɹɐ ǝp oıɔıɹnɐɯ
>
>
> [A mensagem original inteira não está incluída.]
>
> --
> Esta mensagem foi verificada pelo sistema de antivírus e
> acredita-se estar livre de perigo.
>

-- 
Esta mensagem foi verificada pelo sistema de antiv�rus e
 acredita-se estar livre de perigo.



[obm-l] Re: [obm-l] Problema da 18ª Olimpíada de Maio

2015-06-26 Por tôpico Rogerio Ponce
Ola' Mariana,
trace por M uma perpendicular ao lado BC, e chame de "E" sua intersecao com
DB.
Chame de "F" a intersecao de DM com CE.

Por construcao, o triangulo EBC e' isosceles.
Como CD e' perpendicular 'a CA, entao CD e' bissetriz ( externa ) do angulo
entre o lado CD e o prolongamento do lado BC do triangulo EBC.

Portanto, como CD e CA sao bissetrizes externa e interna, temos que:
DB / DE = AB / EA

Aplicando Menelaus 'a reta DM e o triangulo EBC, temos que:
(DB / DE)  * (MC / BM) * (FE / CF) = 1

Substituindo o valor de DB/DE de uma equacao em outra, e observando que
MC=BM, vem:
AB / EA = CF / FE

Logo,
AB / (EA+AB) = CF / (CF+FE)

Como EA+AB = CF+FE,
AB = CF

Logo os triangulos AMB e FMC sao simetricos, e os angulos FMC e AMB sao
iguais.

Ou seja,
DMC=AMB

[]'
Rogerio Ponce


2015-06-22 17:34 GMT-03:00 Mariana Groff :

> Boa Tarde,
>
> No triângulo ABC, verificamos que  90 . Seja M o ponto médio
> de BC. A perpendicular por C ao lado AC corta a reta AB no ponto D.
> Demonstre que 
> Obrigada,
> Mariana
>
> --
> Esta mensagem foi verificada pelo sistema de antivírus e
> acredita-se estar livre de perigo.

-- 
Esta mensagem foi verificada pelo sistema de antiv�rus e
 acredita-se estar livre de perigo.



Re: [obm-l] Seis Pontos

2015-06-22 Por tôpico Rogerio Ponce
Ola' Mariana, Pedro e colegas da lista,

digamos que um segmento seja do tipo "M" ou "m" caso ele
seja,respectivamente, o maior lado ou o menor lado de algum triangulo.
Eventualmente um segmento pode ser simultaneamente dos 2 tipos, "M" e "m",
(tipo "M+m") e cuja existencia e' o que queremos demonstrar.
E vamos chamar de tipo "x" ao segmento que nao seja nem do tipo "M" e nem
do tipo "m".

Sejam os 6 pontos 0,1,2,3,4,5.
Suponhamos (por absurdo) que nao haja qualquer segmento do tipo "M+m".

Consideremos o ponto "0", por exemplo, de onde partem 5 segmentos.
Podemos fazer algumas afirmacoes:

1) Suponhamos que 2 dos segmentos, 01 e 02 por exemplo, fossem do tipo "x".
Entao, no triangulo 012, o lado 12 teria que ser o menor e tambem o maior
entre os 3 lados, o que e' absurdo.
Logo, partindo de um mesmo ponto, tem que haver menos que 2 segmentos do
tipo "x".
_

2) Suponhamos que 3 dos segmentos, 01, 02 e 03, por exemplo, fossem somente
do tipo "M".
Entao, podemos afirmar que
o triangulo 012 tem o lado 12 do tipo "m".
o triangulo 013 tem o lado 13 do tipo "m".
o triangulo 023 tem o lado 23 do tipo "m".

Acontece que o triangulo formado por 123 (lados 12,13,23) tem que ter um
lado do tipo "M", fazendo com que um desses segmentos seja do tipo "M+m", o
que seria uma contradicao ('a nossa hipotese inicial de nao haver segmento
desse tipo).

Logo, partindo de um mesmo ponto, tem que haver menos que 3 segmentos do
tipo "M".
Similarmente, partindo de um mesmo ponto, tem que haver menos que 3
segmentos do tipo "m".
_

3) Portanto, com as duas conclusoes anteriores, vemos que partindo de um
mesmo ponto tem que haver exatamente um segmento do tipo "x" , dois
segmentos do tipo "M" , e dois segmentos do tipo "m".

Assim, considerando os 5 segmentos que partem do ponto "0", seja o segmento
01 do tipo "x" , 02 e 03 do tipo "M" , 04 e 05 do tipo "m".

Entao, o lado 12 do triangulo 012 tem que ser do tipo "m" , pois 01 e' do
tipo "x" e 03 e' do tipo "M".

Da mesma forma, o lado 13 do triangulo 013 tem que ser do tipo "m" , pois
01 e' do tipo "x" e 02 e' do tipo "M".

E o lado 23 do triangulo 023 tem que ser do tipo "m" , pois 02 e 03 sao do
tipo "M".

Portanto, o triangulo 123 e' formado por segmentos do tipo "m".

Como um deles e' o maior dos tres, entao esse segmento tambem e' do tipo
"M", ou seja, e' do tipo "M+m" , o que e' uma contradicao 'a hipotese
inicial (de que nao haveria segmentos "M+m").

Logo, nossa hipotese inicial e' falsa, e portanto existe algum segmento que
e' simultaneamente o maior lado de algum triangulo e o menor lado de algum
triangulo.

[]'s
Rogerio Ponce


2015-05-08 20:25 GMT-03:00 Mariana Groff :

> Boa Noite,
> Alguém poderia ajudar-me no seguinte problema:
> Temos seis pontos de maneira que não haja três pontos colineares e que os
> comprimentos dos segmentos determinados por estes pontos sejam todos
> distintos. Consideramos todos os triângulos que têm seus vértices nesses
> pontos. Demonstre que um dos segmentos é, ao mesmo tempo, o menor lado de
> um desses triângulos e o maior lado de outro.
> Obrigada,
> Mariana
>
> --
> Esta mensagem foi verificada pelo sistema de antivírus e
> acredita-se estar livre de perigo.

-- 
Esta mensagem foi verificada pelo sistema de antiv�rus e
 acredita-se estar livre de perigo.



Re: [obm-l] Seis Pontos

2015-06-19 Por tôpico Rogerio Ponce
Ola' Pedro,
imagine um hexagono , e suas 3 diagonais maiores.

Vemos que, usando essa topologia, e' perfeitamente possivel haver 9
segmentos, e nenhum triangulo formado.

Portanto, o maximo que se pode garantir, e' que P1P2 pertence a algum
triangulo com UM lado menor que P1P2.

Sua demonstracao falha nessa passagem.

O problema proposto (que achei bem interessante) continua em aberto.

[]'s
Rogerio Ponce


2015-05-12 9:14 GMT-03:00 Pedro José :

> Bom dia!
>
> Tem que fazer ainda para os casos 2 e 6, 3 e 4 Pois os complementares são
> resolvidos praticamente da mesma forma (só trocar maior por menor e máximo
> por mínimo e vice-versa).
> Porém a afirmação que : "Então se tivermos pelo menos dois segmentos
> maiores que P1P2 e dois menores que P1P2 o problema está resolvido." está
> totalmente errada pois tem-se que garantir que formarão o triângulo. Dá um
> pouco de trabalho, no momento ocupado, mais tarde posto a solução para os
> outros casos.
>
> PJMS
>
> Em 11 de maio de 2015 18:24, Pedro José  escreveu:
>
>> Boa tarde!
>>
>> Se são seis pontos (P1, P2, P3...P6) nós temos 15 seguimentos. Numero
>> combinatório de 6 dois a dois = 6*5/2 = 15.
>>
>> Sem perda de generalidade o segmento central na sequência ordenada, ou
>> seja o oitavo da fila,  será denominado P1P2.
>>
>> Logo nós temos 8 segmentos que podem formar um triângulo com esse lado. P1P3,
>> P1P4, P1P5, P1P6, P2P3, P2P4, P2P5 E P2P6 não necessariamente com essa
>> ordem de comprimento.
>>
>> |_1_| |_2_| |_3_| |_4_| |_5_| |_6_| |_7_| |_P1P2_| |_9_| |_10_| |_11_|
>> |_12_| |_13_| |_14_| |_15_|
>>
>> Então as possibilidades serão 7 para segmentos de comprimento maior
>> do que o de P1P2 e mais 7 para segmentos com comprimentos menores do que
>> P1P2.
>> Como temos 8 segmentos..
>> Logo teremos pelo menos um segmento com comprimento menor que P1P2 e pelo
>> menos um com comprimento menor que P1P2.
>> Então se tivermos pelo menos dois segmentos maiores que P1P2 e dois
>> menores que P1P2 o problema está resolvido.
>>
>> Pois, construo um triângulo com dois segmentos menores e P1P2 ==> P1P2
>> será o maior lado desse triângulo.
>> Construo um outro com dois segmentos maiores e P1P2 ==> P1P2 será o menor
>> lado.
>>
>> então P1P2 só não será simultaneamente mínimo e máximo para algum
>> triângulo nas seguintes condições.
>>
>> (i) só há um dos segmentos destacados em amarelo com comprimento menor
>> que P1P2.
>> (ii) só há um dos segmentos destacados em amarelo com comprimento maior
>> que P1P2.
>>
>> Vamos estudar o primeiro caso, pois; são análogos:
>> Sem perda de generalidade. Chamemos o seguimento que tem
>> comprimento menor do que P1P2 de P2P6.
>>
>> Então os segmentos de comprimento inferior a P1P2 são; P3P4, P3P5, P3P6,
>> P4P5, P4P6, P5P6 E P2P6.
>>
>>
>> Então tomemos Três vértices que formem um triângulo com lados nos
>> destacados em azul por exemplo: P3, P4 e P5.
>>
>>
>> (a) Determine o lado de comprimento máximo, s.p.g., chamemos de P3P4.
>>
>>  Então P3P4 é o maior lado do triângulo de vértices:  P3, P4 e P5.
>>
>> (b) Já para o triângulo de vértices P1, P3 e P4, P3P4 é o menor lado do
>> triângulo; pois, Todos segmentos destacados em azul são menores que os
>> destacados em amarelo.
>>
>>
>>  (ii) Para esse caso é só fazer a separação escolher e seguir o mesmo
>> caminho só que ao invés de escolher o lado máximo, escolha o lado mínimo do
>> triângulo de vértices P3, P4 e P5 e 
>>
>> Esse problema fez parte 18a Olimpiada de Maio no ano de 2012.
>> Questionei o pessoal da OBM se há arquivo com solução, mas ainda não me
>> responderam. Talvez a solução deles seja mais simples.
>>
>> O probema pode ser encontrado no mesmo caminho que indicara:
>> http://www.obm.org.br/opencms/revista_eureka/
>> Procurar Eureka 37 (o arquivo em word está dano bug quando abro, só deu
>> certo o PDF), página 4.
>>
>> Saudações,
>> PJMS
>>
>>
>>
>>
>>
>>
>>
>>
>>
>>
>>
>> Em 8 de maio de 2015 20:25, Mariana Groff > > escreveu:
>>
>>> Boa Noite,
>>> Alguém poderia ajudar-me no seguinte problema:
>>> Temos seis pontos de maneira que não haja três pontos colineares e que
>>> os comprimentos dos segmentos determinados por estes pontos sejam todos
>>> distintos. Consideramos todos os triângulos que têm seus vértices nesses
>>> pontos. Demonstre que um dos segmentos é, ao mesmo tempo, o menor lado de
>>> um desses triângulos e o maior lado de outro.
>>> Obrigada,
>>> Mariana
>>>
>>> --
>>> Esta mensagem foi verificada pelo sistema de antivírus e
>>> acredita-se estar livre de perigo.
>>
>>
>>
>
> --
> Esta mensagem foi verificada pelo sistema de antivírus e
> acredita-se estar livre de perigo.

-- 
Esta mensagem foi verificada pelo sistema de antiv�rus e
 acredita-se estar livre de perigo.



[obm-l] Re: [obm-l] Combinatória

2015-05-24 Por tôpico Rogerio Ponce
A sequencia comeca com um IMPAR e a segunda e' PAR, e vao se alternando
sucessivamente...

2015-05-24 15:35 GMT-03:00 Rogerio Ponce :

> Oi Bernardo, obrigado, engoli "a soma".
> Indo de um em um, a "soma" do primeiro e' par, a proxima e' impar, etc.
> (afinal o Marcone nao queria saber quantos numeros pares existiam na
> sequencia...)
> :)
> []'s
> Rogerio Ponce
>
> 2015-05-24 12:56 GMT-03:00 Rogerio Ponce :
>
>> Ola' Marcone,
>> os numeros de 9 algarismos comecam em 1, e terminam em 9.
>> Indo de um em um, o primeiro e' par, o proximo e' impar, o seguinte e'
>> par, etc...
>> A sequencia comeca com um par e termina com um impar.
>> Portanto tem a mesma quantidade de elementos pares e impares.
>> Ou seja, 45000 elementos pares e 45000 elementos impares.
>> []'s
>> Rogerio Ponce
>>
>> 2015-05-23 21:31 GMT-03:00 marcone augusto araújo borges <
>> marconeborge...@hotmail.com>:
>>
>>> Quantos números de 9 algarismos tem a soma dos seus algarismos par?
>>>
>>> Eu achei 45000.Não tenho o gabarito.
>>> Notei que esse número é a metade do total de números de 9 algarismos
>>> Seria metade dos números com soma dos seus algarismos par e metade
>>> com soma dos algarismos ímpar.Se isso for verdade, é mera coincidência
>>> ou teria como justificar?
>>>
>>> --
>>> Esta mensagem foi verificada pelo sistema de antivírus e
>>> acredita-se estar livre de perigo.
>>>
>>
>>
>

-- 
Esta mensagem foi verificada pelo sistema de antiv�rus e
 acredita-se estar livre de perigo.



[obm-l] Re: [obm-l] Combinatória

2015-05-24 Por tôpico Rogerio Ponce
Oi Bernardo, obrigado, engoli "a soma".
Indo de um em um, a "soma" do primeiro e' par, a proxima e' impar, etc.
(afinal o Marcone nao queria saber quantos numeros pares existiam na
sequencia...)
:)
[]'s
Rogerio Ponce

2015-05-24 12:56 GMT-03:00 Rogerio Ponce :

> Ola' Marcone,
> os numeros de 9 algarismos comecam em 1, e terminam em 9.
> Indo de um em um, o primeiro e' par, o proximo e' impar, o seguinte e'
> par, etc...
> A sequencia comeca com um par e termina com um impar.
> Portanto tem a mesma quantidade de elementos pares e impares.
> Ou seja, 45000 elementos pares e 45000 elementos impares.
> []'s
> Rogerio Ponce
>
> 2015-05-23 21:31 GMT-03:00 marcone augusto araújo borges <
> marconeborge...@hotmail.com>:
>
>> Quantos números de 9 algarismos tem a soma dos seus algarismos par?
>>
>> Eu achei 45000.Não tenho o gabarito.
>> Notei que esse número é a metade do total de números de 9 algarismos
>> Seria metade dos números com soma dos seus algarismos par e metade
>> com soma dos algarismos ímpar.Se isso for verdade, é mera coincidência
>> ou teria como justificar?
>>
>> --
>> Esta mensagem foi verificada pelo sistema de antivírus e
>> acredita-se estar livre de perigo.
>>
>
>

-- 
Esta mensagem foi verificada pelo sistema de antiv�rus e
 acredita-se estar livre de perigo.



[obm-l] Re: [obm-l] Combinatória

2015-05-24 Por tôpico Rogerio Ponce
Ola' Marcone,
os numeros de 9 algarismos comecam em 1, e terminam em 9.
Indo de um em um, o primeiro e' par, o proximo e' impar, o seguinte e' par,
etc...
A sequencia comeca com um par e termina com um impar.
Portanto tem a mesma quantidade de elementos pares e impares.
Ou seja, 45000 elementos pares e 45000 elementos impares.
[]'s
Rogerio Ponce

2015-05-23 21:31 GMT-03:00 marcone augusto araújo borges <
marconeborge...@hotmail.com>:

> Quantos números de 9 algarismos tem a soma dos seus algarismos par?
>
> Eu achei 45000.Não tenho o gabarito.
> Notei que esse número é a metade do total de números de 9 algarismos
> Seria metade dos números com soma dos seus algarismos par e metade
> com soma dos algarismos ímpar.Se isso for verdade, é mera coincidência
> ou teria como justificar?
>
> --
> Esta mensagem foi verificada pelo sistema de antivírus e
> acredita-se estar livre de perigo.
>

-- 
Esta mensagem foi verificada pelo sistema de antiv�rus e
 acredita-se estar livre de perigo.



[obm-l] Re: [obm-l] Re: [obm-l] Re: [obm-l] FW: pentágono cíclico

2015-04-25 Por tôpico Rogerio Ponce
Perfeito, Ralph!
E a solucao mostra que dados os comprimentos dos lados, qualquer poligono
pode ser ciclico.
[]'s
Rogerio Ponce

2015-04-25 0:57 GMT-03:00 Ralph Teixeira :

> Hmmm... Mas *faz* sentido -- se voce dah apenas os 4 comprimentos dos
> lados, o quadrilatero nao estah fixo. Acho que eh sempre possivel deformar
> seus angulos ateh ele ficar ciclico...
>
> 2015-04-24 20:41 GMT-03:00 Sergio Lima :
>
>> Oi Ralph,
>>
>> Desculpe a minha ignorância, mas o seu método funcionaria
>> a princípio para qualquer quadrilátero, o que não faz sentido.
>>
>> De todo modo, vindo do Luís, acredito que seja um problema
>> de contrução com régua e compasso.
>>
>> Abraço,
>> Sergio
>>
>> On Friday, April 24, 2015, Ralph Teixeira  wrote:
>>
>>> Construir, tipo, com regua e compasso? Ou, num sentido mais teorico e
>>> geral?
>>>
>>> Pegue um crculo com raio 300, marque pontos ABCDEF tal que as cordas
>>> AB, BC, CD, DE e EF tenham os comprimentos pedidos. Agora diminua o raio do
>>> circulo ateh que A=F... Hmmm... alguem tem algum motivo para essa
>>> construcao NAO dar certo? (Eu queria fazer no Geogebra, mas estou sem
>>> Geogebra neste instante).
>>>
>>> Tem que fazer algum argumento do tipo "nenhum dos lados eh maior do qu
>>> eo diametro do circulo" durante minha construcao, mas acho *ACHO* que isto
>>> eh equivalente a cada lado ser menor que a soma dos outros.
>>>
>>> Abraco, Ralph
>>>
>>> 2015-04-24 11:18 GMT-03:00 Luís :
>>>
>>>> Sauda,c~oes,
>>>>
>>>> Alguém saberia responder ?
>>>>
>>>> Abraços,
>>>> Luís
>>>>
>>>> > Date: Fri, 24 Apr 2015 10:31:14 +0100
>>>> > Subject: pentágono cíclico
>>>> >
>>>> > Estimado, Luís
>>>> >
>>>> > ¿Es posible construir un pentágono inscrito en una circunferencia
>>>> cuyos
>>>> > lados a1,a2,a3,a4,a5 tiene longitudes: 13, 13, 5 + 12*Sqrt[3],
>>>> > 20*Sqrt[3], -5 + 12*Sqrt[3]?
>>>> >
>>>> > Un saludo
>>>> > Angel
>>>>
>>>> --
>>>> Esta mensagem foi verificada pelo sistema de antivírus e
>>>> acredita-se estar livre de perigo.
>>>>
>>>
>>>
>>> --
>>> Esta mensagem foi verificada pelo sistema de antivírus e
>>> acredita-se estar livre de perigo.
>>
>>
>> --
>> Esta mensagem foi verificada pelo sistema de antivírus e
>> acredita-se estar livre de perigo.
>
>
>
> --
> Esta mensagem foi verificada pelo sistema de antivírus e
> acredita-se estar livre de perigo.

-- 
Esta mensagem foi verificada pelo sistema de antiv�rus e
 acredita-se estar livre de perigo.



Re: [obm-l] Problema das caixas

2015-04-22 Por tôpico Rogerio Ponce
Ola' Mariana,
como as bolinhas "andam" somente para a direita, a ultima caixa 'a direita
e' o destino de todas as bolinhas.
Repare que temos que esvaziar primeiramente a caixa mais 'a esquerda, em
seguida a proxima do seu lado direito, e assim sucessivamente, caso
contrario o fluxo seria interrompido.

Outra observacao e' que cada bolinha deve seguir da caixa mais 'a esquerda
(caixa 1) ate' a caixa mais 'a direita (caixa 27), para somente entao
movimentarmos uma outra bolinha da caixa mais 'a esquerda, repetindo o
processo ate' esvaziarmos a caixa 1, e entao a caixa 2, etc...

Assim, a caixa 1 deve ter 12 bolinhas, pois e' o minimo.
A caixa 2 , 13 bolinhas (e quando ele recebe 1 bolinha da caixa 1, ela
passa a ter 14 bolinhas).
A caixa 3 , 15 bolinhas (para que possa receber a decima-quarta bolinha da
caixa 2).
A caixa 4, 17 bolinhas (pelas razoes anteriores).
etc..
A caixa 27, 13+25*2 = 63 bolinhas.

Portanto, da caixa 2 ate' a caixa 27, temos uma progressao aritmetica com
razao 2, cuja soma da'
(13 + 63) * 26 / 2 = 988 bolinhas.
Acrescentando as 12 bolinhas da caixa 1, obtemos o total de 1000 bolinhas.

[]'s
Rogerio Ponce






2015-04-21 15:21 GMT-03:00 Mariana Groff :

> Boa tarde,
> Alguém poderia me ajudar no problema a seguir?
>
> Temos 27 caixas em fila; cada uma delas contém pelo menos 12 bolinhas. A
> operação permitida é transferir uma bolinha de uma caixa para sua vizinha
> da direita, se essa vizinha da direita tem mais bolinhas. Dizemos que uma
> distribuição inicial das bolinhas é *feliz* se é possível, mediante uma
> sucessão de operações permitidas, fazer com que todas as bolinhas fiquem
> numa mesma caixa. Determine o menor número total de bolinhas de uma
> distribuição inicial feliz.
>
> Obrigada,
> Mariana
>
> --
> Esta mensagem foi verificada pelo sistema de antivírus e
> acredita-se estar livre de perigo.

-- 
Esta mensagem foi verificada pelo sistema de antiv�rus e
 acredita-se estar livre de perigo.



Re: [obm-l] Tabuleiro 3x3 com 4 cores

2015-03-31 Por tôpico Rogerio Ponce
Ola' Pacini,
o loop que eliminava a igualdade por rotacao, tambem ja' contava cada
combinacao permitida.
Neste caso, o total e' de 9612 pinturas.

[]'s
Rogerio Ponce


2015-03-30 14:55 GMT-03:00 Pacini Bores :

> Oi Ponce, na verdade é para considerar todas as possibilidades, ou seja,
> não é um tabuleiro apesar do enunciado ter sido inicialmente com o
> tabuleiro, ok ? Desculpe, caso tenha dado algum transtorno.
>
> abraços
>
> Pacini
>
> Em 30 de março de 2015 13:38, Rogerio Ponce  escreveu:
>
>> Ooopa, quero dizer, 2472.
>>
>> []'s
>> Rogerio Ponce
>>
>> 2015-03-30 11:59 GMT-03:00 Rogerio Ponce :
>>
>> Ola' pessoal,
>>> eu acho que a questao e' um pouco mais complicada, pois e' razoavel que
>>> pinturas obtidas por rotacao do tabuleiro sejam consideradas a mesma
>>> pintura.
>>>
>>> Utilizando forca bruta, encontrei apenas 2724 modos diferentes de se
>>> pintar o tabuleiro.
>>>
>>> []'s
>>> Rogerio Ponce
>>>
>>> 2015-03-30 11:16 GMT-03:00 Pedro José :
>>>
>>>> Bom dia!
>>>>
>>>> Havia feito para exatamente quatro cores. Mas, é fácil adaptar para até
>>>> quatro cores, há até menos restrições.
>>>> Resolvi por grafo, fazendo opções.
>>>> Preenchimento primeiramente de a1,1, depois o par a2,1 e a1,2, depois o
>>>> par a2,2 e a1,3 em seguida a3,2 e a2,3 e por último a3,1 e a3,3.
>>>> Abri o grafo sempre iguais ou diferentes.
>>>> Certamente, não está otimizado.
>>>> Encontrei: 8640 possibilidades com exatamente 4 cores.
>>>>
>>>> Vou refazer para até quatro cores e vos envio o grafo, se possível
>>>> ainda hoje ao final da tarde (ocupado), vai ser escaneado, pois fiz na mão.
>>>>
>>>> Saudações,
>>>> PJMS
>>>>
>>>>
>>>>
>>>>
>>>>
>>>> Em 30 de março de 2015 10:49, Carlos Victor 
>>>> escreveu:
>>>>
>>>>> Acredito que  ideia do Bob Roy é o mais rápida para obter a solução.
>>>>>
>>>>> Carlos  Victor
>>>>>
>>>>> Em 30 de março de 2015 10:39, Pacini Bores 
>>>>> escreveu:
>>>>>
>>>>>> Sim Pedro, esta é uma solução; ou seja, há possibilidade de se usar
>>>>>> até quatro cores.
>>>>>>
>>>>>> Pacini
>>>>>>
>>>>>> Em 30 de março de 2015 10:23, Pedro José 
>>>>>> escreveu:
>>>>>>
>>>>>>> Bom dia!
>>>>>>>
>>>>>>> Uma dúvida há necessidade de se usar as quatro cores ou há a
>>>>>>> possibilidade de se usar até quatro cores?
>>>>>>>
>>>>>>> Por exemplo,
>>>>>>>
>>>>>>> 0 1 0
>>>>>>> 1 0 1
>>>>>>> 0 1 0
>>>>>>>
>>>>>>> onde 0 e 1 representam duas cores distintas, seria uma solução?
>>>>>>>
>>>>>>> Saudações,
>>>>>>>
>>>>>>> PJMS
>>>>>>>
>>>>>>>
>>>>>>>
>>>>>>>
>>>>>>>
>>>>>>> Em 29 de março de 2015 11:26, Bob Roy  escreveu:
>>>>>>>
>>>>>>>> Olá, O melhor para este problema é utlizar  o que o grande mestre
>>>>>>>> Morgado falava : devemos inicialmente eliminar as dificuldades.
>>>>>>>>
>>>>>>>> Considerando uma matriz 3x3 , temos que os quadradinhos a12, a21,
>>>>>>>> a23 e a32 não poderão ter todas as cores diferentes.
>>>>>>>>
>>>>>>>> Comece fazendo a análise com  duas cores iguais, três cores iguais
>>>>>>>> e depois quatro cores iguais para essas posições.
>>>>>>>>
>>>>>>>> A análise ficará menos trabalhosa .
>>>>>>>>
>>>>>>>> Farei as contas e depois eu posto o resultado.
>>>>>>>>
>>>>>>>> Roy
>>>>>>>>
>>>>>>>>
>>>>>>>> Em 28 de março de 2015 10:22, Carlos Victor >>>>>>> > escreveu:
>>>>>>>>
>>>>>>>>>

Re: [obm-l] Tabuleiro 3x3 com 4 cores

2015-03-30 Por tôpico Rogerio Ponce
Ooopa, quero dizer, 2472.

[]'s
Rogerio Ponce

2015-03-30 11:59 GMT-03:00 Rogerio Ponce :

> Ola' pessoal,
> eu acho que a questao e' um pouco mais complicada, pois e' razoavel que
> pinturas obtidas por rotacao do tabuleiro sejam consideradas a mesma
> pintura.
>
> Utilizando forca bruta, encontrei apenas 2724 modos diferentes de se
> pintar o tabuleiro.
>
> []'s
> Rogerio Ponce
>
> 2015-03-30 11:16 GMT-03:00 Pedro José :
>
>> Bom dia!
>>
>> Havia feito para exatamente quatro cores. Mas, é fácil adaptar para até
>> quatro cores, há até menos restrições.
>> Resolvi por grafo, fazendo opções.
>> Preenchimento primeiramente de a1,1, depois o par a2,1 e a1,2, depois o
>> par a2,2 e a1,3 em seguida a3,2 e a2,3 e por último a3,1 e a3,3.
>> Abri o grafo sempre iguais ou diferentes.
>> Certamente, não está otimizado.
>> Encontrei: 8640 possibilidades com exatamente 4 cores.
>>
>> Vou refazer para até quatro cores e vos envio o grafo, se possível ainda
>> hoje ao final da tarde (ocupado), vai ser escaneado, pois fiz na mão.
>>
>> Saudações,
>> PJMS
>>
>>
>>
>>
>>
>> Em 30 de março de 2015 10:49, Carlos Victor 
>> escreveu:
>>
>>> Acredito que  ideia do Bob Roy é o mais rápida para obter a solução.
>>>
>>> Carlos  Victor
>>>
>>> Em 30 de março de 2015 10:39, Pacini Bores 
>>> escreveu:
>>>
>>>> Sim Pedro, esta é uma solução; ou seja, há possibilidade de se usar até
>>>> quatro cores.
>>>>
>>>> Pacini
>>>>
>>>> Em 30 de março de 2015 10:23, Pedro José 
>>>> escreveu:
>>>>
>>>>> Bom dia!
>>>>>
>>>>> Uma dúvida há necessidade de se usar as quatro cores ou há a
>>>>> possibilidade de se usar até quatro cores?
>>>>>
>>>>> Por exemplo,
>>>>>
>>>>> 0 1 0
>>>>> 1 0 1
>>>>> 0 1 0
>>>>>
>>>>> onde 0 e 1 representam duas cores distintas, seria uma solução?
>>>>>
>>>>> Saudações,
>>>>>
>>>>> PJMS
>>>>>
>>>>>
>>>>>
>>>>>
>>>>>
>>>>> Em 29 de março de 2015 11:26, Bob Roy  escreveu:
>>>>>
>>>>>> Olá, O melhor para este problema é utlizar  o que o grande mestre
>>>>>> Morgado falava : devemos inicialmente eliminar as dificuldades.
>>>>>>
>>>>>> Considerando uma matriz 3x3 , temos que os quadradinhos a12, a21, a23
>>>>>> e a32 não poderão ter todas as cores diferentes.
>>>>>>
>>>>>> Comece fazendo a análise com  duas cores iguais, três cores iguais e
>>>>>> depois quatro cores iguais para essas posições.
>>>>>>
>>>>>> A análise ficará menos trabalhosa .
>>>>>>
>>>>>> Farei as contas e depois eu posto o resultado.
>>>>>>
>>>>>> Roy
>>>>>>
>>>>>>
>>>>>> Em 28 de março de 2015 10:22, Carlos Victor 
>>>>>> escreveu:
>>>>>>
>>>>>>> Comece pelo centro e pelas laterais, isto deve diminuir as
>>>>>>> dificuldades. Abrirão vários casos para serem analisados.
>>>>>>>
>>>>>>> E se  não me engano, esta questão tem como origem  não considerando
>>>>>>> os quadrados pelos vértices com as mesmas cores. Neste  caso a análise 
>>>>>>> fica
>>>>>>> mais silmplificada.
>>>>>>>
>>>>>>> Abraços
>>>>>>>
>>>>>>> Carlos Victor
>>>>>>>
>>>>>>> Em 28 de março de 2015 09:38, Pacini Bores 
>>>>>>> escreveu:
>>>>>>>
>>>>>>>> Olá pessoal,  como pensar nesta ?
>>>>>>>>
>>>>>>>> De quantas maneiras podemos pintar um tabuleiro 3x3 com 4 cores de
>>>>>>>> tal forma que não tenhamos cores adjacentes ?
>>>>>>>>
>>>>>>>> Nota : em diagonal não é considerado adjacente.
>>>>>>>>
>>>>>>>> Agradeço desde já
>>>>>>>>
>>>>>>>> Pacini.
>>>>>>>>
>>>>>>>> --
>>>>>>>> Esta mensagem foi verificada pelo sistema de antivírus e
>>>>>>>> acredita-se estar livre de perigo.
>>>>>>>
>>>>>>>
>>>>>>>
>>>>>>> --
>>>>>>> Esta mensagem foi verificada pelo sistema de antivírus e
>>>>>>> acredita-se estar livre de perigo.
>>>>>>>
>>>>>>
>>>>>>
>>>>>> --
>>>>>> Esta mensagem foi verificada pelo sistema de antivírus e
>>>>>> acredita-se estar livre de perigo.
>>>>>>
>>>>>
>>>>>
>>>>> --
>>>>> Esta mensagem foi verificada pelo sistema de antivírus e
>>>>> acredita-se estar livre de perigo.
>>>>>
>>>>
>>>>
>>>> --
>>>> Esta mensagem foi verificada pelo sistema de antivírus e
>>>> acredita-se estar livre de perigo.
>>>>
>>>
>>>
>>> --
>>> Esta mensagem foi verificada pelo sistema de antivírus e
>>> acredita-se estar livre de perigo.
>>>
>>
>>
>> --
>> Esta mensagem foi verificada pelo sistema de antivírus e
>> acredita-se estar livre de perigo.
>
>
>

-- 
Esta mensagem foi verificada pelo sistema de antiv�rus e
 acredita-se estar livre de perigo.



Re: [obm-l] Tabuleiro 3x3 com 4 cores

2015-03-30 Por tôpico Rogerio Ponce
Ola' pessoal,
eu acho que a questao e' um pouco mais complicada, pois e' razoavel que
pinturas obtidas por rotacao do tabuleiro sejam consideradas a mesma
pintura.

Utilizando forca bruta, encontrei apenas 2724 modos diferentes de se pintar
o tabuleiro.

[]'s
Rogerio Ponce

2015-03-30 11:16 GMT-03:00 Pedro José :

> Bom dia!
>
> Havia feito para exatamente quatro cores. Mas, é fácil adaptar para até
> quatro cores, há até menos restrições.
> Resolvi por grafo, fazendo opções.
> Preenchimento primeiramente de a1,1, depois o par a2,1 e a1,2, depois o
> par a2,2 e a1,3 em seguida a3,2 e a2,3 e por último a3,1 e a3,3.
> Abri o grafo sempre iguais ou diferentes.
> Certamente, não está otimizado.
> Encontrei: 8640 possibilidades com exatamente 4 cores.
>
> Vou refazer para até quatro cores e vos envio o grafo, se possível ainda
> hoje ao final da tarde (ocupado), vai ser escaneado, pois fiz na mão.
>
> Saudações,
> PJMS
>
>
>
>
>
> Em 30 de março de 2015 10:49, Carlos Victor 
> escreveu:
>
>> Acredito que  ideia do Bob Roy é o mais rápida para obter a solução.
>>
>> Carlos  Victor
>>
>> Em 30 de março de 2015 10:39, Pacini Bores 
>> escreveu:
>>
>>> Sim Pedro, esta é uma solução; ou seja, há possibilidade de se usar até
>>> quatro cores.
>>>
>>> Pacini
>>>
>>> Em 30 de março de 2015 10:23, Pedro José  escreveu:
>>>
>>>> Bom dia!
>>>>
>>>> Uma dúvida há necessidade de se usar as quatro cores ou há a
>>>> possibilidade de se usar até quatro cores?
>>>>
>>>> Por exemplo,
>>>>
>>>> 0 1 0
>>>> 1 0 1
>>>> 0 1 0
>>>>
>>>> onde 0 e 1 representam duas cores distintas, seria uma solução?
>>>>
>>>> Saudações,
>>>>
>>>> PJMS
>>>>
>>>>
>>>>
>>>>
>>>>
>>>> Em 29 de março de 2015 11:26, Bob Roy  escreveu:
>>>>
>>>>> Olá, O melhor para este problema é utlizar  o que o grande mestre
>>>>> Morgado falava : devemos inicialmente eliminar as dificuldades.
>>>>>
>>>>> Considerando uma matriz 3x3 , temos que os quadradinhos a12, a21, a23
>>>>> e a32 não poderão ter todas as cores diferentes.
>>>>>
>>>>> Comece fazendo a análise com  duas cores iguais, três cores iguais e
>>>>> depois quatro cores iguais para essas posições.
>>>>>
>>>>> A análise ficará menos trabalhosa .
>>>>>
>>>>> Farei as contas e depois eu posto o resultado.
>>>>>
>>>>> Roy
>>>>>
>>>>>
>>>>> Em 28 de março de 2015 10:22, Carlos Victor 
>>>>> escreveu:
>>>>>
>>>>>> Comece pelo centro e pelas laterais, isto deve diminuir as
>>>>>> dificuldades. Abrirão vários casos para serem analisados.
>>>>>>
>>>>>> E se  não me engano, esta questão tem como origem  não considerando
>>>>>> os quadrados pelos vértices com as mesmas cores. Neste  caso a análise 
>>>>>> fica
>>>>>> mais silmplificada.
>>>>>>
>>>>>> Abraços
>>>>>>
>>>>>> Carlos Victor
>>>>>>
>>>>>> Em 28 de março de 2015 09:38, Pacini Bores 
>>>>>> escreveu:
>>>>>>
>>>>>>> Olá pessoal,  como pensar nesta ?
>>>>>>>
>>>>>>> De quantas maneiras podemos pintar um tabuleiro 3x3 com 4 cores de
>>>>>>> tal forma que não tenhamos cores adjacentes ?
>>>>>>>
>>>>>>> Nota : em diagonal não é considerado adjacente.
>>>>>>>
>>>>>>> Agradeço desde já
>>>>>>>
>>>>>>> Pacini.
>>>>>>>
>>>>>>> --
>>>>>>> Esta mensagem foi verificada pelo sistema de antivírus e
>>>>>>> acredita-se estar livre de perigo.
>>>>>>
>>>>>>
>>>>>>
>>>>>> --
>>>>>> Esta mensagem foi verificada pelo sistema de antivírus e
>>>>>> acredita-se estar livre de perigo.
>>>>>>
>>>>>
>>>>>
>>>>> --
>>>>> Esta mensagem foi verificada pelo sistema de antivírus e
>>>>> acredita-se estar livre de perigo.
>>>>>
>>>>
>>>>
>>>> --
>>>> Esta mensagem foi verificada pelo sistema de antivírus e
>>>> acredita-se estar livre de perigo.
>>>>
>>>
>>>
>>> --
>>> Esta mensagem foi verificada pelo sistema de antivírus e
>>> acredita-se estar livre de perigo.
>>>
>>
>>
>> --
>> Esta mensagem foi verificada pelo sistema de antivírus e
>> acredita-se estar livre de perigo.
>>
>
>
> --
> Esta mensagem foi verificada pelo sistema de antivírus e
> acredita-se estar livre de perigo.

-- 
Esta mensagem foi verificada pelo sistema de antiv�rus e
 acredita-se estar livre de perigo.



[obm-l] Re: [obm-l] Re: [obm-l] Re: [obm-l] Número de cinco algarismos

2015-03-18 Por tôpico Rogerio Ponce
Oi Douglas e Roger,
eu resolvi apenas a primeira parte da questao, que seria descobrir
"quantos numeros divisiveis por 3, de 5 algarismos, nao possuem o algarismo
6 em qualquer casa".

Agora bastar vermos quantos numeros divisiveis por 3, de 5 algarismos
existem, e entao fazermos a subtracao.

Considerando a casa menos significativa como a primeira, temos 10 opcoes
para a 1a. , 10 opcoes para a 2a. , 10 opcoes para a 3a. , e 10 opcoes para
a 4a.

Usando o mesmo raciocinio da minha mensagem anterior, vemos que para a 5a.
casa (a mais significativa), independentemente do modulo da soma das 4
primeiras casas, sempre havera' 3 opcoes: se modulo=0, opcoes=[3,6,9] ; se
modulo=1, opcoes=[2,5,8] ; se modulo=2, opcoes=[1,4,7] .

Assim, o total de numeros divisiveis por 3 vale 10*10*10*10*3=3 , e a
quantidade que estamos procurando vale 3-17496=12504.
Portanto, a resposta correta e' letra "e".

[]'s
Rogerio Ponce



2015-03-18 18:16 GMT-03:00 Douglas Oliveira de Lima <
profdouglaso.del...@gmail.com>:

> Não entendi muito bem a pergunta,  e porque não pode entrar 6 no início? O
> 6 aparece somente uma vez?
> Em 18/03/2015 17:33, "Rogerio Ponce"  escreveu:
>
>> Ola' Roger,
>> para que o numero seja divisivel por 3, a soma (em modulo 3) de todos os
>> seus algarismos tem que dar zero.
>> Na casa mais significativa nao podemos ter nem 0 e nem 6, de forma que
>> temos 8 escolhas.
>> Para as proximas 3 casas, temos 9 escolhas em cada uma.
>> Caso a soma (em modulo 3) das 4 primeiras casas seja 0 , temos 3 opcoes
>> para a ultima casa: 0,3,9
>> Caso a soma seja 1, tambem temos 3 opcoes para a ultima casa: 2,5,8
>> E caso a soma seja 2, novamente temos 3 opcoes para a ultima casa: 1,4,7
>> Assim, independentemente da escolha das 4 primeiras casas, existem sempre
>> 3 escolhas para a casa menos significativa.
>> Portanto, ha' 8*9*9*9*3 = 17496 formas de se construir o numero, e a
>> resposta e' a letra "b".
>>
>> []'s
>> Rogerio Ponce
>>
>> 2015-03-18 8:19 GMT-03:00 Roger :
>>
>>> Por gentileza, a questão abaixo caso alguém consiga a solução da mesma.
>>>
>>> 1) Quantos números de cinco algarismos são divisíveis por 3 e possuem 6
>>> como um dos seus algarismos? a) 2 b) 17496 c) 12503 d) 18456 e) 12504
>>>
>>> --
>>> Esta mensagem foi verificada pelo sistema de antivírus e
>>> acredita-se estar livre de perigo.
>>
>>
>>
>> --
>> Esta mensagem foi verificada pelo sistema de antivírus e
>> acredita-se estar livre de perigo.
>
>
> --
> Esta mensagem foi verificada pelo sistema de antivírus e
> acredita-se estar livre de perigo.

-- 
Esta mensagem foi verificada pelo sistema de antiv�rus e
 acredita-se estar livre de perigo.



[obm-l] Re: [obm-l] Número de cinco algarismos

2015-03-18 Por tôpico Rogerio Ponce
Ola' Roger,
para que o numero seja divisivel por 3, a soma (em modulo 3) de todos os
seus algarismos tem que dar zero.
Na casa mais significativa nao podemos ter nem 0 e nem 6, de forma que
temos 8 escolhas.
Para as proximas 3 casas, temos 9 escolhas em cada uma.
Caso a soma (em modulo 3) das 4 primeiras casas seja 0 , temos 3 opcoes
para a ultima casa: 0,3,9
Caso a soma seja 1, tambem temos 3 opcoes para a ultima casa: 2,5,8
E caso a soma seja 2, novamente temos 3 opcoes para a ultima casa: 1,4,7
Assim, independentemente da escolha das 4 primeiras casas, existem sempre 3
escolhas para a casa menos significativa.
Portanto, ha' 8*9*9*9*3 = 17496 formas de se construir o numero, e a
resposta e' a letra "b".

[]'s
Rogerio Ponce

2015-03-18 8:19 GMT-03:00 Roger :

> Por gentileza, a questão abaixo caso alguém consiga a solução da mesma.
>
> 1) Quantos números de cinco algarismos são divisíveis por 3 e possuem 6
> como um dos seus algarismos? a) 2 b) 17496 c) 12503 d) 18456 e) 12504
>
> --
> Esta mensagem foi verificada pelo sistema de antivírus e
> acredita-se estar livre de perigo.

-- 
Esta mensagem foi verificada pelo sistema de antiv�rus e
 acredita-se estar livre de perigo.



Re: [obm-l] Geometria plana

2015-03-03 Por tôpico Rogerio Ponce
E' verdade, Douglas, engraxei a meia...
:)

[]'s
Rogerio Ponce

2015-03-02 20:42 GMT-03:00 Douglas Oliveira de Lima <
profdouglaso.del...@gmail.com>:

> Está correto Ponce de uma olhada com calma.
> Forte abraço.
> Em 02/03/2015 19:56, "Rogerio Ponce"  escreveu:
>
>> Ola' Douglas,
>> eu acho que tem algum engano no enunciado.
>> Se D pertence ao lado BC, me parece impossivel que os angulos BAC e BED
>> sejam iguais entre si.
>> []'s
>> Rogerio Ponce
>>
>> 2015-03-02 9:23 GMT-03:00 Douglas Oliveira de Lima <
>> profdouglaso.del...@gmail.com>:
>>
>>> Olá,  bom dia quero compartilhar uma boa questão de geometria com os
>>> senhores,
>>> Q1) Num triângulo isósceles ABC com AB=AC,  toma-se um ponto D no lado
>>> BC de forma que BD=2CD e um ponto E em AD tal que os ângulos BAC e BED
>>> sejam iguais a 80 graus,   encontrar o valor do ângulo DEC.
>>>
>>> Douglas Oliveira.
>>>
>>> --
>>> Esta mensagem foi verificada pelo sistema de antivírus e
>>> acredita-se estar livre de perigo.
>>
>>
>>
>> --
>> Esta mensagem foi verificada pelo sistema de antivírus e
>> acredita-se estar livre de perigo.
>
>
> --
> Esta mensagem foi verificada pelo sistema de antivírus e
> acredita-se estar livre de perigo.

-- 
Esta mensagem foi verificada pelo sistema de antiv�rus e
 acredita-se estar livre de perigo.



Re: [obm-l] Geometria plana

2015-03-02 Por tôpico Rogerio Ponce
Ola' Douglas,
eu acho que tem algum engano no enunciado.
Se D pertence ao lado BC, me parece impossivel que os angulos BAC e BED
sejam iguais entre si.
[]'s
Rogerio Ponce

2015-03-02 9:23 GMT-03:00 Douglas Oliveira de Lima <
profdouglaso.del...@gmail.com>:

> Olá,  bom dia quero compartilhar uma boa questão de geometria com os
> senhores,
> Q1) Num triângulo isósceles ABC com AB=AC,  toma-se um ponto D no lado BC
> de forma que BD=2CD e um ponto E em AD tal que os ângulos BAC e BED sejam
> iguais a 80 graus,   encontrar o valor do ângulo DEC.
>
> Douglas Oliveira.
>
> --
> Esta mensagem foi verificada pelo sistema de antivírus e
> acredita-se estar livre de perigo.

-- 
Esta mensagem foi verificada pelo sistema de antiv�rus e
 acredita-se estar livre de perigo.



[obm-l] Re: [obm-l] Centro da circunferência

2015-01-07 Por tôpico Rogerio Ponce
Ola' pessoal,

Chamando as 3 cordas de r,s e t, vamos inicialmente analisar r e s:

Sejam A,C as intersecoes de r com a circunferencia, e B,D as intersecoes de
S com a circunferencia, tal que percorrendo a circunferencia num mesmo
sentido encontremos A, B, C, D.
Consideremos os arcos a=AB b=BC c=CD.
Como as cordas tem o mesmo comprimento, elas determinam arcos iguais, de
modo que
a+b=b+c ,
de modo que
a=c
Portanto, o angulo  BDA e' igual ao angulo CAD , pois ambos estao sobre a
circunferencia, e compreendem arcos iguais.
Assim, o triangulo PAD e' isosceles com base AD, de modo que seu vertice P
esta' sobre a mediatriz da corda AD.

Analogamente, observando as cordas r e t, concluimos que P tambem se
encontra sobre a mediatriz de uma outra corda do mesmo circulo.
Logo P e' o centro do circulo.

[]'s
Rogerio Ponce


2015-01-06 12:47 GMT-02:00 Carlos Gomes :

> Olá amigos,
>
> Algum de você pode me ajudar com essa questão:
>
> Seja P um ponto no interior de um círculo tal que existem três cordas que
> passam
> por P e tem o mesmo comprimento. Prove que P é o centro do círculo.
>
> Grato, Cgomes.
>
> --
> Esta mensagem foi verificada pelo sistema de antivírus e
> acredita-se estar livre de perigo.

-- 
Esta mensagem foi verificada pelo sistema de antiv�rus e
 acredita-se estar livre de perigo.



Re: [obm-l] problema real - off topic

2014-11-08 Por tôpico Rogerio Ponce
Ola' Hermann,
escolha uma das chapas de 120cm de largura.

Se for a de 200cm de comprimento, a divisao do comprimento por 5 (e da
largura por 2) gera retangulos de 40cmx60cm.
Portanto voce obtera' 10 pedacos do tamanho desejado.

Se for a de 300cm, a divisao do comprimento por 5 (e da largura por 3)
tambem gera retangulos de 60cmx40cm.
Neste caso, voce obtera' 15 pedacos do tamanho desejado.

O rendimento e' o mesmo, e a decisao devera' ser funcao do total de pedacos
de que voce precisa.

Repare que, ao considerar o total, talvez mesmo uma chapa com menor
rendimento seja mais apropriada.
Exemplo: se voce precisa de apenas dois pedacos de 38x56, use a chapa de
100x200.

[]'s
Rogerio Ponce

2014-11-07 18:54 GMT-02:00 Hermann :

>  Meus amigos estou com um problema real de "otimização"
>
> Preciso cortar chapas de
>  38cm x 56cm
>
> e gostaria de saber qual dos tamanhos de chapa abaixo seria o melhor (ou
> seja, menor perda)
> 200cm x 100cm
> 200cm x 120cm
> 300cm x 100cm
> 300cm x 120cm
>
> Se alguém puder me ajudar agradeço muito
> Abraços
> Hermann
>
> --
> Esta mensagem foi verificada pelo sistema de antivírus e
> acredita-se estar livre de perigo.
>

-- 
Esta mensagem foi verificada pelo sistema de antiv�rus e
 acredita-se estar livre de perigo.



Re: [obm-l] Problema de encaixotamento de esferas:

2014-10-14 Por tôpico Rogerio Ponce
Ola' Fabio,
as esferas devem ficar em uma das diagonais principais da caixa.
Assim, elas sao tangentes em um ponto sobre essa diagonal, de modo que seus
centros distam
7+8=15 cm entre si.
Alem disso, o centro de cada esfera fica a uma certa distancia do vertice
mais proximo.
Essas distancias sao sqrt( r^2 + r^2 + r^2 ) = r*sqrt(3) , ou seja,
7*sqrt(3) cm  e  8*sqrt(3) cm
Logo, a distancia entre os vertices opostos da caixa mede
15*(1+sqrt(3)) cm
Portanto, a aresta mede
15*(1+sqrt(3)) /sqrt(3)  cm , ou seja,
15+5*sqrt(3) cm

[]'s
Rogerio Ponce





2014-10-12 9:50 GMT-03:00 FaBio Honorato :

> Bom dia pessoal, gostaria de compartilhar com vocês a seguinte questão:
> Para que uma caixa cúbica, com tampa, possa guardar juntas duas esferas de
> raios 7 cm e 8 cm, suas arestas devem medir, em cm, pelo menos:
>
> Abraços
>
> --
> Esta mensagem foi verificada pelo sistema de antivírus e
> acredita-se estar livre de perigo.
>

-- 
Esta mensagem foi verificada pelo sistema de antiv�rus e
 acredita-se estar livre de perigo.



Re: [obm-l] Inteiros

2014-09-28 Por tôpico Rogerio Ponce
Ola' Marcone,
x^2 + x + y^2 + y + z^2 + z = 1
x^2 + x + 1/4 + y^2 + y + 1/4 + z^2 + z + 1/4 = 7/4
(2x+1)^2 + (2y+1)^2 + (2z+1)^2 = 7
Como 7 nao e' soma de 3 quadrados...
[]'s
Rogerio Ponce

2014-09-28 11:07 GMT-03:00 marcone augusto araújo borges <
marconeborge...@hotmail.com>:

> Olá Douglas.Como ficou simples!Eu tinha pensado em uma esfera de raio
> raiz(7/4)
> e centro (1/2,1/2,1/2).
> Pelo intervalo de variação de x,y e z,dá pra concluir.
> Mas daquela sugestão é que não saquei o porquê.
> Obrigado!
>
> --
> Esta mensagem foi verificada pelo sistema de antivírus e
> acredita-se estar livre de perigo.
>

-- 
Esta mensagem foi verificada pelo sistema de antiv�rus e
 acredita-se estar livre de perigo.



Re: [obm-l] uma colinearidade

2014-09-20 Por tôpico Rogerio Ponce
Ola' Luis,
por construcao, os arcos (do circulo phi1) PV e QV sao iguais.
Assim, como U esta' sobre o circulo phi1, os angulos PUV e QUV sao iguais.
Ou seja, a reta "d" e' bissetriz do angulo formado pelas retas "r" e "s".
Portanto, a reflexao (em relacao a "d") de um ponto em "r" tem que estar em
"s", e vice-versa.
Como "d" passa pelo centro de phi2, M=Q' e N=P'.
[]'s
Rogerio Ponce

2014-09-19 11:07 GMT-03:00 Luís :

> Sauda,c~oes,
>
> Bom dia.
>
> Como provar que M=Q' e N=P' ? Continue a ler.
>
> Desenhe um circulo phi_1 e uma secante "d" com
> interseções U e V. Então UV é uma corda de phi_1.
>
> Desenhe um circulo phi_2 de centro V e raio "b"
> de modo que phi_1 e phi_2 se intersectam em P e Q.
>
> Trace as retas r=(P,U) e s=(Q,U)  e sejam M e N as
> interseções de "r" e "s" com phi_2.  M=r \cap phi_2 e
> N=s \cap phi_2.
>
> Sejam P' e Q' as reflexões de P e Q na secante "d".
> Como d é um diâmetro de phi_2, P' e Q' estão em phi_2.
>
> Mas fazendo esta figura com o Geogebra percebi que
> M=Q' e N=P' e não preciso construir os pontos P' e Q'.
>
> Assim, a construção de ABC dados A,b,d_c, onde d_c é
> bissetriz interna de C fica um pouco mais leve.
>
> Aqui phi_1 seria o arco capaz de A sobre o segmento (corda)
> D_cC=d_c e P e Q seriam os dois vértices A1 e A2 do triângulo.
>
> Depois disto tudo, minha pergunta: como provar que M=Q' e N=P' ?
>
> Abs,
> Luís
>
>
> --
> Esta mensagem foi verificada pelo sistema de antivírus e
> acredita-se estar livre de perigo.
>

-- 
Esta mensagem foi verificada pelo sistema de antiv�rus e
 acredita-se estar livre de perigo.



[obm-l] Parabens Nehab!

2014-07-02 Por tôpico Rogerio Ponce
Ola' pessoal, perdoem-me pelo off-topic, mas nao posso deixar de comentar:

Acabei de saber que nosso amigo Nehab, aqui da lista OBM,  foi eleito o
melhor professor de matematica do Brasil.

Foi um concurso promovido pela Vivo, aberto a todos os professores de
matematica, com video produzido por cada participante, e votacao aberta ao
publico no site http://www.vivoeducacao.com.br/concurso .

Como grande educador que e', a distincao foi mais que merecida.

Abracos a todos,
Rogerio Ponce

-- 
Esta mensagem foi verificada pelo sistema de antiv�rus e
 acredita-se estar livre de perigo.



Re: [obm-l] Proposta de Metodo para Resolver Certos Problemas de Geometria

2014-07-02 Por tôpico Rogerio Ponce
Ola' Felipe,
em relacao ao problema do pentagono que voce descreveu, talvez o enunciado
do problema estivesse incompleto, e o artificio de se levar a construcao a
uma situacao limite nao pudesse ser usado.
Ou seja, teriamos que, primeiramente, provar que o angulo CEM nao depende
do comprimento de AB.

Claro que para responder a uma questao de multipla escolha, vale o metodo -
tambem gosto dessas trapacas! -, mas se fosse uma questao discursiva bem
elaborada, certamente o  enunciado pediria para provar que o angulo seria
constante, independentemente das outras medidas do pentagono.

Entao, maos 'a obra!
Tente provar que o angulo CEM e' constante (e faca o favor de postar a
solucao!)

Grande abraco,
Rogerio Ponce



2014-06-26 11:30 GMT-03:00 luiz silva :

> Pessoal,
>
> Descobri o seguinte teorema em um EXCELENTE livro de geometria peruano,
> que um amigo comprou : dado um quadrilátero convexo qqer, construa 4
> quadrados "externos" ao mesmo, onde cada lado do quadrilatero seja um dos
> lados de um dos quadrados. Una os centros dos quadrados opostos. O teorema
> diz que os segmentos que unem os centros dos quadrados opostos tem a mesma
> medida e que o angulo entre eles é de 90o.. Ainda não consegui
> demonstra-lo, porem acho que fiz algo interessante :
>  - O teorema é válido para qqer media dos lados do quadrilátero. Então,
> pequemos um dos lados do quadrilátero e dividamos por 2, assim, teremos um
> novo quadrilátero, mas o teorema ainda é válido, faça o mesmo processo
> neste mesmo lado, indefinidamente. Quando o número de iterações tender a
> infinito, a medida de um dos lados do quadrilátero irá tender a zero (um
> ponto).
>
> Ou seja, o quadrilátero tenderá a se tornar um triângulo, o quadrado
> referente a esse lado que foi sendo dividido por dois, torna-se um ponto (o
> vértice desse triangulo), porém, ainda assim, o teorema ainda será válido;
> só que agora, ao invés de termos os centros de 4 quadrados, teremos o
> centro de 3 quadrados e o vértice do traingulo.
> Eu usei esse mesmo raciocínio para resolver um problema clássico de
> ângulos : Dado um pentágono ABCDEF, onde EA=ED e E=90; CB=CD e C=90.
> Calcular o ângulo CEM, onde M é medio de AB.Da mesma forma que acima, fui
> reduzindo a medida do segmento AB, até o mesmo se tornar um ponto. Quando
> isso ocorre, os lados se tornam iguais (quadrado), o segmento EM tende a
> ser congruente a EA e EC tende a ser a diagonal desse quadrado. Ou seja, o
> ângulo CEM = 45o.
> Creio que possamos validar esse método através da geometria analítica:
> essas “propriedades regulares” (cumprimento, ângulos entre retas) são
> função das coordenadas dos pontos envolvidos no problema. E essas
> "propriedades regulares" são descritas por funções contínuas em R. Ainda
> não fiz, mas acho que não deve ser difícil demonstrar que esses resultados
> são válidos mesmo quando um dos cumprimentos envolvidos no problema se
> reduz a zero (a um ponto).
>  Creio que podemos aplicar esse mesmo método para um problema recente
> proposto por um colega, problema este que é uma variação desse problema do
> pentágono.
>
> Abs
> Felipe
>
> --
> Esta mensagem foi verificada pelo sistema de antivírus e
> acredita-se estar livre de perigo.
>

-- 
Esta mensagem foi verificada pelo sistema de antiv�rus e
 acredita-se estar livre de perigo.



[obm-l] Re: [obm-l] Soma trigonométrica

2014-06-02 Por tôpico Rogerio Ponce
Ola' pessoal,
tem um probleminha que se esqueceram de fazer:

http://www.mail-archive.com/obm-l@mat.puc-rio.br/msg52124.html>
http://www.mail-archive.com/obm-l@mat.puc-rio.br/msg52124.html


[]'s
Rogerio Ponce


2014-05-07 8:42 GMT-03:00 Vanderlei Nemitz :

> Alguém tem alguma ideia? Tentei utilizar a fórmula da tangente do arco
> duplo, mas ficou complicado.
>
>
> Mostre que *tg²(1°) + tg²(3°) + tg²(5°) + ...+ tg²(89°)*  é um número
> inteiro.
>
>
> Obrigado!
>
> --
> Esta mensagem foi verificada pelo sistema de antivírus e
> acredita-se estar livre de perigo.

-- 
Esta mensagem foi verificada pelo sistema de antiv�rus e
 acredita-se estar livre de perigo.



Re: [obm-l] ensinando tabelas verdade

2014-04-16 Por tôpico Rogerio Ponce
Ola' pessoal,

"... a  melhor forma de analisar a veracidade é verificando o que a
negativa é : p e ^ ~q (não q) ."

Nem sempre.
'As vezes e' mais confortavel/direto analisarmos a simples equivalencia "q
ou nao p" :  q v ~p .

Usando o mesmo exemplo dado...
(2 = 4) ==> (qualquer homem voa)
e' falso ou verdadeiro?

Como vale ~p  (pois "2=4" e' falso) , entao a assertiva e' verdadeira.


[]'s
Rogerio Ponce



2014-04-14 16:21 GMT-03:00 Pedro José :

> Boa tarde!
>
> Não tenho texto pronto. Mas, é um pouco mais complicado que
>
> *e , ou.*
>
> p
>
> q
>
> P ==> q
>
> V
>
> V
>
> V
>
> V
>
> F
>
> F
>
> F
>
> V
>
> V
>
> F
>
> F
>
> V
>
>
> P (F) e Q (F ou V, tanto faz) ==> P ==>Q (V)
>
> Exemplo: 2 = 4 ==> qualquer homem voa (V)
>
> Embora entenda que a  melhor forma de analisar a veracidade é verificando
> o que a negativa é.
>
> p e ^ ~q (não q)
>
>
> Para o exemplo acima:
>
> 2 = 4 e Existe pelo menos um homem que não voa (F e V) ==> (F) se a
> negativa é F, assertiva é V.
>
> x^2 pertence 2 |N ==> x pertence a 2 |N.
>
> Analise a negativa.
>
> x^2 pertence a 2 |N e x  pertence a 2|N +1
>
> x^2 pertence a 2|N ==> x^2 ≡ 0 mod 2.  e x ≡ 1 mod2 (absurdo), pois se  x
> ≡ 1 mod2 temos que  x^2 ≡1  mod 2 (conservação da multiplicação)
>
>
> Então só temos p (F) e q (V) ou p (V) e q (F), pois 2 pertence a |P
> (conjunto dos primos).
>
>
> Ou poder-se-ia analizar Existe pelo menos um x^2 pertencente a 2|N e x
> pertence a 2 |N + 1. Isso é falso.
>
>
> x pertence a 2|N+1 ==> existe k pertencente a |N | x = 2k+1 ==> x^2 = 4k^2
> + 4*K + 1= 2 (2k^2+2*K), pelo fechamento da adição, multiplicação e
> potência em \N temos que Existe s = (2k^2+2*K) pertencente a 2|N, logo x^2
> pertence a 2\N+1.
>
>
> Espero que lhe ajude.
>
>
> Saudações,
>
> PJMS
>
>
>
>
>
> Em 20 de abril de 2014 15:28, Hermann  escreveu:
>
>>  Ensinar tabela verdade,
>> é fácl para os conectivos e e ou
>> mas alguém tem uma dica de como ensinar a lógica da tabela verdade da
>> condicional p->q.
>>
>>
>> Abraços
>> Hermann
>>
>> ps: se tiverem um texto pronto de alguém e quiserem mandar para o meu
>> email, agradeço.
>>
>> --
>> Esta mensagem foi verificada pelo sistema de antivírus e
>> acredita-se estar livre de perigo.
>>
>
>
> --
> Esta mensagem foi verificada pelo sistema de antivírus e
> acredita-se estar livre de perigo.

-- 
Esta mensagem foi verificada pelo sistema de antiv�rus e
 acredita-se estar livre de perigo.



[obm-l] Re: [obm-l]Geometria - OBM2012 - Terceira Fase - Nível 2

2014-04-06 Por tôpico PONCE
 Amigo Raphael,
 Vai abaixo uma resolução simplificada.Inicialmente,prova-se
facilmente,EB = EC  e  BEC = 36 graus.Devido a simétria, em
relação a mediatriz do lado CD, conclui-se que o triangulo BME  é
equilátero.Dai   EC = EB =EM e, portanto, conclui-se que E é o
centro de uma circunferência que passa pelos pontos  M,B e C. 
 Das propriedades de ângulos inscrito e central de uma
circunferência, tem-se:  2 BMC = BEC
= 36 graus, o que implica  BMC = 18 graus.
 Ainda da simétria mencionada acima, EMD = BMC = 18
graus.Consequentemente, do vértice M do triãngulo equilátero MNP,  
 CMD = 60 - EMD - BMC = 60 - 36 =
24 graus.RESPOSTA: 24 grausDesculpe-me por qualquer falha e a
resolução simplificada acima.Do sempre amigo
 LUIZ PONCE 
 On Dom 06/04/14 12:15 , Raphael Aureliano raphael0...@gmail.com sent:
Boa tarde pessoal, 
 Alguém poderia me ajudar no problema que segue abaixo?   

Seja ABCDE um pentágono regular inscrito em um triângulo equilatero
MNP, determine o ângulo CMD. 

Na figura, CD está em NP,  B em MN e E em MP.  

Obrigado pela atenção  

Cordialmente,  

Raphael Aureliano 

Praticante de Oficial de Náutica (Piloto)
 Guarda-Marinha (RM-2) 
 -- 
 Esta mensagem foi verificada pelo sistema de antivírus e  
  acredita-se estar livre de perigo. 

-- 
Esta mensagem foi verificada pelo sistema de antiv�rus e
 acredita-se estar livre de perigo.



Re: [obm-l] probabilidade trivial

2014-04-04 Por tôpico PONCE
 Caro amigo,Vai abaixo uma sugestão. Desculpe-me por qualquer
erro.Solução:Sejam os eventos  A: ocorrer um produto que teve
aumento de preço.D: evento complementar de A, isto é, ocorrer um
produto que não teve aumento de preço.
  Para que ao menos três produtos tenham apresentado aumento de
preço no período,ao escolher aleatoriamente 4 produtos, devemos ter
ao menos uma das sequencias de eventos sucessivos abaixo: 
  AAAD, AADA,ADAA,DAAA, Claramente, a
probabilidade de qualquer um deles é dada pelo produto:  (1/2)
x(1/2)x(1/2)x(1/2) = 1/16Portanto, a probabilidade pedida é dada por
5 x 1/16 = 5/16Do sempre amigo.
 LUIZ PONCE 
 On Sex 04/04/14 20:22 , saulo nilson saulo.nil...@gmail.com sent:
 1/16+1/16=1/8
 2014-04-01 19:02 GMT-03:00 João Sousa :
  Gostaria de ajuda para resolver:
Um estudo mostrou que, de 100 produtos pesquisados num determinado
mês, 50 tiveram aumento de preço ao consumidor em relação ao mês
anterior, 30 mantiveram seus preços e 20  
apresentaram redução de preço. 
Se quatro desses produtos forem aleatoriamente escolhidos, com
reposição, qual a probabilidade de que ao menos três tenham
apresentado aumento de preço no período? 
Abs,
João Sousa 
 -- 
 Esta mensagem foi verificada pelo sistema de antivírus e  
  acredita-se estar livre de perigo.  
 -- 
 Esta mensagem foi verificada pelo sistema de antivírus e  
  acredita-se estar livre de perigo. 

-- 
Esta mensagem foi verificada pelo sistema de antiv�rus e
 acredita-se estar livre de perigo.



[obm-l] Re: [obm-l] Um belíssimo problema de geometria....

2014-03-21 Por tôpico Rogerio Ponce
Explicando melhor: alguem que encontre esta questao, vai gastar 15
segundos para dar a resposta correta, bastando considerar uma elipse
com eixo vertical.

Parece-me que o interessante (e mais dificil) e' provar que qualquer
que seja a inclinacao da elipse, o resultado vale sempre "a+b".

[]'s
Rogerio Ponce



2014-03-21 15:27 GMT-03:00 Rogerio Ponce :
> Ola' Douglas,
> com este enunciado e' perfeitamente possivel o entendimento de que
> entre duas retas horizontais estao a circunferencia e a elipse (ao
> lado da circunferencia).
> Assim, a distancia entre as retas seria "2a", por exemplo , e a
> "largura" da elipse seria "2b", de modo que a distancia entre os
> centros seria "a+b".
>
> []'s
> Rogerio Ponce
>
>
> 2014-03-21 10:29 GMT-03:00  :
>> Esse vai em homenagem ao meu amigo Carlos Victor que pediu caridosamente pra
>> que se eu tivesse problemas legais mesmo que já tivesse resolvido , para
>> postar para os camaradas da lista... então gostaria de compartilhar um belo
>> problema de geometria com meus camaradas amantes dessa brilhante disciplina.
>>
>> PROBLEMA: Dados duas retas paralelas , e uma circunferência que as
>> tangencia. Considere uma elipse de eixos maior e menor respectivamente
>> iguais à 2a e 2b que tangência as retas e a circunferência. Calcular o
>> tamanho do segmento que une os centros da elipse e da circunferência.
>>
>>
>>
>> Um abraço do Douglas Oliveira.
>>
>>
>>
>> --
>> Esta mensagem foi verificada pelo sistema de antivírus e
>> acredita-se estar livre de perigo.

-- 
Esta mensagem foi verificada pelo sistema de antivírus e
 acredita-se estar livre de perigo.


=
Instruções para entrar na lista, sair da lista e usar a lista em
http://www.mat.puc-rio.br/~obmlistas/obm-l.html
=


[obm-l] Re: [obm-l] Um belíssimo problema de geometria....

2014-03-21 Por tôpico Rogerio Ponce
Ola' Douglas,
com este enunciado e' perfeitamente possivel o entendimento de que
entre duas retas horizontais estao a circunferencia e a elipse (ao
lado da circunferencia).
Assim, a distancia entre as retas seria "2a", por exemplo , e a
"largura" da elipse seria "2b", de modo que a distancia entre os
centros seria "a+b".

[]'s
Rogerio Ponce


2014-03-21 10:29 GMT-03:00  :
> Esse vai em homenagem ao meu amigo Carlos Victor que pediu caridosamente pra
> que se eu tivesse problemas legais mesmo que já tivesse resolvido , para
> postar para os camaradas da lista... então gostaria de compartilhar um belo
> problema de geometria com meus camaradas amantes dessa brilhante disciplina.
>
> PROBLEMA: Dados duas retas paralelas , e uma circunferência que as
> tangencia. Considere uma elipse de eixos maior e menor respectivamente
> iguais à 2a e 2b que tangência as retas e a circunferência. Calcular o
> tamanho do segmento que une os centros da elipse e da circunferência.
>
>
>
> Um abraço do Douglas Oliveira.
>
>
>
> --
> Esta mensagem foi verificada pelo sistema de antivírus e
> acredita-se estar livre de perigo.

-- 
Esta mensagem foi verificada pelo sistema de antivírus e
 acredita-se estar livre de perigo.


=
Instruções para entrar na lista, sair da lista e usar a lista em
http://www.mat.puc-rio.br/~obmlistas/obm-l.html
=


Re: [obm-l] Conjecturas especiosas

2013-10-24 Por tôpico Rogerio Ponce
Nao funciona para n=41, por exemplo.

[]'s
Rogerio Ponce


2013/10/23 marcone augusto araújo borges 

> Não sei se ajuda muito mas n^2 - n + 41 parece que gera só primos
>
> --
> Date: Wed, 23 Oct 2013 18:38:11 -0300
> Subject: [obm-l] Conjecturas especiosas
> From: rigillesbmene...@gmail.com
> To: obm-l@mat.puc-rio.br
>
> Olá pessoal, não vou falar sobre nenhuma questão, só fazer uma pergunta.
> Que conjecturas vocês conhecem que parecem muito ser verdade, sem o ser?
> Tenho procurado por elas para ter alguns exemplos da importância de provar
> coisas na matemática para mostrar a meus amigos :)
> Abraços,
> Rígille Scherrer Borges Menezes
>
> --
> Esta mensagem foi verificada pelo sistema de antivírus e
> acredita-se estar livre de perigo.
>
> --
> Esta mensagem foi verificada pelo sistema de antivírus e
> acredita-se estar livre de perigo.
>

-- 
Esta mensagem foi verificada pelo sistema de antivírus e
 acredita-se estar livre de perigo.



Re: [obm-l] Problemas interessantes

2013-08-23 Por tôpico PONCE
 Obrigado Benedito,
 pelos belos problemas. 
 LUIZ PONCE 
 On Qui 22/08/13 04:39 , "Benedito" bened...@ufrnet.br sent:
Segue dois problemas interessantes.

Benedito
Problema 1

Um triângulo equilátero de lado 2012 está dividido em 2012 
triângulos equiláteros menores de lado 1

mediante paralelas ao seus lados. Em cada vértice de um triângulo
menor há uma formiga. No mesmo instante, 

todas as formigas começam a caminhar com a mesma velocidade pelas
retas da triangulação.

Ao chegar  a outro vértice giram 60º ou 120º à esquerda ou à
direita e  seguem movendo-se.

Determinar se é possível que este movimento se desenvolva para
sempre sem ter  nunca duas

formigas em um mesmo vértice de um triângulo menor.
Problema 2

Associar aos vértices de um polígono convexo de 33 lados os
números inteiros de 1 a 33, sem repetir, e em seguida, associar aos
lados do polígono a soma dos números de seus extremos. 

O objetivo é que os números associados aos lados sejam 33 inteiros
consecutivos ordenados.
 -- 
 Esta mensagem foi verificada pelo sistema de antivírus e  
  acredita-se estar livre de perigo. 

-- 
Esta mensagem foi verificada pelo sistema de antiv�rus e
 acredita-se estar livre de perigo.



Re: [obm-l] Problema para (quase) iniciantes: FIGURA

2013-08-21 Por tôpico Rogerio Ponce
Segue a figurinha do problema...
[]'s
Rogerio Ponce


2013/8/20 Nehab 

>  Oi, amigos,
>
> O seguinte problema foi proposto no "Canguru - 2013 - Nível "Estudante" -
> Q11, e permite uma generalização legal pros alunos iniciantes (ou quase
> iniciantes).
> (Há referência ao "Canguru brasileiro" no site da OBM:
> http://www.cangurudematematicabrasil.com.br/ mas o problema a seguir foi
> obtido no site http://www.mat.uc.pt/canguru/
>
> Vamos, inicialmente, ao problema propriamente dito:
>
> *Ana tem várias peças idênticas com a forma de um pentágono re­gular e as
> cola, face a face, de modo a completar um aro circular, como representado
> na figura. *
>
>
>
>
>  *Quantas peças possui o aro assim construído?**
> A)** 8  B) 9  C) 10** **D)** 11E) 12**
> *
>
>
> *Generalização* (me corrijam, se necessário ou generalizem mais ainda...)
> a) Mostre que se for exigido que a figura “interna” ao aro seja um
> polígono convexo (no exercício proposto, será um decágono regular) as
> únicas “peças” polígonos regulares convexos que permitem que se construa um
> aro (fechado, é claro) são o próprio pentágono, o hexágono, o octógono e o
> dodecágono.
> b) (sem dicas)
> Se for permitido que a figura “interna ao aro” seja um polígono estrelado,
> o problema fica muito, mas muito mais interessante.
> Investigue essa situação.
>
>
>
> --
> Esta mensagem foi verificada pelo sistema de antivírus e
> acredita-se estar livre de perigo.
>

-- 
Esta mensagem foi verificada pelo sistema de antivírus e
 acredita-se estar livre de perigo.

<>

[obm-l] Re: [obm-l] Re: [obm-l] Re: [obm-l] Re: [obm-l] Re: [obm-l] Uma variação do Problema de Monty Hall

2013-08-16 Por tôpico Rogerio Ponce
Pois e', o Ralph sofre dessa mania:
 e' sempre muito didatico, e apresenta explicacoes que fazem tudo parecer
extremamente simples...
:)

Abracao,
Rogerio Ponce


2013/8/14 Ralph Teixeira 

> Ah, Luiz, mas ai que eu discordo -- o ato de conferencia te traz nova
> informacao. Isto altera SIM as probabilidades.
>
> Vamos ao jogo com 3 cartas numeradas de 1 a 3. Eu escolho 1, voce fica com
> as outras 2. Quem tiver a carta 3 ganha.
>
> Quando o jogo comeca, sim, a probabilidade de eu ganhar eh apenas 1/3.
>
> Agora, voce escolhe uma de suas cartas aleatoriamente e me mostra. Digamos
> que voce mostrou a carta 2: as probabilidades mudam! Nova informacao induz
> novas probabilidades! Agora as probabilidades sao 50% para cada um.
>
> Mas note que isto depende IMENSAMENTE de COMO voce escolhe a carta. Supus
> ali em cima que voce escolheu sem olhar. Se voce olhou as suas cartas e
> escolheu a menor, bom, entao sua chance de ganhar eh 100% (se a menor eh 2,
> certamente voce tem a 3). Se voce olhou suas cartas e escolheu a maior,
> entao eh 0% para voce e 100% para mim.
>
> Em suma -- no problema dos bodes, eh essencial saber COMO o cara abriu a
> porta do bode. Se ele seguiu a regra "nunca abra o carro", eh uma coisa. Se
> ele "abriu aleatoriamente e apareceu um bode", eh outra.
>
> Abraco,
> Ralph
>
> P.S.: Outro experimento que gosto de fazer com meus alunos: eu jogo 2
> moedas de forma que o Joao consiga ver uma delas. Ai eu pergunto pra Maria
> que estah lah do outro lado da sala "Qual a probabilidade de ambas serem
> "Cara"?". Maria responde 25%; entao eu pergunto pro Joao, que estah vendo
> uma cara Ele pensa um pouco e diz "50%" porque na pratica ele soh estah
> se perguntando sobre a outra moeda. Entao eu olho as duas e digo "A
> probabilidade eh 0%."... Foi a mesma pergunta 3 vezes, com 3 respostas
> diferentes, quem estah errado? Resposta: NINGUEM estah errado -- o fato eh
> que probabilidades DEPENDEM da informacao que voce tem!
>
>
> 2013/8/14 luiz silva 
>
>> Pense da seginte forma :
>>
>> Após a escolha inicial, um tem 1/3 de ganhar ou outro2/3. Após isso, vem
>> a conferência (cada um olha as suas cartas para verificar quem ganhou).
>> Imagine que a pessoa com 2 cartas faça a sua conferência antes da que tem 1
>> carta. Ela vira a primeira carta e ve que não é a premiada. As suas chances
>> continuam sendo 2/3 de ganhar, pois o ato de verificação (ver se a carta
>> premiada está ou não com vc) não altera as probabilidades iniciais, pois o
>> espaço amostral continua o mesmo e o as escolhas iniciais também. Agora, se
>> após a primeria verificação houver o embaralhjamento e nova escolha, aí
>> cada um terá 50% de cahnces, pois houve alteração do espaço amostral e das
>> escolhas efetuadas.
>>
>> Reumo :A verificação não altera a probabilidade; virando-se as duas
>> cartas em sequencia ou ao mesmo tempo, a probabilidade de ganho continua
>> sendo 2/3 para quem escolheu duas cartas e 1/3 para quem escolheu uma
>> carta. Assim, trocar a carta é probabilisticamente vantajoso para quem
>> esolheu somente uma carta.
>>
>> Dica : imagine a mesma situação com 100 cartas; uma pessoa com 1 e outra
>> com 99; se for conferir uma a uma, vai levar um tempo, mas a chance da
>> carta vencedora estar em uma das 99 é 99%. Então, a última carta a ser
>> conferida continua dando 99% de chances de ganho para quem escolheu as 99
>> cartas.
>>
>> Abs
>> Felipe
>>   *De:* Bob Roy 
>> *Para:* obm-l@mat.puc-rio.br
>> *Enviadas:* Quarta-feira, 14 de Agosto de 2013 10:03
>> *Assunto:* [obm-l] Re: [obm-l] Re: [obm-l] Uma variação do Problema de
>> Monty Hall
>>
>>  Olá ,
>>
>> desculpem, mas fiquei confuso; então quer dizer que ( nas novas
>> condições) trocando ou não , ele fica com a chance de ganhar igual a 1/3; é
>> isso ?
>> abs
>> Bob
>>
>>
>> Em 13 de agosto de 2013 20:56, Ralph Teixeira escreveu:
>>
>> Bom, no problema original eh importante ressaltar as regras: o
>> apresentador nunca abre a porta do espectador nem a porta do carro. Nessa
>> nova versao, ele nao tem como seguir a segunda regra. Na nova versao, se
>> ele abriu um bode, a probabilidade de cada porta eh 1/2.
>> Muito vagamente, funciona assim:
>> No problema classico, de cada 900 shows, o apresentador deixa um bode na
>> outra porta fechada 300 vezes (sempre que o espectador acerta o carro de
>> primeira) e deixa um carro na outra porta fechada 600 vezes.
>> Agora, na sua nova versao, de cada 900 shows, o apresentador deixa um
>> bode na out

[obm-l] Re: [obm-l] Re: [obm-l] Re: [obm-l] Análise combinatória

2013-07-12 Por tôpico Rogerio Ponce
Ola' Marcos,
eu escrevi errado.
Como os "blocos" representam 4 elementos, que ocupam 7 casas, e' como se
houvesse 93 casas livres e 4 ocupadas, com um total de 100-(2+2+2+1)+4=97
casas.
Ou seja, existem binom(97,4) formas de distribuirmos os 4 blocos dentro de
[1,100].

[]'s
Rogerio Ponce


2013/7/12 Marcos Martinelli 

> Só não entendi essa parte: "100-(2+2+2+1)=97".
>
>
> Em 12 de julho de 2013 09:08, Marcos Martinelli 
> escreveu:
>
>> Legal.
>>
>>
>> Em 12 de julho de 2013 09:02, Rogerio Ponce escreveu:
>>
>> Ola' Artur,
>>> como queremos que a distancia minima entre os elementos seja de pelo
>>> menos 2, podemos imaginar que devemos distribuir , dentro do segmento
>>> [0,100], 3 "blocos" com comprimento 2 , e um bloco com comprimento 1 (o
>>> bloco mais 'a direita).
>>> Como existem 100-(2+2+2+1)=97 vagas, o resultado vale
>>> binom(97,4)=3464840.
>>>
>>> []'s
>>> Rogerio Ponce
>>>
>>>
>>> 2013/7/11 Artur Costa Steiner 
>>>
>>>> Não consegui achar uma forma de resolver isto sem recorrer a um
>>>> computador.
>>>>
>>>> Com os inteiros de 1 a 100, quantos conjuntos de 4 elementos podemos
>>>> formar de modo que a diferença positiva entre dois elementos do conjunto
>>>> seja maior ou igual a 2?
>>>>
>>>> Abraços.
>>>>
>>>> Artur Costa Steiner
>>>> --
>>>> Esta mensagem foi verificada pelo sistema de antivírus e
>>>>  acredita-se estar livre de perigo.
>>>>
>>>>
>>>>
>>>> =
>>>> Instruções para entrar na lista, sair da lista e usar a lista em
>>>> http://www.mat.puc-rio.br/~obmlistas/obm-l.html
>>>>
>>>> =
>>>>
>>>
>>>
>>> --
>>> Esta mensagem foi verificada pelo sistema de antivírus e
>>> acredita-se estar livre de perigo.
>>>
>>
>>
>
> --
> Esta mensagem foi verificada pelo sistema de antivírus e
> acredita-se estar livre de perigo.

-- 
Esta mensagem foi verificada pelo sistema de antivírus e
 acredita-se estar livre de perigo.



[obm-l] Re: [obm-l] Análise combinatória

2013-07-12 Por tôpico Rogerio Ponce
Ola' Artur,
como queremos que a distancia minima entre os elementos seja de pelo menos
2, podemos imaginar que devemos distribuir , dentro do segmento [0,100], 3
"blocos" com comprimento 2 , e um bloco com comprimento 1 (o bloco mais 'a
direita).
Como existem 100-(2+2+2+1)=97 vagas, o resultado vale binom(97,4)=3464840.

[]'s
Rogerio Ponce


2013/7/11 Artur Costa Steiner 

> Não consegui achar uma forma de resolver isto sem recorrer a um
> computador.
>
> Com os inteiros de 1 a 100, quantos conjuntos de 4 elementos podemos
> formar de modo que a diferença positiva entre dois elementos do conjunto
> seja maior ou igual a 2?
>
> Abraços.
>
> Artur Costa Steiner
> --
> Esta mensagem foi verificada pelo sistema de antivírus e
>  acredita-se estar livre de perigo.
>
>
> =
> Instruções para entrar na lista, sair da lista e usar a lista em
> http://www.mat.puc-rio.br/~obmlistas/obm-l.html
> =
>

-- 
Esta mensagem foi verificada pelo sistema de antivírus e
 acredita-se estar livre de perigo.



Re: [obm-l] Primos

2013-07-12 Por tôpico Rogerio Ponce
Ola' Marcos,
todo primo (maior que 3) e' da forma 6k+1 ou 6k-1, assim como todo impar e'
da forma 2k+1.

[]'s
Rogerio Ponce

PS: Dizer que a soma de dois primos consecutivos, A e B, seria o dobro de
um terceiro primo, C,
e' o mesmo que dizer que C e' a media entre A e B, que necessariamente se
situa entre A e B (inclusive).
Como A e B sao consecutivos, C nao pode estar entre eles.

[]'s,
Rogerio Ponce


2013/7/12 Marcos Martinelli 

> Acho que não existe uma fórmula fechada para os primos.
>
> Acho que tentamos encontrá-la há um bom tempo... mas sem sucesso, apesar
> de inúmeras outras portas que foram abertas com a teoria analítica dos
> números.
>
> Em sexta-feira, 12 de julho de 2013, Nehab escreveu:
>
>>  Oi, Marcone,
>>
>> Números primos são da forma 6k - 1 ou 6k + 1.
>> Imediato...
>>
>> Nehab
>>
>> On 11/07/2013 23:16, marcone augusto araújo borges wrote:
>>
>> Mostre que a soma de dois primos consecutivos nunca é  o dobro de um
>> primo
>>Peço ajuda.
>>
>> --
>> Esta mensagem foi verificada pelo sistema de antivírus e
>> acredita-se estar livre de perigo.
>>
>>
>>
>>
>> --
>> Esta mensagem foi verificada pelo sistema de antivírus e
>> acredita-se estar livre de perigo.
>>
>
> --
> Esta mensagem foi verificada pelo sistema de antivírus e
> acredita-se estar livre de perigo.

-- 
Esta mensagem foi verificada pelo sistema de antivírus e
 acredita-se estar livre de perigo.



Re: [obm-l] Probabiliedade

2013-07-09 Por tôpico Rogerio Ponce
Ola' pessoal,
existem 4*3/2 = 6 jogos diferentes, de modo que serao 6 vitorias, ao todo.
Portanto, 4 times nao poderao ter  a mesma quantidade de vitorias (1.5
vitorias para cada time).

Para JUBA terminar isolado em primeiro, nao bastam 2 vitorias (pois
necessariamente havera' algum outro time com pelo menos 2 vitorias). Assim,
JUBA precisa ter 3 vitorias (ganhou os 3 jogos que disputou), e cada um dos
outros 3 jogos pode ter qualquer resultado. Assim, seriam 2*2*2=8 formas
diferentes de JUBA ser campeao isolado, num total de 2**6 resultados
possiveis.
Logo, a chance de JUBA isolado em primeiro e' de 8/64=1/8.

Para que o torneio tenha 3 times em primeiro, cada um deles teria que ter 2
vitorias (apenas 1 vitoria nao pode, pois neste caso, o quarto time teria 3
vitorias, e seria ele o campeao).
Assim, o quarto time perdeu todas, e entre os 3 primeiros houve um
"encadeamento de vitorias" (A ganhou de B que ganhou de C que ganhou de A).
Como so' existem 2 permutacoes assim, existem apenas 2 formas de 3 times
dados serem campeoes. Como ha' 4 escolhas para estes 3 times (ha' 4
escolhas para o time perdedor), entao existe o total de 8 formas de 3 times
serem campeos, num total de 64 resultados possiveis.
Logo, a chance e' de 1/8.

[]'s
Rogerio Ponce


2013/7/9 Pedro Júnior 

> 1. Quatro times, entre os quais o JUBA, disputam um torneio de vôlei em
> que:
>
>- Cada time joga contra cada um dos outros uma única vez;
>- Qualquer partida termina com a vitória de um dos times;
>- Em qualquer partida, os times têm a mesma probabilidade de ganhar;
>- Ao final do torneio, os times são classificados em ordem de vitórias.
>
> a)  É possível que, ao final do torneio, todos os times tenham o mesmo
> número de vitórias? Por quê?
>
> b)  Qual é a probabilidade de que o torneio termine com o JUBA isolado
> em primeiro lugar?
>
> c)   Qual é a probabilidade de que o torneio termine com três times
> empatados em primeiro lugar?
>
> --
> Esta mensagem foi verificada pelo sistema de antivírus e
> acredita-se estar livre de perigo.

-- 
Esta mensagem foi verificada pelo sistema de antivírus e
 acredita-se estar livre de perigo.



Re: [obm-l] Probabilidade - II

2013-07-09 Por tôpico Rogerio Ponce
Ola' pessoal,
no primeiro problema, existe apenas uma distribuição com a bola preta em
primeiro, segundo terceiro, ou quarto. Assim, todos tem a mesma chance
(1/4) de ganhar.

No segundo problema - 2 bolas pretas num total de 8 casas - existem
8*7/2=28 formas de distribuir as bolas pretas, das quais, na primeira
rodada, 7 correspondem ao ganho de Andre, 6 ao ganho de Bianca, 5 ao ganho
de Carlos e 4 ao ganho de Dalva.
E, na segunda rodada, 3 correspondem ao ganho de Andre (1 bola preta na 5a
posicao, e outra em 6,7, ou 8), 2 correspondem ao ganho de Bianca, 1
corresponde ao ganho de Carlos, e 0 para Dalva.

Assim, as probabilidades de ganho sao:
Andre = (7+3)/28 = 10/28
Bianca=(6+2)/28 = 8/28
Carlos=(5+1)/28 = 6/28
Dalva=(4+0)/28 = 4/28

[]'s
Rogerio Ponce

2013/7/9 Pedro Júnior 

> 2. André, Bianca, Carlos e Dalva querem sortear um livro entre si. Para
> isto, colocaram 3 bolas brancas e 1 preta em uma caixa e combinaram que, em
> ordem alfabética de seus nomes, cada um tirará uma bola, sem devolvê-la à
> caixa. Aquele que tirar a bola preta ganhará o livro.
>
> a)  Qual é a probabilidade de que André ganhe o livro?
>
> b)  Qual é a probabilidade de que Dalva ganhe o livro?
>
>
>
> Para sortear outro livro entre eles, André sugeriu usar 2 bolas pretas e 6
> brancas. Como antes, o primeiro que tirar uma bola preta ganhará o livro;
> se as primeiras quatro bolas saírem brancas, eles continuarão a retirar
> bolas, na mesma ordem. Nesse novo sorteio:
>
> a)  Qual é a probabilidade de que André ganhe o livro?
>
> b)  Qual é a probabilidade de que Dalva ganhe o livro?
>
> --
> Esta mensagem foi verificada pelo sistema de antivírus e
> acredita-se estar livre de perigo.

-- 
Esta mensagem foi verificada pelo sistema de antivírus e
 acredita-se estar livre de perigo.



[obm-l] Re: [obm-l] Re: [obm-l] Re: [obm-l] Duas perguntas(teoria dos números)

2013-05-30 Por tôpico Rogerio Ponce
Ola' Jeferson,
parece que existe algum engano no enunciado do seu problema 2,
pois nao existe potencia de 2, com mais de 1 algarismo, que termine em 0 ou
1.

[]'s
Rogerio Ponce




2013/5/29 Jeferson Almir 

> Aproveitando o momento tenho pensado nestes 2 problemas há tempos:
>
> 1. Prove que para todo inteiro positivo a>1 existem infinitos inteiros
> positivos n tais n/aˆ(n)+1.
>
> 2. Prove que existe uma potência de 2 cujos k primeiros algarismos da
> direita para esquerda são iguais a 0 ou 1.
>
> Esse foi o meu primeiro contato com T.N apresentado pelo professor Luiz
> Amancio em sua passagem por Fortaleza e desde então nunca mais fui o mesmo.
> Desde ja agradeço qualquer ajuda.
>
>
> Em 27 de maio de 2013 20:23, Ralph Teixeira  escreveu:
>
>> 1) Suponha, por contradicao, que 2^(x1)+2^(x2)+...+2^(xn)=2^A para
>> x1,x2,...,xn naturais distintos (suponha s.p.d.g. que x1> n>=2).
>>
>> Por um lado, A>B, porque o lado esquerdo eh claramente maior que 2^B;
>> entao A>=B+1.
>> Por outro lado, mesmo que voce use TODAS as potencias de 2 ateh 2^B, nao
>> chega a 2^A. De fato, o lado esquerdo eh menor ou igual a:
>> 1+2+2^2+...+2^B=2^(B+1)-1<2^(B+1)<=2^A (usei soma dos termos da P.G. ali).
>>
>> Entao realmente nao tem como valer a igualdade.
>>
>> 2) Pois eh, como voce viu, m-48=2^a e m+48=2^(a+b) onde b>0 (pois
>> m+48>m-48). Para mostrar que seu par eh o unico que presta, faca a
>> subtracao, como voce disse, e fatore:
>>
>> 2^a (2^b-1) = 96
>>
>> Agora, 2^a SOH TEM FATORES 2, enquanto 2^b-1 eh impar e portanto NAO TEM
>> FATORES 2. Entao, 2^a tem que ser TODA a "parte potencia de 2" na fatoracao
>> de 96=(2^5) * 3, enquanto 2^b-1 tem que ser TODA a parte impar (isto eh,
>> 3). Entao, a unica solucao eh a=5 e b=2, como voce havia dito.
>>
>> Abraco,
>>   Ralph
>>
>>
>> 2013/5/27 marcone augusto araújo borges 
>>
>>> 1) Gostaria de saber se a soma de duas ou mais potencias de base 2
>>> distintas pode ser uma potencia de base 2.
>>>
>>> Acredito que não e escrevendo esses números na base 2 talvez se possa
>>> mostrar isso.
>>>
>>> 2) Desconfio que   2304 + 2^n é um quadrado perfeito para um único valor
>>> de n.
>>>
>>> Eu fiz 2^n = (m + 48)(m - 48)
>>> m + 48 e m - 48 devem ser potencias de base 2
>>> As únicas potencias de base 2 cuja diferença é 96 são 128 e 32
>>> Dai o único valor de n seria 12
>>> Um esclarecimento seria muito bem vindo
>>>
>>> Desde já agradeço
>>>
>>>
>>>
>>
>>
>


[obm-l] Re: [obm-l] Triângulo retângule e bissetrizes

2013-05-19 Por tôpico Rogerio Ponce
Ola' Martins,
a partir de seu vertice, cada bissetriz encontra a outra bissetriz, e entao
o lado oposto.
As medidas se referem a quais segmentos?

[]'s
Rogerio Ponce



2013/5/13 Martins Rama 

> Olá amigos da lista...
> Obrigado pelas colaborações.
>
> Alguém pode me ajudar nessa questão?
>
> "Calcular a área de um triângulo retângulo, sabendo que as bissetrizes dos
> ângulos agudos medem sqr(13) e sqr(104)."
>
> []'s
>
> Martins Rama.
>
>
>
> =
> Instruções para entrar na lista, sair da lista e usar a lista em
> http://www.mat.puc-rio.br/~obmlistas/obm-l.html
> =
>


[obm-l] Re: [obm-l] Preciso de uma opinião a respeito do enunciado de uma questão!!!

2013-05-17 Por tôpico Rogerio Ponce
Ola' Douglas,

"para cada um dos voluntarios, a nutiriconista usou uma correspondencia
diferente entre numeros e letras para uma mesma amostra do produto".
Existem 5! = 120 formas de se estabelecer a correspondencia entre os 5
numeros e as 5 letras.
E como sao 5 amostras, a probabilidade de que a escolhida seja uma em
particular vale 1/5 = 20%

Estas seriam as minhas respostas.
Entretanto, a primeira pergunta pode dar margem a se pensar na
correspondencia entre numeros e amostras, associada 'a correspondencia
entre letras e amostras. E nesse caso, a resposta seria 5! * !5 = 14400.
Mas para isso, o enunciado deveria estar escrito mais ou menos assim:
"...a nutricionista usou uma correspondência diferente entre números,
letras, e as amostras do produto."

Portanto, eu fico com a primeira resposta mesmo.

[]'s
Rogerio Ponce


2013/5/16 

> **
>
> Primeiramente vim aqui nesta lista pois sei que aqui existem muitos
> cérebros fantásticos(Grandes professores e Grandes olimpicos)
>
> , e pessoas de alto nivel que com certeza me trariam
>
> um forte opinião a respeito de um problema, e que seus argumentos com
> certeza iriam valer para uma possível mudança na banca.
>
> Caiu recentemente uma questão na universidade católica de Brasília e na
> minha opinião e de alguns colegas
>
> matemáticos achamos que a questão está com o enunciado confuso e que a
> resposta adotada por eles não convém ,
>
> e ouvi dizer que eles nunca anulam questão( Dizem que sempre estão
> certos), o que seria contra constituição, gostaria de saber a opinião a
> respeito da questão
>
> que colocarei o enunciado abaixo( se puderem é claro), Ai vai:
>
> QUESTÃO 22
>
> Uma nutricionista desejava testar a futura aceitação de uma barra de
> cereais que ela pretendia produzir comercialmente.
>
> Ela preparou cinco amostras com as diferentes possibilidades de composição
> da barra de cereais e então organizou dois testes com voluntários.
>
> o Primeiro teste visava avaliar qual das amostras teria melhor aparência e
> o segundo avaliaria qual delas seria reconhecida como a de melhor sabor.
>
> A fim de evitar a associação das diferentes amostras de um teste para
> outro pelos voluntários, ela usou um código alfanumérico para identificar
> as amostras.
>
> No primeiro teste, as amostras foram numeradas como 101, 102, 103, 104 e
> 105. Já no segundo teste, as amostras eram identificadas apenas por uma
> letra de
>
> a até e. Para cada um dos voluntários, a nutricionista usou uma
> correspondência diferente entre números e letras para uma mesma amostra do
> produto. Calcule
>
> então o número máximo de voluntários que ela pôde usar nesse experimento.
>
>
>
> QUESTÃO 23
>
> Considere o que foi apresentado na questão anterior. Uma das amostras
> testadas será escolhida ao acaso para um segundo teste com vários
> consumidores,
>
> a fim de comparar os resultados com o teste que será feito com a amostra
> escolhida no experimento descrito acima: Qual é a probabilidade, em
> porcentagem,
>
> de que a amostra escolhida aleatoriamente seja a que teve a pior aceitação
> no teste realizado?
>
>
>
> Agradeço a ajuda desde já
>
>
>
> Douglas Oliveira.
>
>


Re: [obm-l] Torneio das Cidades 94

2013-05-05 Por tôpico Rogerio Ponce
Oi Terence,
usando os casos 2 e 3, vemos que seria possivel haver ate' 21 avos
diferentes.
[]'s
Rogerio Ponce


2013/5/5 terence thirteen 

> Minha ideia era algo como uma indução: provar que só existem três avós (ou
> menos). Com quatro fica fácil, e a partir daí, vemos que somos obrigados a
> repetir as avós já usadas.
>
>
> Em 28 de abril de 2013 06:07, Rogerio Ponce  escreveu:
>
>> Ola' pessoal,
>> achei conveniente explicar melhor a solucao...
>>
>> Problema:
>> Existem 20 alunos em uma escola.
>> Quaisquer dois deles possui uma avo' em comum.
>> Prove que pelo menos 14 deles possui uma avo' em comum.
>>
>> Solucao:
>> Seja Antonio um dos 20 alunos.
>> Sejam Maria e Nair suas avos.
>> Portanto, qualquer outro aluno tem uma avo' Maria, ou Nair, ou Maria e
>> Nair simultaneamente.
>>
>> Assim, podemos encaixar cada aluno em um dos grupos A, B e C, definidos
>> da seguinte forma:
>>  - grupo A: o aluno tem uma avo' Maria, e uma avo' Nair.
>> - grupo B: o aluno tem uma avo' Maria, e uma outra avo' que nao e' Nair.
>>  - grupo C: o aluno tem uma avo' Nair, e uma outra avo' que nao e' Maria.
>>
>> Dessa forma, Antonio pertence ao grupo A, e existem 4 possibilidades:
>> 1) os grupos B e C sao vazios:
>>  neste caso, tanto Maria quanto Nair sao avos dos 20 alunos.
>>
>> 2) apenas o grupo C e' vazio:
>>  neste caso, Maria e' avo' dos 20 alunos.
>>
>> 3) apenas o grupo B e' vazio:
>>  neste caso, Nair e' avo' dos 20 alunos.
>>
>> 4) nenhum grupo e' vazio:
>> neste caso, seja Bernardo um aluno do grupo B, e seja Odete a sua outra
>> avo' ( aquela que nao e' Nair).
>> Acontece que qualquer que seja o aluno "c" do grupo C, "c" e Bernardo
>> possuem uma avo' em comum.
>> Como uma avo' de "c" e' Nair, a outra, aquela que nao pode ser Maria, tem
>> que ser Odete.
>> Portanto, todos os alunos do grupo C sao netos de Odete.
>>
>> Agora, seja Camilo um aluno do grupo C.
>> Entao, conforme acabamos de provar, Camilo tem as avos Nair e Odete.
>> Acontece que qualquer que seja o aluno "b" do grupo B, "b" e Camilo
>> possuem uma avo' em comum.
>> Como uma avo' de "b" e' Maria, a outra, aquela que nao pode ser Nair, tem
>> que ser Odete.
>> Portanto todos os alunos do grupo B sao netos de Odete.
>>
>> Desse modo, apenas 3 mulheres - Maria, Nair e Odete - sao as unicas avos
>> de todos os alunos.
>> Como os 20 alunos possuem ao todo 40 avos, uma das 3 mulheres tem que ser
>> avo' pelo menos 14 vezes, ou seja, tem que ser avo' de pelo menos 14 alunos
>> (principio das casas de pombos).
>>
>> Assim, examinadas as possibilidades, sempre podemos afirmar que pelo
>> menos 14 alunos possuem uma avo' em comum.
>>
>> []'s
>> Rogerio Ponce
>>
>>
>
>
> --
> /**/
> 神が祝福
>
> Torres
>


Re: [obm-l] Problema de Geometria

2013-04-28 Por tôpico Rogerio Ponce
Ola' Wagner,
eu "comi" uma letra, e o nome correto e' Teorema de Menelaus.
Embora simplissimo, ele resolve o problema sem necessidade de qualquer
linha auxiliar.
E um site que eu acho interessante e' o http://mathworld.wolfram.com
Abracos,
Rogerio Ponce


2013/4/28 Wagner 

> **
> Olá senhores
> Desculpe a minha pergunta
> Há tempos sou assinante desta, e por vezes vejo termos, que para mim são
> desconhecidos, como por exemplo o teorema abaixo citado.
> Sou graduando em matemática 4º semestre  ( licenciatura )
> Daí pergunto:
> Haveria algum site , ou coleção de livros , ou DVD's , que abranje " todos
> os tópicos de matemática "
> Grato
> Wagner
> PY2RPD
>
> - Original Message -
> *From:* Rogerio Ponce 
> *To:* obm-l@mat.puc-rio.br
> *Sent:* Sunday, April 28, 2013 9:17 PM
> *Subject:* Re: [obm-l] Problema de Geometria
>
>  Ola' Raphael,
> e' so' aplicar o Teorema de Menlaus ao triangulo AOD com a reta CB,
> obtendo:
> AC * FD * OB = DC * OF * AB
> ou seja
> FD = 2 * OF
> Como EF = OE - OF
> entao EF = (a/2) - (b/3)
>
> []'s
> Rogerio Ponce
>
>
> 2013/4/28 Raphael Feijao 
>
>>  O segmento AB é o diametro de uma circunferencia de centro O. Toma-se
>> um ponto C desse círculo e prolonga-se o segmento AC de um segmento CD
>> igual a AC. O segmento OD corta a circunferencia em E e corta o segmento BC
>> em F. Se AB=a e OD=b. Calcule EF.
>>
>
>
>
> __ Informação do ESET Smart Security, versão da vacina 8276
> (20130428) __
>
> A mensagem foi verificada pelo ESET Smart Security.
>
> part000.txt - esta OK
> part001.htm - esta OK
>
> http://www.eset.com
>


Re: [obm-l] Problema de Geometria

2013-04-28 Por tôpico Rogerio Ponce
Ola' Raphael,
e' so' aplicar o Teorema de Menlaus ao triangulo AOD com a reta CB, obtendo:
AC * FD * OB = DC * OF * AB
ou seja
FD = 2 * OF
Como EF = OE - OF
entao EF = (a/2) - (b/3)

[]'s
Rogerio Ponce


2013/4/28 Raphael Feijao 

> O segmento AB é o diametro de uma circunferencia de centro O. Toma-se um
> ponto C desse círculo e prolonga-se o segmento AC de um segmento CD igual a
> AC. O segmento OD corta a circunferencia em E e corta o segmento BC em F.
> Se AB=a e OD=b. Calcule EF.
>


Re: [obm-l] Torneio das Cidades 94

2013-04-28 Por tôpico Rogerio Ponce
Ola' pessoal,
achei conveniente explicar melhor a solucao...

Problema:
Existem 20 alunos em uma escola.
Quaisquer dois deles possui uma avo' em comum.
Prove que pelo menos 14 deles possui uma avo' em comum.

Solucao:
Seja Antonio um dos 20 alunos.
Sejam Maria e Nair suas avos.
Portanto, qualquer outro aluno tem uma avo' Maria, ou Nair, ou Maria e Nair
simultaneamente.

Assim, podemos encaixar cada aluno em um dos grupos A, B e C, definidos da
seguinte forma:
- grupo A: o aluno tem uma avo' Maria, e uma avo' Nair.
- grupo B: o aluno tem uma avo' Maria, e uma outra avo' que nao e' Nair.
- grupo C: o aluno tem uma avo' Nair, e uma outra avo' que nao e' Maria.

Dessa forma, Antonio pertence ao grupo A, e existem 4 possibilidades:
1) os grupos B e C sao vazios:
 neste caso, tanto Maria quanto Nair sao avos dos 20 alunos.

2) apenas o grupo C e' vazio:
 neste caso, Maria e' avo' dos 20 alunos.

3) apenas o grupo B e' vazio:
 neste caso, Nair e' avo' dos 20 alunos.

4) nenhum grupo e' vazio:
neste caso, seja Bernardo um aluno do grupo B, e seja Odete a sua outra
avo' ( aquela que nao e' Nair).
Acontece que qualquer que seja o aluno "c" do grupo C, "c" e Bernardo
possuem uma avo' em comum.
Como uma avo' de "c" e' Nair, a outra, aquela que nao pode ser Maria, tem
que ser Odete.
Portanto, todos os alunos do grupo C sao netos de Odete.

Agora, seja Camilo um aluno do grupo C.
Entao, conforme acabamos de provar, Camilo tem as avos Nair e Odete.
Acontece que qualquer que seja o aluno "b" do grupo B, "b" e Camilo possuem
uma avo' em comum.
Como uma avo' de "b" e' Maria, a outra, aquela que nao pode ser Nair, tem
que ser Odete.
Portanto todos os alunos do grupo B sao netos de Odete.

Desse modo, apenas 3 mulheres - Maria, Nair e Odete - sao as unicas avos de
todos os alunos.
Como os 20 alunos possuem ao todo 40 avos, uma das 3 mulheres tem que ser
avo' pelo menos 14 vezes, ou seja, tem que ser avo' de pelo menos 14 alunos
(principio das casas de pombos).

Assim, examinadas as possibilidades, sempre podemos afirmar que pelo menos
14 alunos possuem uma avo' em comum.

[]'s
Rogerio Ponce


Re: [obm-l] Torneio das Cidades 94

2013-04-15 Por tôpico Rogerio Ponce
Ola' pessoal,
considerando apenas as duas avós de cada aluno (o problema esta' mal
escrito), podemos dizer o seguinte:

Suponhamos que algum aluno tenha as avós Maria e Nair.

Entao, qualquer aluno se encontra em um dos grupos A, B ou C , tal que :

- grupo A: Maria e Nair sejam as avós do aluno.
- grupo B: apenas Maria seja uma das avós do aluno.
- grupo C: apenas Nair seja uma das avós do aluno.

Mas os alunos dos grupos B e C precisam ter alguma avo' em comum.
Portanto, existe uma avo' Odete, por exemplo, comum aos alunos dos grupos B
e C.

Dessa forma, Maria, Nair e Odete sao as unicas entre as 40 avós dos 20
alunos.

Portanto, alguma delas tem que ser avo' de, pelo menos, 14 alunos.

[]'s
Rogerio Ponce



2013/4/11 Bernardo Freitas Paulo da Costa 

> 2013/4/11 terence thirteen :
> > Me parece bastante claro. 20 pessoas tal que, sempre que pegar duas
> delas,
> > elas terão um avô em comum. Então existe um avô que tem 14 netos aí.
> Faça pacotes de 5 alunos, A, B, C, D. Os alunos A e B têm um avô em
> comum (o "primeiro"), C e D idem. Os alunos A e C têm um (segundo) avô
> em comum, B e D idem. Finalmente, os alunos A e D têm um avô (o
> terceiro) em comum, B e C idem. Assim, há ao todo 6 avôs, cada avô é
> avô de 5 + 5 pessoas, cada par tem um avô em comum (trivial dentro dos
> grupos, por construção fora). Portanto, talvez, sejam *apenas*
> consideradas as avós, e assim cada aluno tem 2 avós, por questões de
> sexo.
>
> Abaixo problemas mal-contextualizados!
> --
> Bernardo Freitas Paulo da Costa
>
> --
> Esta mensagem foi verificada pelo sistema de antivírus e
>  acredita-se estar livre de perigo.
>
>
> =
> Instruções para entrar na lista, sair da lista e usar a lista em
> http://www.mat.puc-rio.br/~obmlistas/obm-l.html
> =
>

-- 
Esta mensagem foi verificada pelo sistema de antivírus e
 acredita-se estar livre de perigo.



Re: [obm-l] Torneio das Cidades 94

2013-04-14 Por tôpico PONCE
 
 Amigos,
 Não puder  ler os últimos emails, devido a uma serie de trabalhos e
aulas que tenho diariamente.Ainda hoje faço um esboço da prova que
são 14.O grande amigo Benedito, poderia ter deixado uma solução,
pois este é um tipico problema que elegosta muito e costuma postar na
lista, geralmente com problemas de coloração.U m abraço a todos.Do
amigoPONCE
 On Qui 11/04/13 18:38 , "Vanderlei *" vanderma...@gmail.com sent:
 Se puder dar uma dica...
 Em 11 de abril de 2013 18:00, PONCE  escreveu:
 Jeferson,  
  O enunciado do problema sugere que voce pense no principio da casa
dos pombos. Com isto na cabeça a prova é relativamente simples.
Qualquer duvida, entre em contato que envio um  esboço de uma prova.
PONCE. 
 On Qua 10/04/13 20:05 , Jeferson Almir jefersonram...@gmail.com sent:
Existem 20 alunos em uma escola. Quaisquer dois deles possui um avó
em comum. Prove que pelo menos 14 deles possui um avó em comum.estou
tentando fazer por grafos ..  alguma ajuda ou sugestão??
 -- 
 Esta mensagem foi verificada pelo sistema de antivrus e 
 acredita-se estar livre de perigo. 
 -- 
 Esta mensagem foi verificada pelo sistema de antivírus e 
 acredita-se estar livre de perigo. 
 -- 
 Esta mensagem foi verificada pelo sistema de antivírus e  
  acredita-se estar livre de perigo. 

-- 
Esta mensagem foi verificada pelo sistema de antiv�rus e
 acredita-se estar livre de perigo.



Re: [obm-l] Torneio das Cidades 94

2013-04-14 Por tôpico PONCE
 
 2
 On Qui 11/04/13 18:49 , Bernardo Freitas Paulo da Costa
bernardo...@gmail.com sent:
 2013/4/10 Jeferson Almir :
 > Existem 20 alunos em uma escola. Quaisquer dois deles possui um
avó em
 > comum. Prove que pelo menos 14 deles possui um avó em comum.
 Só uma curiosidade... quantos avós um aluno tem? 2 ou 4?
 -- 
 Bernardo Freitas Paulo da Costa
 -- 
 Esta mensagem foi verificada pelo sistema de antivírus e
  acredita-se estar livre de perigo.
 =
 Instruções para entrar na lista, sair da lista e usar a lista em
 http://www.mat.puc-rio.br/~obmlistas/obm-l.html [2]
 =

-- 
Esta mensagem foi verificada pelo sistema de antiv�rus e
 acredita-se estar livre de perigo.



Re: [obm-l] Torneio das Cidades 94

2013-04-11 Por tôpico PONCE
 Jeferson,
 O enunciado do problema sugere que voce pense no principio da casa
dos pombos.Com isto na cabeça a prova é relativamente
simples.Qualquer duvida, entre em contato que envio um  esboço de uma
prova.PONCE.
 On Qua 10/04/13 20:05 , Jeferson Almir jefersonram...@gmail.com sent:
Existem 20 alunos em uma escola. Quaisquer dois deles possui um avó
em comum. Prove que pelo menos 14 deles possui um avó em comum. estou
tentando fazer por grafos ..  alguma ajuda ou sugestão??
 -- 
 Esta mensagem foi verificada pelo sistema de antivrus e  
  acredita-se estar livre de perigo. 

-- 
Esta mensagem foi verificada pelo sistema de antiv�rus e
 acredita-se estar livre de perigo.



[obm-l] Re: [obm-l] Questões legais - AVENTURA ????

2013-04-10 Por tôpico Rogerio Ponce
Ola' pessoal,

No problema1, observemos que:
- duas "casas" pares, quando combinadas, geram uma diferenca tambem par, e
a quantidade de numeros impares nao se altera.
- uma casa impar quando combinada com uma casa par, gera uma diferenca
impar, e a quantidade de impares nao se altera.
- duas casas impares, quando combinadas, geram uma diferenca par, e a
quantidade de impares diminui de 2.

Como existem 999 impares no quadro negro, e a quantidade de impares diminui
somente em "pulos" de 2, e' impossivel obtermos um numero par no final do
processo.


No problema2, existem 11 estradas.
Suponhamos que haja uma reta que corte todas as estradas.
Ao percorrermos todas as cidades, a partir da cidade 1 (e voltando para
ela), ao atravessarmos a tal reta, mudariamos de lado (em relacao a tal
reta) 11 vezes, ou seja, terminariamos o percurso do lado oposto ao lado da
partida, o que e' absurdo.
Logo nao existe a tal reta.

[]'s
Rogerio Ponce



2013/4/10 Robério Alves 

>
>  *PROBLEMA1*
> Os números naturais de 1 até 1998 são escritos em um imenso quadro negro.
> Em seguida, um aluno apaga dois quaisquer colocando no lugar sua diferença
> (não negativa). Depois de muitas operações, um único número ficará escrito
> no quadro. É possível que esse número seja zero?
> ** **
> ** **
> *PROBLEMA 2*
> Em uma ilha plana existem 11 cidades numeradas de 1 a 11. Estradas retas
> ligam 1 a 2, 2 a 3, 3 a 4, ..., 10 a 11 e 11 a 1. É possível que uma reta
> corte todas as estradas?
>
>
> AVENTURA 
>
>
>
>
>
>
> --
> Esta mensagem foi verificada pelo sistema de antivírus e
> acredita-se estar livre de perigo.
>

-- 
Esta mensagem foi verificada pelo sistema de antivírus e
 acredita-se estar livre de perigo.



[obm-l] Re: [obm-l] Conjunto com Contagem - DIFÍCIL

2013-03-01 Por tôpico Rogerio Ponce
Ola' Pedro,
deve haver alguma diferenca em relacao ao enunciado original.
Segundo esta formula, para n=5 existe uma quantidade nao inteira de escolhas.

[]'s
Rogerio Ponce


Em 27 de fevereiro de 2013 20:24, Pedro Júnior
 escreveu:
> Prezados, não consegui avançar na resolução do seguinte problema:
>
> Seja n pertencente ao conjunto dos números naturais. Quatro naturais
> diferentes a, b, c, d, são escolhidos do
> conjunto (1, 2 ... n) de tal modo que a + c = b + d. Mostre que o número de
> maneiras de fazermos tais escolhas
> é exatamente n(n - 2)(2n - 5)/24.
>
> --
>
> Pedro Jerônimo S. de O. Júnior
>
> Professor de Matemática
>
> Geo João Pessoa – PB

-- 
Esta mensagem foi verificada pelo sistema de antivírus e
 acredita-se estar livre de perigo.


=
Instruções para entrar na lista, sair da lista e usar a lista em
http://www.mat.puc-rio.br/~obmlistas/obm-l.html
=


Re: [obm-l] Quadradinhos

2013-02-10 Por tôpico Rogerio Ponce
Ola' Joao,
vamos escolher "a" de um total de "a+b+c" para pintarmos de azul, e em
seguida "b" de um total de "b+c" para pintarmos de vermelho. Os que
sobrarem serao pintados de verde.

Assim, o resultado e'

C(a+b+c,a) * C(b+c,b) , ou seja,

(a+b+c)! * (b+c)! / [ (b+c)! * a! * c! * b! ] =

(a+b+c)! / [ a! * b! * c! ]

[]'s
Rogerio Ponce


Em 10 de fevereiro de 2013 23:10, João Maldonado
 escreveu:
> Temos a+b+c quadradinhos
> "a" devem ser pintados da cor azul
> "b" devem ser pintados da cor vermelha
> "c" devem ser pintados da cor verde
>
> Quantas configurações distintas podemos ter?
>
> []'s
> João

=
Instruções para entrar na lista, sair da lista e usar a lista em
http://www.mat.puc-rio.br/~obmlistas/obm-l.html
=


[obm-l] Re: [obm-l] Re: [obm-l] Re: [obm-l] series para 9º ano

2013-01-23 Por tôpico Rogerio Ponce
Ola' Thelio,
se quiser usar a intuicao (que frequentemente leva a resultados
errados), entao basta observar que os diametros dos circulos C0, C1,
C2, etc, vao "caminhando" sobre a altura do triangulo, na direcao do
vertice.

Ao considerarmos todos eles (uma quantidade infinita), obtemos a
altura do triangulo.

[]'s
Rogerio Ponce



Em 22 de janeiro de 2013 14:23, Thelio Gama  escreveu:
> Prof. Rogério,
>
> Muito obrigado! Será que existe uma forma de resolver sem o conhecimento de
> progressões. Pergunto isso porque a prova era para alunos do 9º ano.
> Tentando resolver, caí numa progressão geométrica de razão 1/3, mas como
> entender quanto dá essa soma sem conhecer PG? Existe uma forma intuitiva de
> entender??
>
> Em 22 de janeiro de 2013 02:10, Rogerio Ponce  escreveu:
>>
>> Ola' Thelio,
>> trace a altura do triangulo, relativa ao vertice B.
>>
>> Agora trace retas paralelas ao lado AC, tangentes a C0 e C1, C1 e C2,
>> C2 e C3, etc...
>>
>> Observe que os triangulos formados sao homoteticos, com centro de
>> homotetia em B.
>>
>> Assim, os circulos C0, C1,C2... sao proporcionais aos seus diametros,
>> que por sua vez, formam uma progressao geometrica de razao menor que
>> 1.
>>
>> E a soma dos termos dessa progressao e'  justamente o comprimento da
>> altura do triangulo.
>>
>> Descontando-se o diametro de C0, e multiplicando-se por 2 (nao se
>> esqueca do vertice C), obtemos o resultado procurado.
>>
>> []'s
>> Rogerio Ponce
>>
>> ___
>>
>> Em 21/01/13, Thelio Gama escreveu:
>> > Prezados mestres,
>> >
>> > a questão em anexo foi retirada de uma prova de concurso técnico para
>> > alunos do 9º ano. Poderiam me explicar como resolver a mesma com
>> > conhecimentos do 9º ano?
>> >
>>
>> =
>> Instruções para entrar na lista, sair da lista e usar a lista em
>> http://www.mat.puc-rio.br/~obmlistas/obm-l.html
>> =
>
>

=
Instruções para entrar na lista, sair da lista e usar a lista em
http://www.mat.puc-rio.br/~obmlistas/obm-l.html
=


[obm-l] Re: [obm-l] series para 9º ano

2013-01-21 Por tôpico Rogerio Ponce
Ola' Thelio,
trace a altura do triangulo, relativa ao vertice B.

Agora trace retas paralelas ao lado AC, tangentes a C0 e C1, C1 e C2,
C2 e C3, etc...

Observe que os triangulos formados sao homoteticos, com centro de
homotetia em B.

Assim, os circulos C0, C1,C2... sao proporcionais aos seus diametros,
que por sua vez, formam uma progressao geometrica de razao menor que
1.

E a soma dos termos dessa progressao e'  justamente o comprimento da
altura do triangulo.

Descontando-se o diametro de C0, e multiplicando-se por 2 (nao se
esqueca do vertice C), obtemos o resultado procurado.

[]'s
Rogerio Ponce

___

Em 21/01/13, Thelio Gama escreveu:
> Prezados mestres,
>
> a questão em anexo foi retirada de uma prova de concurso técnico para
> alunos do 9º ano. Poderiam me explicar como resolver a mesma com
> conhecimentos do 9º ano?
>

=
Instruções para entrar na lista, sair da lista e usar a lista em
http://www.mat.puc-rio.br/~obmlistas/obm-l.html
=


Re: [obm-l] PROBLEMA

2012-12-07 Por tôpico Rogerio Ponce
Para fazer justica ao enunciado, leia-se "paralelas ao plano P" em vez de
"paralelas ao plano horizontal".

[]'s
Rogerio Ponce

Em 7 de dezembro de 2012 21:06, Rogerio Ponce  escreveu:

> Ola' Luis e Bernardo,
> a letra "d" (a funcao leva circunferencias em circunferencias) esta'
> errada porque, de modo geral, leva circunferencias (nao paralelas ao plano
> horizontal) em elipses.
> []'s
> Rogerio Ponce
>
> Em 5 de dezembro de 2012 20:52, Luís Lopes escreveu:
>
>>  Sauda,c~oes,
>>
>> Mais uma tentativa de ver esta mensagem chegar na lista.
>>
>> O Bernardo já respondeu numa troca de emails particular.
>>
>> Esta mensagem deve ser lida de baixo para cima.
>>
>> Abraços,
>> Luís
>>
>>
>> O arquivo está em
>> http://www.escolademestres.com/download/questao_luis_lopes_20121205.pdf
>>
>> > Date: Wed, 5 Dec 2012 09:59:41 -0500
>> > Subject: Re: [obm-l] RE: PROBLEMA
>> > From: bernardo...@gmail.com
>> > To: qed_te...@hotmail.com
>> >
>> > 2012/12/5 Luís Lopes :
>> > > Oi Bernardo,
>> > >
>> > > Não consigo me comunicar com a lista. Mas pelo jeito a
>> > > lista (você) recebe(u) minhas msgs.
>> > Estranho.
>> >
>> > > Tenho recebido somente algumas msgs da lista. Num outro
>> > > email (outra conta) acho que recebo todos. Pode ser problema
>> > > com o hotmail também, sei lá.
>> > Pode ser...
>> >
>> > > Tudo começou com o email abaixo, que mandei pra lista:
>> > Pode ser uma questão de PDFs. Sei lá. Pode ser mesmo "esse PDF em
>> particular".
>> >
>> > Para responder a pergunta: todas as afirmações são verdadeiras, é
>> > claro. O problema da "d" é que a projeção estereográfica envia
>> > círculos passando pelo polo Norte em retas do plano + ponto no
>> > infinito. Mas como f está definida apenas na esfera - polo Norte,
>> > nenhuma dessas circunferências serve, porque f não aplica o polo norte
>> > em lugar nenhum. Logo, as circunferências que você *pode* usar são
>> > realmente mandadas em círculos do plano.
>> >
>> > Ah, óbvio que isso é um problema de geometria complexa, mas fica muito
>> > mais legal retirar todo o contexto e fazer uma questão absurda de
>> > vestibular. Pena, né.
>> > --
>> > Bernardo Freitas Paulo da Costa
>>
>> --
>> From: qed_te...@hotmail.com
>> To: obm-l@mat.puc-rio.br
>> Subject: FW: PROBLEMA
>> Date: Mon, 3 Dec 2012 12:44:01 +
>>
>> Sauda,c~oes,
>>
>> Mais uma vez, peço a ajuda de vocês para uma resposta
>> mais completa e interessante.
>>
>> Espero que o anexo passe. Se não, alguém poderia colocá-lo
>> disponível em algum site?
>>
>> 
>> Subject: PROBLEMA
>> Date: Mon, 3 Dec 2012 08:17:24 -0300
>>
>> Prezado Luís,
>>
>> Gostaria de sua ajuda para o problema(conforme arquivo PDF anexo).
>> Se possível, gostaria que justificasse porque os demais itens são
>> verdadeiros.
>>
>> Abraços,
>>
>> FERNANDO FORTALEZA-CE.
>> 
>>
>> PS: E eu gostaria de saber também por que o item falso é falso.
>> Obrigado.
>>
>> Abraços,
>> Luís
>>
>>
>


Re: [obm-l] PROBLEMA

2012-12-07 Por tôpico Rogerio Ponce
Ola' Luis e Bernardo,
a letra "d" (a funcao leva circunferencias em circunferencias) esta' errada
porque, de modo geral, leva circunferencias (nao paralelas ao plano
horizontal) em elipses.
[]'s
Rogerio Ponce

Em 5 de dezembro de 2012 20:52, Luís Lopes  escreveu:

>  Sauda,c~oes,
>
> Mais uma tentativa de ver esta mensagem chegar na lista.
>
> O Bernardo já respondeu numa troca de emails particular.
>
> Esta mensagem deve ser lida de baixo para cima.
>
> Abraços,
> Luís
>
>
> O arquivo está em
> http://www.escolademestres.com/download/questao_luis_lopes_20121205.pdf
>
> > Date: Wed, 5 Dec 2012 09:59:41 -0500
> > Subject: Re: [obm-l] RE: PROBLEMA
> > From: bernardo...@gmail.com
> > To: qed_te...@hotmail.com
> >
> > 2012/12/5 Luís Lopes :
> > > Oi Bernardo,
> > >
> > > Não consigo me comunicar com a lista. Mas pelo jeito a
> > > lista (você) recebe(u) minhas msgs.
> > Estranho.
> >
> > > Tenho recebido somente algumas msgs da lista. Num outro
> > > email (outra conta) acho que recebo todos. Pode ser problema
> > > com o hotmail também, sei lá.
> > Pode ser...
> >
> > > Tudo começou com o email abaixo, que mandei pra lista:
> > Pode ser uma questão de PDFs. Sei lá. Pode ser mesmo "esse PDF em
> particular".
> >
> > Para responder a pergunta: todas as afirmações são verdadeiras, é
> > claro. O problema da "d" é que a projeção estereográfica envia
> > círculos passando pelo polo Norte em retas do plano + ponto no
> > infinito. Mas como f está definida apenas na esfera - polo Norte,
> > nenhuma dessas circunferências serve, porque f não aplica o polo norte
> > em lugar nenhum. Logo, as circunferências que você *pode* usar são
> > realmente mandadas em círculos do plano.
> >
> > Ah, óbvio que isso é um problema de geometria complexa, mas fica muito
> > mais legal retirar todo o contexto e fazer uma questão absurda de
> > vestibular. Pena, né.
> > --
> > Bernardo Freitas Paulo da Costa
>
> --
> From: qed_te...@hotmail.com
> To: obm-l@mat.puc-rio.br
> Subject: FW: PROBLEMA
> Date: Mon, 3 Dec 2012 12:44:01 +
>
> Sauda,c~oes,
>
> Mais uma vez, peço a ajuda de vocês para uma resposta
> mais completa e interessante.
>
> Espero que o anexo passe. Se não, alguém poderia colocá-lo
> disponível em algum site?
>
> 
> Subject: PROBLEMA
> Date: Mon, 3 Dec 2012 08:17:24 -0300
>
> Prezado Luís,
>
> Gostaria de sua ajuda para o problema(conforme arquivo PDF anexo).
> Se possível, gostaria que justificasse porque os demais itens são
> verdadeiros.
>
> Abraços,
>
> FERNANDO FORTALEZA-CE.
> 
>
> PS: E eu gostaria de saber também por que o item falso é falso.
> Obrigado.
>
> Abraços,
> Luís
>
>


[obm-l] Re: [obm-l] Preciso de uma Solução ensino médio

2012-10-13 Por tôpico Rogerio Ponce
(reenviando para a lista)

Ola' Douglas,
chamemos de
a: quantidade de tijolos com altura 10
b: quantidade de tijolos com altura 19
c: quantidade de tijolos com altura 4

Assim, a altura H da torre vale 10a+19b+4c
ou seja,
H = 4(a+b+c) + 3(2a+5b) = 4*94 + 3(2a+5b)

Dessa forma, basta calcularmos quantos valores diferentes existem para
 2a+5b
sabendo-se que 0 =< a+b =< 94

Para cada "b" par, obtemos as terminacoes 0 2 4 6 8 (fazendo "a" variar
entre 0 e 4)
Para cada "b" impar, obtemos as terminacoes 5 7 9 1 3 (fazendo "a" variar
entre 0 e 4).
Observe que o primeiro "b" impar e' 5, de modo que os resultados impares 1
e 3 nao podem ser obtidos.
Assim, variando de b=0 ate b=90, todos os inteiros (com excecao dos valores
1 e 3) podem ser obtidos.
Alem disso, para b entre 89 e 94, observamos o seguinte:

b=89, a=[0,1,2,3,4] ->  445+0, 445+2, 445+4, 445+6, 445+8 ->  445, 447,
449, 451, 453
b=90, a=[0,1,2,3,4] ->  450+0, 450+2, 450+4, 450+6, 450+8 ->  450, 452,
454, 456, 458
b=91, a=[0,1,2,3  ] ->  455+0, 455+2, 455+4, 455+6->  455, 457,
459, 461
b=92, a=[0,1,2] ->  460+0, 460+2, 460+4   ->  460, 462, 464
b=93, a=[0,1  ] ->  465+0, 465+2  ->  465, 467
b=94, a=[0] ->  470+0 ->  470

Ou seja, de 0 ate' o valor 462 (inclusive) todos os inteiros podem ser
obtidos (com excecao do 1 e do 3), o que nos da' 461 valores.
Alem desses, tambem existem o 464, 465, 467 e 470, perfazendo um total de
465 valores.

[]'s
Rogerio Ponce

Em 1 de outubro de 2012 23:04, escreveu:

> **
>
> Caros amigos , preciso de uma solução de ensino médio para a seguinte
> questao:
>
> 1) Noventa e quatro tijolos de medidas 4 x 10 x 19 serão empilhados um
> em cima do outro de modo a formar uma torre com 94 tijolos. Sabendo que
> cada tijolo pode ser disposto de modo a contribuir com 4, 10 ou 19 na
> altura total da torre, o número de torres de alturas distintas que podem
> ser obtidas  utilizando-se todos estes 94 tijolos é igual a :
>
>


[obm-l] Re: [obm-l] Re: [obm-l] Re: [obm-l] Re: [obm-l] Re: [obm-l] Quantos dígitos tem o fatorial de 7000?

2012-09-27 Por tôpico Rogerio Ponce
Nao precisa lamentar...
:)
Apenas faltou incluir os ultimos digitos diferentes de zero quando ao
considerarmos os fatores multiplos de 10 (dezenas, centenas e milhares).
O que "atrapalhou" a abordagem anterior foi a presenca dos numeros
terminados em zeros e cincos (que tambem acabam gerando zeros ao serem
multiplicados por fatores pares).
O problema e' que ao multiplicarmos por 10, o ultimo digito diferente de
zero no numero original nao se altera, mas isso ja' nao funciona ao
multiplicarmos por 30, por exemplo. Entao, vamos separar tudo aquilo que
pode gerar zeros de tudo o que nao contribui para os zeros, ou seja vou
"retirar" os numeros que acabam em 5 e em 0, e mais um fator 4 (para
agrupar com os multiplos de 5) em cada grupo de 10 numeros.

Esse fator "4" vou conseguir da seguinte forma:

Nos grupos impares (1 a 10, 21 a 30, 41 a 50, etc) vou dividir o segundo e
o quarto termos por 2, de modo que os algarismos correpondentes se
transformem de "2" em "1", e de "4" em "2". Assim, o grupo de 21 a 30, por
exemplo, se transforma em
 21 , 22/2=11 , 23 , 24/2=12 , 25 (sera' retirado), 26 , 27, 28 , 29 , 30
(sera' retirado)
gerando a sequencia
1 , 1 , 3 , 2 , 6 , 7 , 8 , 9
cujos termos, quando multiplicados, geram um produto com o ultimo digito
igual a 4.

Nos grupos pares (11 a 20, 31 a 40, 51 a 60, etc) vou dividir o segundo e o
quarto termos por 2, de modo que os algarismos correpondentes se
transformem de "2" em "6", e de "4" em "7". Assim, o grupo de 11 a 20, por
exemplo, se transforma em
11 , 12/2=6 , 13 , 14/2=7 , 15 (sera' retirado) , 16 , 17 , 18 , 19 , 20
(sera' retirado)
gerando a sequencia
1 , 6 , 3, 7 , 6 , 7 , 8 , 9
cujos termos, quando multiplicados, geram um produto com ultimo digito
igual a 4.

Assim, qualquer grupo de 10 numeros consecutivos (1 a 10, 11 a 20, etc...)
, quando usado dentro do fatorial (sem considerar os multiplos de 5, e
"separando" o tal fator 4) contribui multiplicando por 4 o ultimo digito
diferente de zero.

Bem, para simplificar a escrita, chamemos de UD{x} o ultimo digito
diferente de zero em x.
Observando (mais uma vez) cada grupo de 10 numeros consecutivos em 7000!
(de 1 a 10, de 11 a 20, etc), vemos que:
UD{ 7000! } =
UD{ [(1*1*3*2*6*7*8*9) ** 350] * [(1*6*3*7*6*7*8*9) ** 350 ] * [(2*2)**700]
* [5*10*15*20*...*7000] } =
UD{ [4**700] * [4**700] * [5*10*15*20*...*7000] } =
UD{ [4**700] * [4**700] * [5**1400] * [ 1*2*...*1400] } =
UD{ [4**700] * [10**1400] * 1400! } =
UD{ [4**700] * 1400! }
Aplicando o mesmo raciocinio para 1400! , e sucessivamente, o resultado
procurado e'
UD{ [4**700] * [4**140] * 280! } =
UD{ [4**700] * [4**140] * [4**28] * 56! }  =
UD{ [4**700] * [4**140] * [4**28] * 50! * (1*2*3*4*55*6) }  =
UD{ [4**700] * [4**140] * [4**28] * [4**5] * 10! * (1*2*3*4*55*6) }  =
UD{ [4**700] * [4**140] * [4**28] * [4**5] * [4**1] * 2! * (1*2*3*4*55*6) }
Como UD{ 2! * (1*2*3*4*55*6) } = 4, a expressao acima se transforma em
UD{ [4**(700+140+28+5+1)] * 4 } =
UD{ [4**875]} = 4,
pois as potencias de 4 se repetem em um ciclo de 2, isto e'
UD{ 4**impar } = 4
UD{ 4**par } = 6

Assim, o ultimo digito diferente de zero em 7000! e' 4.
[]'s
Rogerio Ponce


Em 26 de setembro de 2012 11:14, terence thirteen
escreveu:

> Sinto informar mas o pari-gp afirma que este último dígito é 4.
>
> Em 23 de setembro de 2012 22:38, Rogerio Ponce 
> escreveu:
> > Ola' pessoal,
> > respondendo ao Terence: qual o ultimo digito de 7000! , diferente de
> zero?
> >
> > Bem, 8 e' o ultimo digito diferente de zero em fatorial de 10.
> >
> > Alem disso, sabemos que
> > 8**1 termina em 8
> > 8**2 termina em 4
> > 8**3 termina em 2
> > 8**4 termina em 6
> > 8**5 termina em 8 novamente, estabelecendo um ciclo de 4 potencias ate'
> que
> > o ultimo digito se repita novamente.
> >
> > Portanto, ao calcularmos o fatorial de 7000, partindo de 1, o que
> acontece
> > e' que a cada 10 numeros (de 1 a 10, de 11 a 20, etc) o ultimo digito
> > diferente de zero (no resultado) e' multiplicado por 8.
> >
> > Assim, depois de 7000/10 = 700 dezenas, o ultimo algarismo diferente de
> zero
> > vale o mesmo que o ultimo algarismo de 8**700.
> > Logo, vale 6.
> >
> > []'s
> > Rogerio Ponce
> >
> >
> >
> > Em 22 de setembro de 2012 13:03, terence thirteen <
> peterdirich...@gmail.com>
> > escreveu:
> >>
> >> Quantos dígitos? Isso é a parte inteira de log(7000!)/log 10. Usando
> >> alguma aproximação acho que dá.
> >>
> >> mais divertido é saber qual o último dígito diferente de zero...
> >>
> >>
> >> Em 13 de setembro de 2012 18:37,
> >>

[obm-l] Re: [obm-l] Re: [obm-l] Re: [obm-l] Quantos dígitos tem o fatorial de 7000?

2012-09-23 Por tôpico Rogerio Ponce
Ola' pessoal,
respondendo ao Terence: qual o ultimo digito de 7000! , diferente de zero?

Bem, 8 e' o ultimo digito diferente de zero em fatorial de 10.

Alem disso, sabemos que
8**1 termina em 8
8**2 termina em 4
8**3 termina em 2
8**4 termina em 6
8**5 termina em 8 novamente, estabelecendo um ciclo de 4 potencias ate' que
o ultimo digito se repita novamente.

Portanto, ao calcularmos o fatorial de 7000, partindo de 1, o que acontece
e' que a cada 10 numeros (de 1 a 10, de 11 a 20, etc) o ultimo digito
diferente de zero (no resultado) e' multiplicado por 8.

Assim, depois de 7000/10 = 700 dezenas, o ultimo algarismo diferente de
zero vale o mesmo que o ultimo algarismo de 8**700.
Logo, vale 6.

[]'s
Rogerio Ponce


Em 22 de setembro de 2012 13:03, terence thirteen
escreveu:

> Quantos dígitos? Isso é a parte inteira de log(7000!)/log 10. Usando
> alguma aproximação acho que dá.
>
> mais divertido é saber qual o último dígito diferente de zero...
>
>
> Em 13 de setembro de 2012 18:37,
>  escreveu:
> >
> >
> > Ops , verdade, bom sendo assim use a aproximacao de um fatorial pela
> fórmula
> > de stirling ok
> >
> > On Thu, 13 Sep 2012 09:55:57 -0400, Bernardo Freitas Paulo da Costa
> wrote:
> >
> > 2012/9/13 ennius :
> >
> > Prezados Colegas, Qual o melhor método para calcular quantos dígitos
> tem o
> > fatorial de 7000 (ou de qualquer outro número natural grande)?
> >
> > Calcule o logaritmo em base 10.
> >
> > Vai dar uma soma bem grande. A única coisa que falta é aproximar a
> > soma por uma integral, calculando o erro da aproximação.
> >
> >
> >
> >
>
>
>
> --
> /**/
> 神が祝福
>
> Torres
>
> =
> Instru�ões para entrar na lista, sair da lista e usar a lista em
> http://www.mat.puc-rio.br/~obmlistas/obm-l.html
> =
>


Re: [obm-l] probabilidade

2012-09-21 Por tôpico Rogerio Ponce
Ola' Bernardo,
nada como "colocar mais lenha na fogueira" de uma forma saudavel...
E cade o Ralph???
:)

Bem, nao resisto a acrescentar que exatamente a NASA jogou fora, em
1999, 4 anos de trabalho e 650 milhoes de dolares por nao especificar
adequadamente...
...as unidades de medida ( !!! ) a serem usadas na comunicacao entre
os programas de controle da sonda "Mars Climate Orbiter".

No caso da sonda, alguma coisa fundamental precisava, e podia ser
perguntada. Mas nao foi. Deu no que deu.

Em outras situacoes, voce vai ter que fazer nao uma aproximacao, mas
uma suposicao sobre os dados de que voce dispoe. Ou sobre a falta
deles. Sem direito a perguntas.
( todo homem casado tem varios exemplos sobre isto... )

Bem, numa prova discursiva, certamente voce poderia abordar todos os
pontos de vista.
Mas suponha que a prova seja de multipla escolha. Voce tera' que
considerar que, talvez (e muito provavelmente) o examinador
"pretendeu" dizer que a "marcacao ao acaso" era "uniformemente
distribuida".
E' uma "especificacao" deficiente, mas seria a interpretacao que eu
daria para questao original.

E, apos a prova, o examinador deveria ser abordado, principalmente
para que ele pensasse um pouco mais sobre o que anda propondo aos
alunos...

Grande abraco,
Rogerio Ponce


Em 20/09/12, Bernardo Freitas Paulo da Costa escreveu:
> 2012/9/20 Rogerio Ponce 
>> Ola' pessoal,
> Oi Ponce, e demais colegas da lista!
>
>> acho otimo que se discuta o "como deveria ser", pois serve para melhorar
>> a
>> visao critica e a capacidade de comunicacao de todos.
> Certíssimo.
>
>> Mas, paralelamente, cito uma velha maxima, valida em todos os exames e,
>> muito frequentemente, em situacoes do dia a dia:
>>  "A interpretacao faz parte do problema."
> Eu acho muito diferente você pedir uma interpretação de um problema
> concreto, numa situação real, onde você vai TER QUE fazer alguma
> aproximação (por exemplo, ignorar o atrito com o ar), e uma
> interpretação errada de um conceito matemático. Dizer que os alunos
> escolheram uma resposta ao acaso tem um sentido matemático bem
> definido (se você interpretar isso como probabilidade uniforme, o que
> faz parte sim do problema - bom, talvez o Ralph diga que seria melhor
> estar escrito também, e eu talvez concorde com ele, mas se no
> enunicado estiver "chutaram" talvez volte a fazer parte de uma
> interpretação razoável) e que é diferente, insisto, diferente, do
> sentido de "cada resposta foi marcada o mesmo número de vezes pelos
> alunos que chutaram". Talvez o jeito certo de tratar esse problema
> seja justamente dizer:
>
> "Bom, o que está no enunciado é uma coisa. E é um problema bem
> difícil. Muito difícil mesmo. Vejam só (passar um tempo explicando que
> vai ter que fazer uma disjunção de casos, que para piorar não são
> equiprováveis, etc, etc, etc). Mas a gente pode ter uma idéia razoável
> de quanto vai ser se a gente supuser que de tudo o que pode acontecer,
> o evento "médio" deve dar um resultado suficientemente próximo do
> resultado real. E daí fica bem mais fácil, né?"
>
> Aliás, eu concordaria ainda mais com você sobre interpretação se os
> exames também fossem mais inteligentes do que o que temos visto por
> aí. Com questões discursivas (e apenas discursivas) onde o aluno pode
> pegar um problema e tratar do início ao fim, formulando hipóteses,
> escolhendo um modelo, ajustando as constantes para que faça parte do
> sentido real, e daí você pode se permitir:
> - deixar o aluno interpretar e aceitar diferentes raciocínios
> - ignorar os erros de contas (mas não a preguiça de verificar que
> algumas etapas fazem sentido físico).
>
> Mas uma prova dessa leva mais tempo para elaborar, pro aluno fazer, e
> principalmente pro professor corrigir. Mas com uma visão destas,
> talvez finalmente os alunos achem que matemática é mais do que um
> bando de regrinhas estranhas para fazer contas e brincar com
> triângulos. (nada contra essas duas, muito menos o aspecto lúdico, mas
> sejamos honestos, compreender o que o Galileu disse sobre "a natureza
> está escrita na linguagem da matemática", eu acho muito mais
> estimulante)
>
>> Se tudo o que voce fizer sempre depender de uma especificacao
>> absolutamente completa e consistente, ninguem (seus pais, esposa, filhos,
>> amigos, vizinhos, chefes, empregados, etc) vai lhe pedir coisa alguma...
> Talvez. Mas veja bem que isso é uma questão de mentalidade. O pessoal
> de programação da NASA (eu vi uma reportagem há pouco tempo) faz
> exatamente isso. Especificações completas do que cada parte tem que
> fazer. Tratando de tdas as possibilidades. Porque veja bem,
> quando u

Re: [obm-l] Conjectura "abc"

2012-09-15 Por tôpico Rogerio Ponce
Ola' pessoal,
foi anunciada a prova da conjectura "abc".
Mais em:

http://www.scientificamerican.com/article.cfm?id=proof-claimed-for-deep-connection-between-prime-numbers

[]'s
Rogerio Ponce


Re: [obm-l] Ajuda em Polinomios

2012-09-13 Por tôpico Rogerio Ponce
E' verdade! Otimo contra-exemplo!
:)

[]'s
Rogerio Ponce

Em 12 de setembro de 2012 15:26, Bernardo Freitas Paulo da Costa <
bernardo...@gmail.com> escreveu:

> 2012/9/12 Rogerio Ponce :
> > Humm... eu justificaria da seguinte forma:
> >
> > Se o polinomio "resto da divisao de P(x)/Q(x)" assume o valor zero para
> > infinitos valores de x, ou ele possui uma quantidade infinita de raizes
> ou
> > ele e' identicamente igual a zero.
> > Como ele nao pode ter uma quantidade infinita de raizes, entao ele e'
> nulo.
> > Portanto Q(x) divide P(x).
> >
> > Isso seria suficiente?
> >
> > []'s
> > Rogerio Ponce
> Acho que não, porque P(x)/Q(x) ser inteiro não implica que o resto é
> zero. Veja que (x^2 + 1)/(x + 1) tem resto 2 (e quociente = x - 1),
> mas em x=1 temos que P(x)=2=Q(x). Acho que tem que ter algum argumento
> de limpeza como fez o Ralph.
>
> >> 2012/9/12 Heitor Bueno Ponchio Xavier 
> >>>
> >>> Não consigo fazer a seguinte questão:
> >>> Mostre que se P(x) e Q(x) são polinômios de coeficientes inteiros tais
> >>> que P(x)/Q(x) é inteiro para infinitos valores inteiros de x então Q(x)
> >>> divide P(x).
> >>
> >>
> >
>
>
>
> --
> Bernardo Freitas Paulo da Costa
>
> =
> Instruções para entrar na lista, sair da lista e usar a lista em
> http://www.mat.puc-rio.br/~obmlistas/obm-l.html
> =
>


Re: [obm-l] Ajuda em Polinomios

2012-09-12 Por tôpico Rogerio Ponce
Humm... eu justificaria da seguinte forma:

Se o polinomio "resto da divisao de P(x)/Q(x)" assume o valor zero para
infinitos valores de x, ou ele possui uma quantidade infinita de raizes ou
ele e' identicamente igual a zero.
Como ele nao pode ter uma quantidade infinita de raizes, entao ele e' nulo.
Portanto Q(x) divide P(x).

Isso seria suficiente?

[]'s
Rogerio Ponce


Em 12 de setembro de 2012 13:31, Ralph Teixeira escreveu:

> Vou fazer usando uns canhoes:
>
> Lema: se R(x) eh um polinomio (nao nulo) com grau menor que Q(x), entao
> R(x)/Q(x) nao pode ser inteiro para infinitos valores de x.
>
> Prova:como lim(|x|->+Inf) R(x)/Q(x)=0, existe um certo N0 a partir do qual
> |R(x)/Q(x)| < 1 (isto eh, se |x|>N0 teriamos |R(x)/Q(x)|<1). Mas naquela
> lista de infinitos valores de x haveria infinitos com modulo maior que N0,
> entao R(x)=0 para todos eles, o que eh absurdo (o numero de raizes de R(x)
> eh finito).
>
> ---///---
>
> Agora, ao problema original: dividindo P(x) por Q(x), fica
> P(x)=Q(x)H(x)+R(x), isto eh, P(x)/Q(x)=H(x)+R(x)/Q(x) onde o grau de R(x)
> eh menor do que o de Q(x) e os coeficientes de H(x) sao racionais. Pegue o
> mmc de todos os denominadores dos coeficientes de H, digamos, M, e
> multiplique a coisa toda por M.
>
> MP(x)/Q(x)=MH(x)+MR(x)/Q(x)
>
> Note que, se x eh inteiro, entao MH tambem eh (pois os coeficientes de MH
> agora sao inteiros). Assim, se houvesse infinitos valores inteiros de x que
> fizessem P/Q ser inteiro, teriamos MR/Q inteiro tambem. Como o grau de MR
> eh menor que o grau de Q, usando o lema, temos que R eh identicamente nulo.
>
> Abraco,
>   Ralph
> 2012/9/12 Heitor Bueno Ponchio Xavier 
>
>> Não consigo fazer a seguinte questão:
>> Mostre que se P(x) e Q(x) são polinômios de coeficientes inteiros tais
>> que P(x)/Q(x) é inteiro para infinitos valores inteiros de x então Q(x)
>> divide P(x).
>>
>
>


[obm-l] Re: [obm-l] RE: [obm-l] Re: [obm-l] Socorro em geometria (construçã o)

2012-09-11 Por tôpico Rogerio Ponce
Ola' Luis,
com apenas 3 elementos conhecidos ( a diferenca das bases, e as diagonais )
o terceiro problema esta' indeterminado.
[]'s
Rogerio Ponce



Em 8 de setembro de 2012 12:45, Luís Lopes  escreveu:

>  Sauda,c~oes,
>
> > Por T passe uma paralela a AT. Com centro em A desenhe um arco de raio
> a. Esse
> Typo. Por C 
>
> Ou seja, TC é o transformado de BD pela translação do vetor DC.
>
> Outras construções: construir um trapézio ABCD, conhecendo-se as bases
> AB=a, CD=c, e
>
> 1) os dois lados, BC=b e AD=d.
>
> 2) as diagonais AC=p e BD=q.
>
> 3) a diferença das bases AB - CD = d, as diagonais AC = p e BD = q.
>
> 1) e 2) são problemas do tipo: dado o comprimento e a direção de um
> segmento,
> apoiá-lo em duas linhas (curvas).
>
> Apesar de parecido com o que foi acabado de ser construído, não consegui
> resolver o 3). Ou seja, não descobri a translação conveniente.
>
> Considero a disposição dos vértices como em
>
> http://en.wikipedia.org/wiki/Trapezoid
>
> Luís
>
>
> > From: saldana...@pucp.edu.pe
> > To: obm-l@mat.puc-rio.br
> > CC:
> > Subject: [obm-l] Re: [obm-l] Socorro em geometria (construçã o)
> > Date: Sat, 8 Sep 2012 06:51:19 -0500
> >
> >
> >
> > Contrói o triângulo ACT com lados AC=p, CT=q e AT=s.
> >
> > Por T passe uma paralela a AT. Com centro em A desenhe um arco de raio
> a. Esse
> > arco vai cortar à paralela (tem 2 soluções). Chame de D a ese ponto de
> corte.
> > Por D traçe uma paralea a CT, o ponto de corte entre essa paralea e AT
> será o
> > ponto B do trapecio ABCD procurado.
> >
> > Julio Saldaña
> >
> >
> > -- Mensaje original ---
> > De : obm-l@mat.puc-rio.br
> > Para : obm-l@mat.puc-rio.br
> > Fecha : Sat, 8 Sep 2012 03:09:59 +
> > Asunto : [obm-l] Socorro em geometria (construçã o)
> > >
> > >
> > >
> > >
> > >Construir o trapézio ABCD conhecendo a soma das bases AB +CD = s,as
> diagonais
> > AC = p e BD = q e o lado AD = a.Justifique.
> >
> > __
> > Si desea recibir, semanalmente, el Boletín Electrónico de la PUCP,
> ingrese a:
> > http://www.pucp.edu.pe/puntoedu/suscribete/
> >
> > =
> > Instruções para entrar na lista, sair da lista e usar a lista em
> > http://www.mat.puc-rio.br/~obmlistas/obm-l.html
> > =
>


Re: [obm-l] Ajuda

2012-07-16 Por tôpico Rogerio Ponce
Ola' Marcelo,
ambos sao parecidos, mas o geogebra e' gratuito, e o sketchpad e' pago.
Alem disso, o sketchpad funciona somente em Windows ou Macs, enquanto o
geogebra roda em Windows, Mac, e Linux.

Como eu so' uso Linux, nao tenho muito mais a acrescentar...
[]'s
Rogerio Ponce


Em 14 de julho de 2012 07:28, Marcelo de Moura Costa
escreveu:

> Alguém sabe dizer algo sobre o software geometer's sketchpad? É gratuito,
> compatível com linux ou windows? Melhor que geogebra? Vantagens e
> desvantagens em relação a outros de geometria dinâmica?
>


[obm-l] Re: [obm-l] Re: [obm-l] Condição necessária e suficiente

2012-07-01 Por tôpico Rogerio Ponce
Ola' pessoal,
me parece intuitivo o seguinte raciocinio:

Dizer que q e' condicao necessaria para p, significa dizer que para que p
ocorra entao q tem que ocorrer. Portanto p e' condicao suficiente para q.

Por outro lado, dizer que q e' condicao suficiente para p, significa dizer
que se q ocorre, entao p tem que ocorrer. Portanto, p e' condicao
necessaria para q.

Assim, p e' condicao necessaria e suficiente para q.

[]'s
Rogerio Ponce


Em 1 de julho de 2012 01:17, Ralph Teixeira  escreveu:

> Eu penso assim: as seguintes frases sao equivalentes entre si:
>
> "Se p, entao q"
> "q, se p"
> "p, somente se q"
> "p implica q"
> "p eh suficiente para q"
> "q eh necessario para p"
>
> Entao, eh claro, tambem sao equivalentes as seguintes frases:
> "Se q, entao p"
> "p, se q"
> "q, somente se p"
> "q implica p"
> "q eh suficiente para p"
> "p eh necessario para q"
>
> Entao, para mim, eh dificil fazer a distincao que voce quer (de qual eh a
> afirmacao imediata, e qual vem depois). Para mim, sao imediatemente
> equivalentes:
> "p, se e somente se q"
> "q, se e somente se p"
> "p eh necessario e suficiente para q"
> "q eh necessario e suficiente para p"
> "p eh equivalente a q"
>
> Abraco, Ralph.
>
>
>
>
>
> 2012/6/30 Paulo Argolo 
>
>> Caros Colegas,
>>
>> Sendo p e q proposições, parece-me que a proposição "p se, e somente se,
>> q" afirma que q é condição necessária e suficiente para p. Ela não afirma,
>> pelo menos de imediato, que p é condição necessária e suficiente para q.
>> Gostaria de saber o que os colegas pensam a respeito.
>> Abraços do Paulo!
>> =
>> Instru�ões para entrar na lista, sair da lista e usar a lista em
>> http://www.mat.puc-rio.br/~obmlistas/obm-l.html
>> =
>>
>
>


[obm-l] Re: [obm-l] Congruência

2012-06-29 Por tôpico Rogerio Ponce
Ola' Thiago,

Vou representar o resto da divisao de N por D como N%D.

Tambem estou considerando que o operador % (resto da divisao) tem
precedencia menor que ** (exponenciacao).

Ou seja, queremos o valor de

  [ 41**41+36**36] % 77

= [ 41**41%77 +36**36%77 ] % 77

= [ 41**41%77 + (-41)**36%77 ] % 77

= [ 41**41%77 +41**36%77 ] % 77

= [ 41**41+41**36] % 77

= [(41**5 + 1) *   41**36] % 77


Ora, o fator (41**5 + 1) pode ser reescrito como ((42-1)**5 + 1), e a
expansao de (42-1)**5 tem quase todos os termos multiplos de potencias de
42, com excecao do ultimo, que vale -1**5 = -1.
Portanto, ((42-1)**5 + 1) e' multiplo de 42, que e' multiplo de 7.

Por outro lado, o mesmo fator (41**5 + 1) pode ser reescrito como
((44-3)**5 + 1), e a expansao de (44-3)**5 tem quase todos os termos
multiplos de potencias de 44 (que e' multiplo de 11), com excecao do
ultimo, que vale -3**5 = -243.
Como -243 + 1 = -242, que tambem e' multiplo de 11, entao ((44-3)**5 + 1)
e' multiplo de 11.

Assim, o fator (41**5 + 1) e' multiplo de 7 e de 11, de modo que a
expressao original e' multipla de 77.

Logo o resto vale zero.

[]'s
Rogerio Ponce


Em 25 de junho de 2012 18:43, Thiago Bersch escreveu:

>  Qual o resto da divisão de 36^36+41^41 por 77 ?
>


[obm-l] Re: [obm-l] Problema difícil

2012-06-14 Por tôpico Rogerio Ponce
Ola',
observe que a resposta correta esta' em

   
http://www.mail-archive.com/obm-l@mat.puc-rio.br/msg24758.html<http://www.mail-archive.com/obm-l@mat.puc-rio.br/msg24758.html>

[]'s
Rogerio Ponce

Em 14 de junho de 2012 13:20, Vanderlei *  escreveu:

> Numa rua, existem 100 casas em fila, numeradas de 1 até 100. Um pintor vem
> e pinta todas as casas de vermelho. Em seguida, vem um segundo pintor e
> pinta de azul as casas de três em três, começando da casa número 3. A
> seguir, vem um terceiro pintor e pinta de vermelho as casas de cinco em
> cinco, começando na casa de número 5 (ele pinta de vermelho, mesmo que a
> casa já seja vermelha). Em seguida, vem um quarto pintor e pinta de azul as
> casas de sete em sete, começando na casa 7. A seguir, vem um quinto pintor,
> e assim por diante, alternando a pintura vermelha, azul, até o pintor de
> número 50. No final, quantas casas são vermelhas?
>


[obm-l] Re: [obm-l] Problema difícil

2012-06-14 Por tôpico Rogerio Ponce
Olá,
esse problema já foi resolvido aqui na lista.
Veja em:
   http://www.mail-archive.com/obm-l@mat.puc-rio.br/msg24658.html


[]'s
Rogerio Ponce

Em 14 de junho de 2012 13:20, Vanderlei *  escreveu:

> Numa rua, existem 100 casas em fila, numeradas de 1 até 100. Um pintor vem
> e pinta todas as casas de vermelho. Em seguida, vem um segundo pintor e
> pinta de azul as casas de três em três, começando da casa número 3. A
> seguir, vem um terceiro pintor e pinta de vermelho as casas de cinco em
> cinco, começando na casa de número 5 (ele pinta de vermelho, mesmo que a
> casa já seja vermelha). Em seguida, vem um quarto pintor e pinta de azul as
> casas de sete em sete, começando na casa 7. A seguir, vem um quinto pintor,
> e assim por diante, alternando a pintura vermelha, azul, até o pintor de
> número 50. No final, quantas casas são vermelhas?
>


Re: [obm-l] (ESFAO) PROBABILIDADE

2012-06-03 Por tôpico Rogerio Ponce
Ola' Arkon,
para que log_(2) N seja inteiro, N precisa ser potencia de 2.
As potencias de 2 com 3 algarismos distintos sao 128, 256 e 512, ou seja,
ha' 3.
Falta calcular quantos inteiros de 3 algarismos distintos existem:
Para o primeiro algarismo, temos 9 escolhas.
Para o segundo, temos 9 escolhas (pois agora o zero pode ser escolhido).
Para o terceiro temos 8 escolhas.
Logo, existem 9*9*8=648 inteiros com 3 algarismos distintos.
Assim, a probabilidade vale 3/648 = 1/216.
A resposta correta e' a letra C.

Seria possivel ser de outra forma?
Bem, so' se considerarmos numeros comecados por zero, embora eu discorde
desse caminho.
Vamos ver:

Potencias de 2: 016,032,064,128,256,512, ou seja ha' 6 potencias de 2.
Quantidade de numeros de 3 algarismos distintos, podendo comecar por zero:
10*9*8 = 720
E a probabilidade seria 6/720 = 1/120, que nao e' igual a nenhuma das
respostas oferecidas.

Portanto, fico com a resposta original 1/216, ou seja, letra C.

[]'s
Rogerio Ponce


Em 2 de junho de 2012 19:34, arkon  escreveu:

> *Alguem pode resolver???*
> *
> *
> *Um numero positivo "N" de 3 algarismos distintos, escrito na base
> decimal, e' escolhido ao acaso. A probabilidade de log_{2} N ser inteiro e':
> *
> *
> *
> *A) **1/450. *
> *B) 1/300. *
> *C) 1/216. *
> *D) 1/180. *
> *E) 1/162.*
> *
> *
> *Gab.: D*
>
> **
>
>
>


Re: [obm-l] Problema Legal

2012-05-21 Por tôpico Rogerio Ponce
Ola' Mauricio,
fazendo a leitura sem interpretacao, ate' poderia ser.
Inclusive, poderia ser dito que nao existem dragoes, e que portanto o
cavalheiro nem estaria preso.
:)
Mas o que realmente se deseja saber e' se existe algum metodo que garanta a
liberdade nos tempo proposto.

[]'s
Rogerio Ponce

Em 20 de maio de 2012 13:30, Mauricio barbosa  escreveu:

> Não pode ser que ocavalheiro , por sorte, separe as 100 moedas em duas
> pilhas de 50, de forma que as 50 mágicas estariam numa pilha e as 50 não
> mágicas  na outra, saindo assim em um dia?
> Em 17/05/2012 18:45, "Benedito Tadeu V. Freire" 
> escreveu:
>
>>
>> O problema abaixo apareceu na Lista de Problemas do pessoal da Argentina.
>>
>> Problema
>> Um dragão dá 100 moedas a um cavalheiro que ele mantém prisioneiro. A
>> metade das moedas são mágicas, mas somente o dragão sabe quais são elas.
>> Cada dia, o cavalheiro tem que dividir as 100 moedas em duas pilhas, não
>> necessariamente do mesmo tamanho.
>> Se algum dia as duas pilhas possuem o mesmo número de moedas mágicas ou
>> as pilhas tem o mesmo número de moedas não mágicas, o cavalheiro ganha a
>> liberdade.
>> Determinar se o cavalheiro pode ganhar sua liberdade em 50 dias ou menos.
>> E em 25 dias ou menos?
>>
>>
>> Benedito
>> --
>> Open WebMail Project (http://openwebmail.org)
>>
>>


Re: [obm-l] Problema Legal

2012-05-20 Por tôpico Rogerio Ponce
Contra-exemplo:
Imagine que a torre A tenha as 50 moedas nao-magicas em sua base, e as 50
moedas magicas no topo.
Em todas as movimentações havera' sempre um numero par de moedas magicas e
um numero par de moedas nao-magicas na pilha B.
Portanto o equilibrio nao acontecera'  em hora alguma.

[]'s
Rogerio Ponce


Em 20 de maio de 2012 07:08, Fernando Candeias escreveu:

> Outra opção.
>
> Moeda mágica=M
>
> Moeda não mágica = N
>
> A pilha original de 100 moedas pode ser concebida como uma superposição de
> 25 blocos de 4 moedas.
>
> Na primeira divisão fazer uma pilha  A com 96 moedas e uma B com 4
> moedas. Havendo um desequilíbrio de uma M ou N, continuará preso.
>
> Repetindo esse procedimento cada dia, agregando à pilha original mais 4
> moedas tiradas da torre maior, obteremos os pares  A=92,B=8; A=88, B=12
> etc. Normalmente os desvios se compensam, e o prisioneiro sairia bem antes
> de atingir o bloco 25, portanto antes do vigésimo quinto dia. Mas mesmo que
> o dragão fosse malicioso e deixasse o desequilíbrio se acumular  numa mesma
> espécie de moeda  até o bloco 24, ele teria de fazer no ultimo bloco a
> compensação necessária, caso contrário a torre não teria começado
> equilibrada.
>
> Abs
> Fernando Candeias
> Em 17 de maio de 2012 22:04, Rogerio Ponce  escreveu:
>
>> Para o cavalheiro ganhar a liberdade em ate' 25 dias:
>>
>> Ele separa as 100 moedas em 2 pilhas (A e B) de 50 moedas.
>>
>> A cada dia ele passa uma moeda da pilha A para a pilha B.
>>
>> E ao fim de 25 dias, na pilha A havera' apenas 25 moedas, e ela tera'
>> passado por alguma situacao de igualdade entre as suas moedas magicas (ou
>> nao-magicas), e as moedas magicas (ou nao-magicas) da pilha B.
>>
>> Vejamos como funciona:
>>
>> 1) Se na pilha A houver 25 moedas magicas, entao o cavalheiro ganha a
>> liberdade imediatamente (pois tambem havera'  25 moedas magicas na pilha B).
>>
>> 2) Se na pilha A houver mais de 25 moedas magicas, entao, em algum dos 25
>> dias subsequentes, esse numero tera' sido reduzido para no maximo 25 moedas
>> magicas. Portanto, em algum momento acontecera' a igualdade entre as moedas
>> magicas das duas pilhas.
>>
>> 3) Se na pilha A houver menos que 25 moedas magicas, entao havera' mais
>> que 25 moedas nao-magicas na pilha A. Portanto, em algum dos 25 dias dias
>> subsequentes, acontecera' uma situacao de igualdade entre as moedas
>> nao-magicas das 2 pilhas.
>>
>> []'s
>> Rogerio Ponce
>>
>>
>>
>>
>> Em 17 de maio de 2012 15:42, Benedito Tadeu V. Freire 
>> escreveu:
>>
>>
>>> O problema abaixo apareceu na Lista de Problemas do pessoal da
>>> Argentina.
>>>
>>> Problema
>>> Um dragão dá 100 moedas a um cavalheiro que ele mantém prisioneiro. A
>>> metade das moedas são mágicas, mas somente o dragão sabe quais são elas.
>>> Cada dia, o cavalheiro tem que dividir as 100 moedas em duas pilhas, não
>>> necessariamente do mesmo tamanho.
>>> Se algum dia as duas pilhas possuem o mesmo número de moedas mágicas ou
>>> as pilhas tem o mesmo número de moedas não mágicas, o cavalheiro ganha a
>>> liberdade.
>>> Determinar se o cavalheiro pode ganhar sua liberdade em 50 dias ou
>>> menos.
>>> E em 25 dias ou menos?
>>>
>>>
>>> Benedito
>>> --
>>> Open WebMail Project (http://openwebmail.org)
>>>
>>>
>>
>


Re: [obm-l] Problema Legal

2012-05-17 Por tôpico Rogerio Ponce
Para o cavalheiro ganhar a liberdade em ate' 25 dias:

Ele separa as 100 moedas em 2 pilhas (A e B) de 50 moedas.

A cada dia ele passa uma moeda da pilha A para a pilha B.

E ao fim de 25 dias, na pilha A havera' apenas 25 moedas, e ela tera'
passado por alguma situacao de igualdade entre as suas moedas magicas (ou
nao-magicas), e as moedas magicas (ou nao-magicas) da pilha B.

Vejamos como funciona:

1) Se na pilha A houver 25 moedas magicas, entao o cavalheiro ganha a
liberdade imediatamente (pois tambem havera'  25 moedas magicas na pilha B).

2) Se na pilha A houver mais de 25 moedas magicas, entao, em algum dos 25
dias subsequentes, esse numero tera' sido reduzido para no maximo 25 moedas
magicas. Portanto, em algum momento acontecera' a igualdade entre as moedas
magicas das duas pilhas.

3) Se na pilha A houver menos que 25 moedas magicas, entao havera' mais que
25 moedas nao-magicas na pilha A. Portanto, em algum dos 25 dias dias
subsequentes, acontecera' uma situacao de igualdade entre as moedas
nao-magicas das 2 pilhas.

[]'s
Rogerio Ponce




Em 17 de maio de 2012 15:42, Benedito Tadeu V. Freire
escreveu:

>
> O problema abaixo apareceu na Lista de Problemas do pessoal da Argentina.
>
> Problema
> Um dragão dá 100 moedas a um cavalheiro que ele mantém prisioneiro. A
> metade das moedas são mágicas, mas somente o dragão sabe quais são elas.
> Cada dia, o cavalheiro tem que dividir as 100 moedas em duas pilhas, não
> necessariamente do mesmo tamanho.
> Se algum dia as duas pilhas possuem o mesmo número de moedas mágicas ou as
> pilhas tem o mesmo número de moedas não mágicas, o cavalheiro ganha a
> liberdade.
> Determinar se o cavalheiro pode ganhar sua liberdade em 50 dias ou menos.
> E em 25 dias ou menos?
>
>
> Benedito
> --
> Open WebMail Project (http://openwebmail.org)
>
>


Re: [obm-l] Fibonacci

2012-04-07 Por tôpico Rogerio Ponce
Ola'  Gabriel,
se cada casal viver por k+0.5 meses (0.5 e' para nao haver confusao
sobre a geracao de descendentes no momento em que o casal morre),
entao basta voce subtrair a quantidade de coelhos com idade igual ou
mais velhos que k+1 meses.
Assim, a resposta para o seu problema seria
F(n) - F(n-k-1)

[]'s
Rogerio Ponce

PS: para quem nao sabe, o problema dos coelhos e' o seguinte:
" Coloca-se um casal de coelhos recem-nascidos em um jardim.
Sabendo-se que a cada mes, a partir dos dois meses de idade, cada
casal de coelhos da' origem a um novo casal, quantos casais de coelhos
havera' no jardim ao fim de n meses? "

Resposta:
 Ao final do mes n, havera' F(n) casais, onde
 F(0)=1
 F(1)=1
 F(n)=F(n-1)+F(n-2)



Em 07/04/12, Gabriel Guedes escreveu:
> Caros colegas da lista,
> alguem conhece um texto sobre o problema dos coelhos  de Fibonacci,
> mas que troque a hipótese dos coelhos nunca morrerem, por uma hipótese
> dos coelhos morrerem após um determinado período de tempo?
> Atenciosamente,
> Gabriel Guedes
>

=
Instruções para entrar na lista, sair da lista e usar a lista em
http://www.mat.puc-rio.br/~obmlistas/obm-l.html
=


Re: [obm-l] Limite x^1/x

2012-04-05 Por tôpico Rogerio Ponce
Oi Joao,
reescrevendo o "x" como "e ^ ln(x)", o que queremos calcular e'
  e ^ [ ln(x) * 1/x ] , quando x->inf.

Agora, acreditando que o limite da funcao seja a funcao do limite,
basta calcularmos o limite de
  ln(x)/x , quando x->inf.

Aplicando LHopital, basta derivarmos numerador e denominador, obtendo
  (1/x) / 1 , que vale zero quando x-> infinito.

Portanto, o limite procurado vale
  e^0 = 1

[]'s
Rogerio Ponce


Em 05/04/12, João Maldonado escreveu:
>
> Como posso provar o limite x^(1/x),   x-> infinito?
>
>
> Acho que consegui uma prova, mais ficou bem complexaVocês podem ver se tem
> algum erro?
> Primeiramente uso o limite (1+n/x)^x , x-> Infinito = e^nDaí vem a parte
> meio conceitual:Vamos definir (por falta de palavras)  um número infinital
> como um número que tende ao infinito e um número normal como um número que
> não tende ao infinitoSe n é infinital (Qualquer que seja esse infinito,
> digamos uma fração do infinito do x, por exemplo, x/100)O limite tende ao
> infinito
> Se n é normal o limite é normal
> Além disso é óbvio que x^(1/x)>1 para x>1
> Vamos supor agora que  limite x^(1/x),   x-> infinito = k, k>1Sendo k =
> 1+k', k'>1 normal
> É óbvio que  k' pode ser escrito como a divisão de dois infinitais, digamos
> n/x
> Temos x = (1+n/x)^x, x-> infinito -> x = e^n = e^(k'.x) = e^k'^x
> Vamos provar agora que e^(k'.x) > x, se x tende ao infinito
> Derivando a função e^(k'.x)-x,  temos k'.e^  (k'.x)-1, que é crescente
> Além disso quando x = ln|1/k'|/k', temos coeficiente angular de 45º e quando
> x=x1=ln(3^(1/2)/k')/k' temos coeficiente angular de 60º, Logo em algum x
> normal  x,  se x
> tende ao infinito,  qualquer que seja  esse k' normal. Como k' pode ser tão
> pequeno como queiramos, o  limite tem que tender a 1
>
> Isso está certo?Além disso tem alguma prova mais fácil?
> []'sJoão

=
Instruções para entrar na lista, sair da lista e usar a lista em
http://www.mat.puc-rio.br/~obmlistas/obm-l.html
=


Re: [obm-l] Desigualdade

2012-04-05 Por tôpico Rogerio Ponce
Ola' Marcone,
quando da' para ser aplicada, a inducao e' uma otima ferramenta.
Mas, neste problema, eu nao vi como facilitar alguma coisa atraves da
inducao.

[]'s
Rogerio Ponce

Em 5 de abril de 2012 09:14, marcone augusto araújo borges <
marconeborge...@hotmail.com> escreveu:

>  Uma solução bastante interessante
> O victor sugeriu indução,um recurso excelente
> Tenho a impressão de que a indução em certa situações pode esconder a
> ´´beleza´´
> Por exemplo, a demonstração da fórmula da soma dos cubos,eu vi agumas
> maneiras  mais ´´bonitas´´do que a indução.
>
> Date: Thu, 5 Apr 2012 01:27:33 -0300
> Subject: Re: [obm-l] Desigualdade
> From: abrlw...@gmail.com
> To: obm-l@mat.puc-rio.br
>
> Ola' Joao,
> a desigualdade vale para qualquer n>0.
>
> Sabemos que para qualquer k:
>  (k+1)*(k-1) / (k*k) < 1
>
> Logo, para qualquer n inteiro positivo, temos:
> 1*3 / (2*2)  < 1
> 3*5 / (4*4)  < 1
> 5*7 / (6*6)  < 1
> ...
> (2n-3)*(2n-1) / [(2n-2)*(2n-2)] < 1
>
> Alem disso, como (2n-1) / (2n) < 1
> também podemos escrever que
> (2n-1) / (2n * 2n) < 1 / (2n)
>
> Multiplicando as inequacoes acima, vem:
> { [1*3*5*...*(2n-1)] ^ 2 } / { [2*4*6*...*(2n)] ^2 }  < 1/(2n)
>
> FInalmente, aplicando raiz quadrada aos dois lados da expressao, obtemos:
> [1*3*5*...*(2n-1)] / [2*4*6*...*(2n)]  < 1 / sqrt(2n)
>
> []'s
> Rogerio Ponce
>
>
>
>  Em 4 de abril de 2012 20:03, João Maldonado 
> escreveu:
>
>  Como provar que (1.3.5.7...2n-1)/(2.4.6...2n) <1/sqrt(2n), para o caso
> n=50 (pergunta da minha prova)?
>
> Isso vale para qualquer inteiro maior que 1 ?
>
>
> []s
> Joao
>
>
>


Re: [obm-l] Desigualdade

2012-04-04 Por tôpico Rogerio Ponce
Ola' Joao,
a desigualdade vale para qualquer n>0.

Sabemos que para qualquer k:
 (k+1)*(k-1) / (k*k) < 1

Logo, para qualquer n inteiro positivo, temos:
1*3 / (2*2)  < 1
3*5 / (4*4)  < 1
5*7 / (6*6)  < 1
...
(2n-3)*(2n-1) / [(2n-2)*(2n-2)] < 1

Alem disso, como (2n-1) / (2n) < 1
também podemos escrever que
(2n-1) / (2n * 2n) < 1 / (2n)

Multiplicando as inequacoes acima, vem:
{ [1*3*5*...*(2n-1)] ^ 2 } / { [2*4*6*...*(2n)] ^2 }  < 1/(2n)

FInalmente, aplicando raiz quadrada aos dois lados da expressao, obtemos:
[1*3*5*...*(2n-1)] / [2*4*6*...*(2n)]  < 1 / sqrt(2n)

[]'s
Rogerio Ponce


Em 4 de abril de 2012 20:03, João Maldonado
escreveu:

> Como provar que (1.3.5.7...2n-1)/(2.4.6...2n) <1/sqrt(2n), para o caso
> n=50 (pergunta da minha prova)?
>
> Isso vale para qualquer inteiro maior que 1 ?
>
>
> []s
> Joao
>


Re: [obm-l] Geometria

2012-04-01 Por tôpico Rogerio Ponce
Ola' Marcone,
pelo teorema de Menelaus, temos o seguinte:
AD * BE * CM = BD * CE * AM
ou seja,
9 * BE * 3 = 3 * CE * 3 ,
de onde 3BE=CE .
Assim, a altura de BED vale 1/4 da altura de BCA.
Como sua base vale a metade, a relacao entre as areas vale 1/8.

[]'s
Rogerio Ponce

Em 1 de abril de 2012 12:46, marcone augusto araújo borges <
marconeborge...@hotmail.com> escreveu:

>  seja ABC um triangulo equilatero cujos lados medem 6.seja MD o
> segmento que intersecta o lado BC no ponto E,sendo M ponto medio de AC e D
> um ponto do prolongamento do lado AB
> B está entre A e D e BD mede 3.Qual a razao entre a area do triangulo BDE
> e do triangulo ABC
>
> Encontrei uma soluçao apenas por geometria analitica ,mas gostaria de uma
> outra
> Alguem poderia ajudar?
>


[obm-l] Re: [obm-l] RE: [obm-l] Re: [obm-l] Permutação circular

2012-02-08 Por tôpico Rogerio Ponce
Ola' pessoal,
vou usar os simbolos "+" significando uniao, e "^" significando
intersecao entre alguns conjuntos formados pelos casais A,B,C,D e E.

Usando-se o "principio da inclusao-exclusao" sobre os conjuntos
formados por todas as permutacoes em que cada casal aparece "junto" (o
homem ao lado da esposa), podemos escrever o seguinte:

#permut(A+B+C+D+E) = [C(5,1) * #permut(A)] - [C(5,2) * #permut(A^B)] +
[C(5,3) * #permut(A^B^C)] - [C(5,4) * #permut(A^B^C^D)] + [C(5,5) *
#permut(A^B^C^D^E)]

onde:
#permut(A+B+C+D+E) = cardinalidade da uniao dos conjuntos em que algum
casal aparece junto.
#permut(A) = numero de permutacoes em que o casal A aparece junto.
#permut(A^B) = numero de permutacoes em que o casal A aparece junto e
o casal B aparece junto.
...
#permut(A^B^C^D^E) = numero de permutacoes em que cada um dos casais
A,B,C,D e E aparece junto.

Para calcularmos #permut(A^B), por exemplo, basta fixarmos a mulher do
casal A (com 2 opcoes de encaixe do homem), apos o que, temos o casal
B (com 2 opcoes de encaixe do homem) e mais 6 pessoas para permutarmos
( 7! opcoes ).

Dessa forma, o termo generico da expressao para n casais vale:
  [(-1)**(n+1)] * [C(5,n) * 2^n * (9-n)!]

Assim,
#permut(A+B+C+D+E) = [C(5,1) * 2^1 * 8!] - [C(5,2) * 2^2 * 7!] +
[C(5,3) * 2^3 * 6!] - [C(5,4) * 2^4 * 5!] + [C(5,5) * 2^5 * 4!]

Fazendo as contas, obtemos:
#permut(A+B+C+D+E) = [5 * 2 * 40320] - [10 * 4 * 5040] + [10 * 8 *
720] - [5 * 16 * 120] + [1 * 32 * 24]
#permut(A+B+C+D+E) = 250368

Como o total de permutacoes possiveis para os 5 casais vale 9!, o
numero de permutacoes em que nenhum casal aparece junto corresponde a
 9! - #permut(A+B+C+D+E) = 362880 - 250368 = 112512

Assim, o numero de permutacoes procurado vale 112512.

[]'s
Rogerio Ponce




>
> Não entendi seu raciocínio :(
> Fiz  um programa  de computador que  calcula todas as possibilidades da
> função f(x),  para x casais
> obtive:f(0) = 0f(1) = 0f(2) = 2f(3) = 32f(4) =  1488f(5) = 112512
>
> Se  considerar  que  a formação horária é  igual a anti-horária, divida
> ainda por 2
> Até  o f(2)  é fácil de se  achar
> Aqui  vai todas as 192  (que  é  32*6) possibilidades do f(3)  em  linha
> (ou  seja ,ignorando a igualdade por rotação e considerando que o primeiro
> termo senta ao lado do último)Sendo 0,1  o primeiro casal, 2,3 o  segundo...
> ['021435', '021534', '024135', '024153', '024315', '025134', '025143',
> '025314', '031425', '031524', '034125', '034152', '034215', '035124',
> '035142', '035214', '041253', '041352', '042135', '042153', '042513',
> '043125', '043152', '043512', '051243', '051342', '052134', '052143',
> '052413', '053124', '053142', '053412', '120435', '120534', '124035',
> '124053', '124305', '125034', '125043', '125304', '130425', '130524',
> '134025', '134052', '134205', '135024', '135042', '135204', '140253',
> '140352', '142035', '142053', '142503', '143025', '143052', '143502',
> '150243', '150342', '152034', '152043', '152403', '153024', '153042',
> '153402', '203415', '203514', '204135', '204315', '204351', '205134',
> '205314', '205341', '213405', '213504', '214035', '214305', '214350',
> '215034', '215304', '215340', '240315', '240351', '240531', '241305',
> '241350', '241530', '243051', '243150', '250314', '250341', '250431',
> '251304', '251340', '251430', '253041', '253140', '302415', '302514',
> '304125', '304215', '304251', '305124', '305214', '305241', '312405',
> '312504', '314025', '314205', '314250', '315024', '315204', '315240',
> '340215', '340251', '340521', '341205', '341250', '341520', '342051',
> '342150', '350214', '350241', '350421', '351204', '351240&#

[obm-l] Re: [obm-l] Re: [obm-l] RE: [obm-l] Re: [obm-l] Permutação circular

2012-02-06 Por tôpico Rogerio Ponce
Ola' Gabriel,
a restricao que voce propos e'  forte demais...:)

Mas mesmo "capenga", o problema resultante ainda oferece alguma dificuldade
- veja so' :
Depois que voce posiciona a 1a esposa, quais as opcoes que existem para a
2a esposa?
E para as outras?

[]'s
Rogerio Ponce

PS: a 1a esposa tinha 3 opcoes de encaixe. A 2a esposa talvez ainda tenha 3
opcoes de encaixe - vai depender de onde a 1a esposa foi posicionada. E
assim por diante.

==

Em 6 de fevereiro de 2012 00:30, Gabriel Merêncio <
gmerencio.san...@gmail.com> escreveu:

> Desculpe se minha resolução não foi muito rigorosa, admito que me guiei
> mais pela intuição... Pelo visto, com resultados pouco positivos.
>
> Mas João, admitindo a restrição adicional de que dois homens não podem
> sentar juntos (ou seja, todo homem senta ao lado de duas mulheres),
> acredito que seja possível resolver facilmente por análise combinatória.
> Primeiro arranja-se os 5 homens de forma circular (separando cada um por
> uma posição vazia), o que pode ser feito de 5!/5 = 4! = 24 maneiras. Uma
> esposa pode ocupar 3 das 5 posições vazias, já que 2 ficam ao lado do
> marido, então há 3! = 6 maneiras de distribuí-la. Portanto, 24 * 6 = 144
> possibilidades no total. Generalizando, a fórmula geral seria f(x) = (x -
> 1)! * (x - 2)!, sendo x o número de casais.
>
> 2012/2/5 João Maldonado 
>
>>
>> Não entendi seu raciocínio :(
>>
>> Fiz  um programa  de computador que  calcula todas as possibilidades da
>> função f(x),  para x casais
>>
>> obtive:
>> f(0) = 0
>> f(1) = 0
>> f(2) = 2
>> f(3) = 32
>> f(4) =  1488
>> f(5) = 112512
>>
>> Se  considerar  que  a formação horária é  igual a anti-horária, divida
>>  ainda por 2
>>
>> Até  o f(2)  é fácil de se  achar
>>
>> Aqui  vai todas as 192  (que  é  32*6) possibilidades do f(3)  em  linha
>>  (ou  seja ,ignorando a igualdade por rotação e considerando que o primeiro
>> termo senta ao lado do último)
>> Sendo 0,1  o primeiro casal, 2,3 o  segundo...
>>
>> ['021435', '021534', '024135', '024153', '024315', '025134', '025143',
>> '025314', '031425', '031524', '034125', '034152', '034215', '035124',
>> '035142', '035214', '041253', '041352', '042135', '042153', '042513',
>> '043125', '043152', '043512', '051243', '051342', '052134', '052143',
>> '052413', '053124', '053142', '053412', '120435', '120534', '124035',
>> '124053', '124305', '125034', '125043', '125304', '130425', '130524',
>> '134025', '134052', '134205', '135024', '135042', '135204', '140253',
>> '140352', '142035', '142053', '142503', '143025', '143052', '143502',
>> '150243', '150342', '152034', '152043', '152403', '153024', '153042',
>> '153402', '203415', '203514', '204135', '204315', '204351', '205134',
>> '205314', '205341', '213405', '213504', '214035', '214305', '214350',
>> '215034', '215304', '215340', '240315', '240351', '240531', '241305',
>> '241350', '241530', '243051', '243150', '250314', '250341', '250431',
>> '251304', '251340', '251430', '253041', '253140', '302415', '302514',
>> '304125', '304215', '304251', '305124', '305214', '305241', '312405',
>> '312504', '314025', '314205', '314250', '315024', '315204', '315240',
>> '340215', '340251', '340521', '341205', '341250', '341520', '342051',
>> '342150', '350214', '350241', '350421', '351204', '351240', '351420',
>> '352041', '352140', '402153', '402513', '

Re: [obm-l] x tende a (mais) infinito

2012-02-01 Por tôpico Rogerio Ponce
Oi Pedro,
sim, o "mais" fica subentendido.

[]'s
Rogerio Ponce

PS: o meu "epsilon" da mensagem anterior foi tao pequeno que o "i"
desapareceu...

-

Em 01/02/12, Pedro Chaves escreveu:
>
> Obrigado, Rogerio!
>
> O que pergunto, na verdade, é se limite de f(x) ( quando x tende a infinito)
> significa o mesmo que limite de f(x) (quando x tende a mais infinito).
> Um abraço!
> Pedro
> 
>
> Date: Tue, 31 Jan 2012 22:56:57 -0200
> Subject: Re: [obm-l] x tende a (mais) infinito
> From: abrlw...@gmail.com
> To: obm-l@mat.puc-rio.br
>
> Ola' Pedro,
> se o limite para x real existir, então os dois limites existem e sao iguais.
> (para x real suficientemente grande, a distancia de f(x) para o limite fica
> menor que qualquer epslon, o que garante a convergencia tambem para x
> natural)
>
>
> Entretanto, pode acontecer de existir apenas o limite para x natural.
> Exemplo:
>  f(x)=sin(pi*x)
>
> []'s
> Rogerio Ponce
>
> Em 31 de janeiro de 2012 11:37, Pedro Chaves 
> escreveu:
>
>
>
> Caros Colegas,
>
>
>
>
>
> Se x representa um número natural ou um número real, existe diferença entre
> o limite de f(x) quando x tende a mais infinito e o limite de f(x) quando x
> tende a infinito?
>
>
>
> Abraços!
>
> Pedro Chaves
>
> =
>
> Instruções para entrar na lista, sair da lista e usar a lista em
>
> http://www.mat.puc-rio.br/~obmlistas/obm-l.html
>
> =
>
>
>   
> =
> Instruções para entrar na lista, sair da lista e usar a lista em
> http://www.mat.puc-rio.br/~obmlistas/obm-l.html
> =
>

=
Instruções para entrar na lista, sair da lista e usar a lista em
http://www.mat.puc-rio.br/~obmlistas/obm-l.html
=


Re: [obm-l] x tende a (mais) infinito

2012-01-31 Por tôpico Rogerio Ponce
Ola' Pedro,
se o limite para x real existir, então os dois limites existem e sao iguais.
(para x real suficientemente grande, a distancia de f(x) para o limite fica
menor que qualquer epslon, o que garante a convergencia tambem para x
natural)

Entretanto, pode acontecer de existir apenas o limite para x natural.
Exemplo:
 f(x)=sin(pi*x)

[]'s
Rogerio Ponce

Em 31 de janeiro de 2012 11:37, Pedro Chaves escreveu:

>
> Caros Colegas,
>
>
> Se x representa um número natural ou um número real, existe diferença
> entre o limite de f(x) quando x tende a mais infinito e o limite de f(x)
> quando x tende a infinito?
>
> Abraços!
> Pedro Chaves
> =
> Instruções para entrar na lista, sair da lista e usar a lista em
> http://www.mat.puc-rio.br/~obmlistas/obm-l.html
> =
>


Re: [obm-l] Probabilidade

2012-01-30 Por tôpico Rogerio Ponce
Ola' Joao,
chamemos de X(k) o numero de caras obtidas pelo jogador X em k lancamentos,
e chamemos de P[z] a probabilidade do evento z ocorrer.
Assim, nosso problema é calcular o valor de P[A(n+1) > B(n)]

Agora, imagine que "B" tenha feito n lances, e que em seguida, "A" tambem
tenha feito n lances.
Ainda falta "A" fazer um lance, e as seguintes situacoes podem acontecer:
1) "A" ja' obteve mais caras que "B" com os n primeiros lances, e sua
chance de ultrapassar B vale 100%.
2) "A" esta' empatado com "B", e tem 50% de chance de obter uma cara.
3) "A" tem menos caras que "B", e tem 0% de chance de ultrapassar B.

Portanto, somando-se as probabilidades das 2 primeiras situacoes, obtemos
P[A(n+1)>B(n)]  =  100% * P[A(n)>B(n)] + 50% * P[A(n)=B(n)]

Sabemos que
  P[A(n)B(n)] = 1
Por simetria,
  P[A(n)>B(n)]  =  P[B(n)>A(n)]
de forma que
2*P[A(n)>B(n)]  + P[A(n)=B(n)] = 1
ou seja,
P[A(n)=B(n)]  =  1 -  2* P[A(n)>B(n)]

Aplicando essa relacao 'a expressao anterior, obtemos
 P[A(n+1)>B(n)] = 50%

Ou seja, a probabilidade de "A" obter mais caras que "B" e' de 50%.

[]'s
Rogerio Ponce



2012/1/18 João Maldonado 

>
>  Se A e B lançam respectivamente n + 1 e n moedas não-viciadas, qual é a
> probabilidade Pn de que A obtenha mais “caras” do que B?
>
> []`s
> Joao
>


Re: [obm-l] probabilidade

2011-11-14 Por tôpico Rogerio Ponce
Ola' Marcone,
esse problema e' equivalente ao calculo da probabilidade P(n) de ocorrer um
sorteio valido numa reuniao de n amigos ocultos.
(sorteio valido de n amigos ocultos e' aquele em que ninguem sorteia a si
mesmo).

Segue uma solucao antiga na lista:


Primeiramente, em um sorteio qualquer, existem sub-grupos do tipo "A
sorteia B, que sorteia C, que sorteia...que sorteia A" , formando um
"loop". Chamemos de "cadeia" essa sequencia de pessoas.

Entao, seja V(n) o numero de sorteios validos com "n" pessoas.

Quando acrescentamos a enesima-primeira pessoa a um grupo com "n" pessoas,
um sorteio valido qualquer correspondera'  as seguintes situacoes:

a) essa pessoa forma uma cadeia com mais de 2 elementos.
b) essa pessoa forma uma cadeia com apenas 2 elementos (ela e uma 2a.
pessoa fazem uma troca mutua de presentes).

No caso "a", podemos considerar que essa pessoa e' inserida em alguma das
cadeias existentes num sorteio valido com apenas "n" pessoas.
No caso "b" , cada sorteio pode ser obtido a partir da escolha do 2o.
elemento, e entao formando-se todos os sorteios validos possiveis com (n-1)
elementos.

Dessa forma, o numero de sorteios validos do tipo "a" vale n*V(n) .
Repare que essa nova pessoa pode ser inserida logo após uma pessoa qualquer
dentre as "n" existentes.

E o numero de sorteios validos do tipo "b" vale n*V(n-1) .
Repare que essa nova pessoa pode fazer par com qualquer uma dentre as "n"
existentes, enquanto as outras (n-1) se organizam como um sorteio valido de
(n-1) elementos.


Assim, V(n+1) = n*V(n) + n*V(n-1)
Fazendo V(n) = n! * W(n) , obtemos a equacao de diferencas, linear e
homogenea, do 1o grau:
[W(n+1) - W(n)] + 1/(n+1) * [W(n) - W(n-1)] = 0

Portanto, a solucao geral e'
W(n+1) - W(n) = C * (-1)^(n+1)/(n+1)!


Como V(1)=0 e V(2)=1 , entao C=1 , pois W(1)=0 e W(2)=1/2 , que nos leva a
W(n+1) = W(n) + (-1)^(n+1)/(n+1)!


Como o numero de sorteios possiveis e' n! , a probabilidade de sorteios
validos com "n" pessoas e' P(n)= V(n)/n! .
Logo, P(n) = W(n) , ou seja,

P(n) = P(n-1) + (-1)^n/n! , onde P(1)=0
de modo que
P(n) = 0 + 1/2! - 1/3! +...+ (-1)^n/n!


Alem disso, e' facil verificar que quando "n" cresce, P(n) converge para
P = 0 + 1/2! - 1/3! + 1/4! ... = 1/e

___

No nosso caso, como n=4, a resposta e'
 P(4)= 1/2! - 1/3! + 1/4! = 3/8

[]'s
Rogerio Ponce




Em 14 de novembro de 2011 22:54, marcone augusto araújo borges <
marconeborge...@hotmail.com> escreveu:

>  tenho 4 cartoes ,cada um para ser destinado a uma determinada
> pessoa.tenho os 4 endereços,mas não sei qual é o endereço de ninguem.qual é
> a probabilidade de que todos os cartoes  sejam enviados para as pessoas
> erradas
>
> eu fiz e encontrei 3/8
> calculei quantas maneiras poderia enviar exatamente 1 certo,exatamente
> 2,exatamente 4
> deu 15=8+6+1,respectivamente
> dai,total 24(4x3x2),menos 15,deu 9
> 9/24 = 3/8
> agradeço por uma solução diferente
>
>


Re: [obm-l] A pulga e o elastico

2011-10-07 Por tôpico Rogerio Ponce
Ola' Bernardo,
como sabemos, pulgas matematicas sao muito persistentes...

Expandindo a sua (correta) solucao - para ninguem ficar no vacuo - vem:

A pulga avanca 1/100 do elastico no primeiro salto, 1/200 no segundo, 1/300
no terceiro, e assim por diante.
Depois de N saltos, a pulga avancou 1/100 * ( 1 + 1/2 + 1/3 + ... + 1/N ) do
elastico.
Assim, queremos calcular o N para o qual
1 + 1/2 + 1/3 + ... + 1/N vale aproximadamente 100.

Usando a aproximacao para a soma dos N primeiros termos da serie harmonica (
vide http://pt.wikipedia.org/wiki/Constante_de_Euler-Mascheroni ), obtemos o
resultado do Bernardo.

Bernardo, eu sugeri esse problema a um amigo faz uns 4 anos, e nao me lembro
qual a origem dele...

Abracao,
Rogerio Ponce


Em 7 de outubro de 2011 10:53, Bernardo Freitas Paulo da Costa <
bernardo...@gmail.com> escreveu:

> 2011/10/7 Rogerio Ponce :
> > hehehe...acontece que a coitadinha e' esperta, e pega uma carona na
> esticada
> > do elastico.
> É a super-pulga (que se apóia nos vazio do elástico)!
>
> > Instante -0,01 s: falta percorrer 1,0 m
> > Instante  0,99 s: falta percorrer 1,98000 m
> > Instante  1,99 s: falta percorrer 2,95500 m
> > Instante  2,99 s: falta percorrer 3,92666... m
>
> Eu gosto de pensar que o elástico não muda de tamanho: afinal, se ele
> dilata uniformemente, a pulga não muda de posição "relativa" sobre o
> elástico. O que acontece é que a pulga vai ficando cada vez mais
> cansada... No primeiro pulo, ela avança de 1/100 do elástico. No
> segundo, 1/200. No terceiro, 1/300. E quando enfim ela chegar no 1.5 *
> 10^(43) segundo (mais ou menos), ela chegará ao fim.
>
> Voltando a situação original: note que se duas "moléculas de elástico"
> estivessem a 10^(-9) m de distância (o que é perto demais, enfim),
> agora elas estarão a 1.5 * 10^(34) metros que dá uns 50 * 10^24
> anos-luz, o que é muito mais do que o diâmetro atual do universo. É
> claro que 1.5 10^43 segundos é também muito, muito, muito mais do que
> a idade do universo (~ 4 * 10^17 segundos). Lembrando que 10^-35 é
> mais ou menos a distância de Planck, isso mostra quão grande ficam as
> coisa no "fim do caminho da pulga".
>
> > []'s
> > Rogerio Ponce
> >
> >
> >
> > Em 7 de outubro de 2011 09:06, geonir paulo schnorr <
> geonirpa...@gmail.com>
> > escreveu:
> >>
> >> Instante 0 s: falta 1 m
> >> Instante 1 s: falta 0,99 m + 1 m
> >> Instante 2 s: falta 0,98 m + 2 m
> >> Instante 3 s: falta 0,97 m + 3 m
> >> ...   ...   ...
> >> ou seja, realmente, nunca chegará ao final da viagem, coitadinha..
>
> Rogerio: da onde é esse problema ? Eu tenho quase certeza que alguém
> já tinha falado um dia para mim, mas eu esqueci...
> --
> Bernardo Freitas Paulo da Costa
>
> =
> Instruções para entrar na lista, sair da lista e usar a lista em
> http://www.mat.puc-rio.br/~obmlistas/obm-l.html
> =
>


[obm-l] Re: [obm-l] Re: [obm-l] Re: [obm-l] Combinatória em uma grade

2011-10-07 Por tôpico Rogerio Ponce
Hahaha, e' verdade!
era para eu ter escrito 6 ** 5 caminhos diferentes.
[]'s
Rogerio Ponce

Em 7 de outubro de 2011 10:17, Bernardo Freitas Paulo da Costa <
bernardo...@gmail.com> escreveu:

> 2011/10/7 Rogerio Ponce :
> > Ola' Azincourt,
> > cada seta horizontal pode ser colocada em 6 "alturas" diferentes.
> > Como sao 5 setas horizontais, existem 6 * 5 = 30 caminhos diferentes.
> 6^5 = muito mais.
>
> Mas a idéia é essa :)
>
> > []'s
> > Rogerio Ponce
> >
> > Em 6 de outubro de 2011 20:32, Azincourt Azincourt <
> aazinco...@yahoo.com.br>
> > escreveu:
> >>
> >> Boa noite!
> >> Como posso resolver o seguinte problema: de quantas maneiras podemos ir
> de
> >> A até B sobre a seguinte grade sem passar duas vezes pelo mesmo local e
> sem
> >> mover-se para a esquerda? A figura em anexo mostra um caminho possível.
> >> (problema e figura retirados de
> conesul2006.tripod.com/Material/comb.pdf )
> >> Eu sei resolver um problema parecido, no qual não há as setas para baixo
> –
> >> envolvia a permutação com repetição das setas “para cima” e “para
> baixo”. No
> >> entanto, não consegui achar resolução análoga para este problema
> (acabava em
> >> uma expressao complicada, que nao parecia ser simplificável). Como
> >> resolvê-lo?
> >> Muito obrigado!
> >>
> >
> >
>
>
>
> --
> Bernardo Freitas Paulo da Costa
>
> =
> Instruções para entrar na lista, sair da lista e usar a lista em
> http://www.mat.puc-rio.br/~obmlistas/obm-l.html
> =
>


Re: [obm-l] A pulga e o elastico

2011-10-07 Por tôpico Rogerio Ponce
Ola' JR,
(complementando minha resposta)
Caso o comprimento do salto aumentasse com a expansao do elastico, a sua
segunda hipotese seria verdadeira.
Entretanto, somente o primeiro salto recebe todas as expansoes juntamente
com o elastico.
O segundo salto "perdeu" a primeira expansao.
O terceiro salto "perdeu" as 2 primeiras expansoes, e assim por diante.
Dessa forma, cada salto da pulga e' relativamente menor que o salto
anterior, mas eventualmente ela chega ao final da viagem.
Falta so' equacionar para descobrirmos...

[]'s
Rogerio Ponce


Em 7 de outubro de 2011 10:13, Rogerio Ponce  escreveu:

> Ola' JR,
> imagine que logo apos cada salto, a pulga pintasse o elastico no ponto em
> contato com seus pes (sim, esta pulga e' pontual).
> Me parece razoavel que a expansao do elastico "carregue" a pulga para mais
> longe da origem juntamente com a marca que ela fez, concorda?
>
> []'s
> Rogerio Ponce
>
>
> Em 7 de outubro de 2011 09:44, J. R. Smolka escreveu:
>
>>  Ok Rogério,
>>
>> Então eu consigo imaginar dois cenários para o problema: [a] existe um
>> referencial galileano absoluto para o elástico e para a pulga - neste caso
>> provavelmente a resposta já apresentada por outros colegas da lista (a pulga
>> não chega nunca ao final do elástico) está correta; ou [b] considerar o
>> elástico como o espaço-tempo da pulga, onde o processo de expansão afeta não
>> só o elástico, mas também a própria pulga e seu referencial de medida de
>> distância - neste caso me parece que o tempo seria idêntico ao caso trivial
>> da não expansão do elástico.
>>
>> O que você acha disso?
>>
>> [ ]'s
>>
>> *J. R. Smolka*
>>
>> *Em 07/10/2011 08:29, Rogerio Ponce escreveu:*
>>
>> como todo elastico bem comportado, ele estica uniformemente entre as 2
>> extremidades.
>>
>> *Em 6 de outubro de 2011 21:28, J. R. Smolka escreveu:
>> *
>>
>>>  Depende... para qual lado o elástico estica? No mesmo sentido ou no
>>> sentido contrário ao deslocamento da pulga?
>>>
>>> *Em 06/10/2011 18:04, Rogerio Ponce escreveu:*
>>>
>>> Ola' pessoal,
>>> no instante zero, uma pulga inicia uma viagem sobre um elastico de
>>> 1metro, indo de uma extremidade para a outra, dando saltos de 1cm de
>>> comprimento a cada segundo.
>>> Entretanto, meio segundo apos o inicio da viagem, o elastico comeca a
>>> sofrer um puxao a cada segundo, de tal forma que ele estica mais 1 metro a
>>> cada puxada.
>>> Assim, com saltos e puxoes intercalados, pergunta-se:
>>> - Quanto tempo levara' a viagem?
>>>
>>>
>


Re: [obm-l] A pulga e o elastico

2011-10-07 Por tôpico Rogerio Ponce
Ola' JR,
imagine que logo apos cada salto, a pulga pintasse o elastico no ponto em
contato com seus pes (sim, esta pulga e' pontual).
Me parece razoavel que a expansao do elastico "carregue" a pulga para mais
longe da origem juntamente com a marca que ela fez, concorda?

[]'s
Rogerio Ponce


Em 7 de outubro de 2011 09:44, J. R. Smolka  escreveu:

>  Ok Rogério,
>
> Então eu consigo imaginar dois cenários para o problema: [a] existe um
> referencial galileano absoluto para o elástico e para a pulga - neste caso
> provavelmente a resposta já apresentada por outros colegas da lista (a pulga
> não chega nunca ao final do elástico) está correta; ou [b] considerar o
> elástico como o espaço-tempo da pulga, onde o processo de expansão afeta não
> só o elástico, mas também a própria pulga e seu referencial de medida de
> distância - neste caso me parece que o tempo seria idêntico ao caso trivial
> da não expansão do elástico.
>
> O que você acha disso?
>
> [ ]'s
>
> *J. R. Smolka*
>
> *Em 07/10/2011 08:29, Rogerio Ponce escreveu:*
>
> como todo elastico bem comportado, ele estica uniformemente entre as 2
> extremidades.
>
> *Em 6 de outubro de 2011 21:28, J. R. Smolka escreveu:
> *
>
>>  Depende... para qual lado o elástico estica? No mesmo sentido ou no
>> sentido contrário ao deslocamento da pulga?
>>
>> *Em 06/10/2011 18:04, Rogerio Ponce escreveu:*
>>
>> Ola' pessoal,
>> no instante zero, uma pulga inicia uma viagem sobre um elastico de 1metro,
>> indo de uma extremidade para a outra, dando saltos de 1cm de comprimento a
>> cada segundo.
>> Entretanto, meio segundo apos o inicio da viagem, o elastico comeca a
>> sofrer um puxao a cada segundo, de tal forma que ele estica mais 1 metro a
>> cada puxada.
>> Assim, com saltos e puxoes intercalados, pergunta-se:
>> - Quanto tempo levara' a viagem?
>>
>>


  1   2   3   4   5   6   7   >